TN - Uepa

Transcrição

TN - Uepa
Marília Brasil Xavier
REITORA
Prof. M. Sc. Rubens Vilhena Fonseca
COORDENADOR GERAL DOS CURSOS DE MATEMÁTICA
MATERIAL DIDÁTICO
COLABORAÇÃO
Maria da Glória Costa Lima
Cleyton Isamu Muto
EDITORAÇÃO ELETRONICA
Odivaldo Teixeira Lopes
ARTE FINAL DA CAPA
Odivaldo Teixeira Lopes
REALIZAÇÃO
Dados Internacionais de Catalogação na Publicação (CIP)
F676t Fonseca, Rubens Vilhena
Teoria dos números / Rubens Vilhena Fonseca – Belém:
UEPA / Centro de Ciências Sociais e Educação, 2011.
204 p.; iI.
ISBN: 978-85-88375-66-6
1.Teoria dos números algébricos. I. Universidade
Estadual do Pará. II. Título.
CDU: 511.68
CDD: 512.7
Índice para catálogo sistemático
1. Teoria dos números algébricos: 511.68
BELÉM – PARÁ – BRASIL
- 2011 -
SUMÁRIO
Capítulo 1:...............................................................................................................................................9
NÚMEROS INTEIROS – NOÇÕES FUNDAMENTAIS ...................................................................................9
1.1 – NÚMEROS INTEIROS ....................................................................................................................................... 10
1.2 – PROPRIEDADES DOS INTEIROS ........................................................................................................................ 11
1.3 – VALOR ABSOLUTO DE UM INTEIRO ................................................................................................................ 12
1.4 – REPRESENTAÇÃO DOS INTEIROS EM OUTRAS BASES ...................................................................................... 14
1.5 – FATORIAL E PRINCÍPIO FUNDAMENTAL DA CONTAGEM .................................................................................. 15
1.6 – PRINCÍPIO FUNDAMENTAL DA CONTAGEM - PFC ............................................................................................ 16
1.7 – NÚMERO BINOMIAL ........................................................................................................................................ 17
1.8 – NÚMEROS BINOMIAIS COMPLEMENTARES...................................................................................................... 18
1.9 – NÚMEROS BINOMIAIS CONSECUTIVOS ............................................................................................................ 18
1. 10 – PISO, TETO E NINT DE UM NÚMERO REAL. ................................................................................................ 20
1.11 – O PRINCÍPIO DA CASA DOS POMBOS (PRINCÍPIO DAS GAVETAS DE DIRICHLET) .............................................. 24
1.12 – CAOS FATORIAL: !N. ..................................................................................................................................... 25
1.13 – LEFT FATORIAL: L!N ..................................................................................................................................... 26
EXERCÍCIOS............................................................................................................................................................ 27
Capítulo 2:.............................................................................................................................................30
INDUÇÃO MATEMÁTICA ........................................................................................................................30
2.1 – ELEMENTO MÍNIMO DE UM CONJUNTO DE INTEIROS ...................................................................................... 30
2.2 – PRINCÍPIO DA BOA ORDENAÇÃO .................................................................................................................... 31
2.3 – PRINCÍPIO DE INDUÇÃO FINITA. ...................................................................................................................... 32
2.4 – INDUÇÃO MATEMÁTICA ................................................................................................................................ 33
2.5. EXEMPLOS DE DEMONSTRAÇÃO POR INDUÇÃO MATEMÁTICA .......................................................................... 35
2.6 . OUTRAS FORMAS DA INDUÇÃO MATEMÁTICA ................................................................................................ 37
EXERCÍCIOS............................................................................................................................................................ 42
Capítulo 3:.............................................................................................................................................43
SOMATÓRIOS E PRODUTÓRIOS .............................................................................................................43
3.1 . SOMATÓRIOS ................................................................................................................................................. 43
3.2. PROPRIEDADES DOS SOMATÓRIOS................................................................................................................... 44
3.3. PRODUTÓRIOS................................................................................................................................................. 45
3.4. PROPRIEDADES DOS PRODUTÓRIOS ................................................................................................................. 46
Capítulo 4 ..............................................................................................................................................48
DIVISIBILIDADE .....................................................................................................................................48
4.1. RELAÇÃO DE DIVISIBILIDADE EM Z ................................................................................................................. 48
4.2. CONJUNTO DOS DIVISORES DE UM INTEIRO ..................................................................................................... 50
4.3. DIVISORES COMUNS DE DOIS INTEIROS ........................................................................................................... 50
4.4. TEOREMA DA DIVISÃO ................................................................................................................................... 51
4.5. PARIDADE DE UM INTEIRO .............................................................................................................................. 54
EXERCÍCIOS............................................................................................................................................................ 56
Capítulo 5 ..............................................................................................................................................58
MÁXIMO DIVISOR COMUM ...................................................................................................................58
5.1. MÁXIMO DIVISOR COMUM DE DOIS INTEIROS .................................................................................................. 58
5.2. EXISTÊNCIA E UNICIDADE DO MDC. ................................................................................................................ 59
5.3. INTEIROS RELATIVAMENTE PRIMOS (COPRIMOS OU PRIMOS ENTRE SI) ............................................................ 61
5.4. CARACTERIZAÇÃO DO MDC DE DOIS INTEIROS ................................................................................................ 64
5.5. MDC DE VÁRIOS INTEIROS .............................................................................................................................. 64
EXERCÍCIOS............................................................................................................................................................ 65
Capítulo 6 ............................................................................................................................................. 67
ALGORITMO DE EUCLIDES – MÍNIMO MÚLTIPLO COMUM................................................................. 67
6.1. ALGORITMO DE EUCLIDES .............................................................................................................................. 67
6.2 . MÚLTIPLOS COMUNS DE DOIS INTEIROS ......................................................................................................... 74
6.3. MÍNIMO MÚLTIPLO COMUM DE DOIS INTEIROS ................................................................................................ 75
6.5. MMC DE VÁRIOS INTEIROS .............................................................................................................................. 76
EXERCÍCIOS ............................................................................................................................................................ 78
Capítulo 7 ............................................................................................................................................. 79
NÚMEROS PRIMOS ................................................................................................................................ 79
7.1. INTRODUÇÃO .................................................................................................................................................. 79
7.2. NÚMEROS PRIMOS (DO LAT. PRIMUS, PRINCIPAL. PRIME EM INGLÊS) .............................................................. 81
7. 3. TEOREMA FUNDAMENTAL DA ARITMÉTICA. ................................................................................................... 82
7.4. A SEQÜÊNCIA DOS NÚMEROS PRIMOS .............................................................................................................. 84
7.5. O CRIVO DE ERATÓSTENES. .............................................................................................................................. 86
7.6. SEQÜÊNCIA DE INTEIROS CONSECUTIVOS COMPOSTOS .................................................................................... 94
7.7 . CONJECTURAS ................................................................................................................................................ 96
7.8. FÓRMULAS QUE GERAM ALGUNS NÚMEROS PRIMOS........................................................................................ 98
7.9. DECOMPOSIÇÃO DO FATORIAL EM FATORES PRIMOS ..................................................................................... 101
7.10. MÉTODO DA FATORAÇÃO DE FERMAT .......................................................................................................... 105
7. 11 – ALGORITMO DE FERMAT ............................................................................................................................ 105
EXERCÍCIOS .......................................................................................................................................................... 107
Capítulo 8: .......................................................................................................................................... 110
EQUAÇÕES DIOFANTINAS LINEARES ................................................................................................. 110
3.1. GENERALIDADES ........................................................................................................................................... 111
3.2. CONDIÇÃO DE EXISTÊNCIA DE SOLUÇÃO ........................................................................................................ 112
3.3. SOLUÇÕES DA EQUAÇÃO AX + BY = C. ........................................................................................................... 113
EXERCÍCIOS .......................................................................................................................................................... 115
Capítulo 9 ........................................................................................................................................... 117
CONGRUÊNCIAS .................................................................................................................................. 117
9.1. CONGRUÊNCIAS ............................................................................................................................................ 117
9.2. CARACTERIZAÇÃO DE INTEIROS CONGRUENTES ............................................................................................ 117
9.3. PROPRIEDADES DAS CONGRUÊNCIAS ............................................................................................................. 118
9.4. SISTEMAS COMPLETOS DE RESTOS................................................................................................................ 121
9.5 – ARITMÉTICA MÓDULO M .............................................................................................................................. 122
9.6. ADIÇÃO E MULTIPLICAÇÃO EM  m .............................................................................................................. 124
9.7. SUBTRAÇÃO EM  m ...................................................................................................................................... 130
9.8. DIVISÃO EM  m ............................................................................................................................................ 131
9.9. POTENCIAÇÃO EM  m .................................................................................................................................. 135
EXERCÍCIOS .......................................................................................................................................................... 139
Capítulo 10 ......................................................................................................................................... 141
TEOREMAS DE FERMAT, WILSON E EULER .......................................................................... 141
10.1. PEQUENO TEOREMA DE FERMAT ....................................................................................................... 141
EXERCÍCIOS .......................................................................................................................................................... 145
10.2. TEOREMA DE WILSON .................................................................................................................................. 146
EXERCÍCIOS ............................................................................................................................................................................. 149
10.3. TEOREMA DE EULER .................................................................................................................................... 150
10.4. FUNÇÃO TOTIENT (N) ................................................................................................................................. 151
10.5 – CÁLCULO DE (N) ...................................................................................................................................... 152
10.6. RESOLUÇÃO DE CONGRUÊNCIAS LINEARES PELO TEOREMA DE EULER ......................................................... 155
10. 7. RESOLUÇÃO DA EQUAÇÃO (N).................................................................................................................. 156
10.8 – VALÊNCIA DA FUNÇÃO TOTIENTE: N (m) . ............................................................................................ 159
EXERCÍCIOS.......................................................................................................................................................... 160
10.9. TEOREMA CHINÊS DO RESTO (TCR) .............................................................................................................. 161
10.10. POTENCIAÇÃO: UMA APLICAÇÃO DO TEOREMA DE EULER ......................................................................... 165
10.11 – POTENCIAÇÃO: UMA APLICAÇÃO DO TEOREMA CHINÊS DO RESTO (TCR) .................................................. 165
Capítulo 11 ..........................................................................................................................................171
CIFRA DE CÉSAR ..................................................................................................................................171
11.1. FUNÇÕES POLINOMIAIS DE CODIFICAÇÃO .................................................................................................... 174
Capítulo 12 ..........................................................................................................................................179
CIFRA DE VIGENÈRE ...........................................................................................................................179
Capítulo 13 ..........................................................................................................................................182
CIFRA DE HILL.....................................................................................................................................182
Capítulo 14 ..........................................................................................................................................190
RSA .......................................................................................................................................................190
14. 1. PRÉ-CODIFICAÇÃO ...................................................................................................................................... 190
14.2 – CODIFICANDO E DECODIFICANDO ............................................................................................................... 191
14. 3. ASSINATURA DIGITAL UTILIZANDO A CRIPTOGRAFIA RSA .......................................................................... 195
Capítulo 15 ..........................................................................................................................................201
PARTILHA DE SENHAS .........................................................................................................................201
CAPÍTULO 1
NÚMEROS INTEIROS – NOÇÕES FUNDAMENTAIS
Capítulo 1:
NÚMEROS INTEIROS – NOÇÕES
FUNDAMENTAIS
INTRODUÇÃO
A
Teoria dos Números nasceu cerca de 600 anos antes de Cristo quando Pitágoras e
os seus discípulos começaram a estudar as propriedades dos números inteiros. Os
pitagóricos rendiam verdadeiro culto místico ao conceito de número,
considerando-o como essência das coisas. Acreditavam que tudo no universo estava
relacionado com números inteiros ou razões de números inteiros (em linguagem atual,
números racionais). Aliás, na antiguidade a designação número aplicava-se só aos inteiros
maiores do que um.
http://nonio.fc.ul.pt/analise1/cap1/hnum.htm
O conceito de número tomou forma num longo desenvolvimento histórico. A origem e
formulação deste conceito ocorreu simultaneamente com o despontar, entenda-se nascimento,
e desenvolvimento da Matemática. As atividades práticas do homem, por um lado, e as
exigências internas da Matemática por outro determinaram o desenvolvimento do conceito de
número. A necessidade de contar objetos levou ao aparecimento do conceito de número
Natural.
Todas as nações que desenvolveram formas de escrita introduziram o conceito de
número Natural e desenvolveram um sistema de contagem. O desenvolvimento subsequente
do conceito de número prosseguiu principalmente devido ao próprio desenvolvimento da
Matemática. Os números negativos aparecem pela primeira vez na China antiga. Os chineses
estavam acostumados a calcular com duas coleções de barras - vermelha para os números
positivos e preta para os números negativos.No entanto, não aceitavam a idéia de um número
negativo poder ser solução de uma equação. Os Matemáticos indianos descobriram os
números negativos quando tentavam formular um algoritmo para a resolução de equações
quadráticas. São exemplo disso as contribuições de Bramaghupta, pois a aritmética
sistematizada dos números negativos encontra-se pela primeira vez na sua obra. As regras
sobre grandezas eram já conhecidas através dos teoremas gregos sobre subtração, como por
exemplo (a - b)(c - d) = ac + bd - ad - bc, mas os hindus converteram-nas
em regras numéricas sobre números negativos e positivos.
Diofanto (Séc. III) operou facilmente com os números negativos. Eles apareciam
constantemente em cálculos intermédios em muitos problemas do seu "Aritmetika", no
entanto havia certos problemas para o qual as soluções eram valores inteiros negativos como
por exemplo:
4x + 20 = 4 ou 3x – 18 = 5x2
Nestas situações Diofanto limitava-se a classificar o problema de absurdo. Nos séculos XVI e
XVII, muitos matemáticos europeus não apreciavam os números negativos e, se esses
9
CAPÍTULO 1
NÚMEROS INTEIROS – NOÇÕES FUNDAMENTAIS
números apareciam nos seus cálculos, eles consideravam-nos falsos ou impossíveis. Exemplo
deste fato seria Michael Stifel (1487- 1567) que se recusou a admitir números negativos como
raízes de uma equação, chamando-lhes de "numeri absurdi". Cardano usou os números
negativos embora chamando-os de "numeri ficti". A situação mudou a partir do (Séc.XVIII)
quando foi descoberta uma interpretação geométrica dos números positivos e negativos como
sendo segmentos de direções opostas.
http://www.somatematica.com.br/historia.php
1.1 – Números Inteiros
Os números inteiros ou apenas os inteiros são:
..., -3, -2, -1, 0, 1, 2, 3,...
cujo conjunto representa-se pela letra Z, isto é:
Z = {..., -3, -2, -1, 0, 1, 2, 3,...}
Neste conjunto Z destacam-se os seguintes subconjuntos:
1) Conjunto Z* dos inteiros não nulos ( 0 ):
Z* = {x
Z| x
0} { 1, 2, 3,...}
2) Conjunto Z dos inteiros não negativos ( 0 ):
{x Z| x 0} = {0, 1, 2, 3,...}
Z
3) Conjunto Z dos inteiros não positivos ( 0 ):
{x Z| x 0} = {0, -1, -2, -3,...}
Z
4) Conjunto Z* dos inteiros positivos (> 0):
Z*
{x Z | x 0} = {1, 2, 3,...}
5) Conjunto Z* dos inteiros negativos (< 0):
Z*
{x Z | x 0} = {-1, -2, -3,...}
Os inteiros positivos são também denominados inteiros naturais e por isso o conjunto
dos inteiros positivos é habitualmente designado pela letra N (N = Z* ).
10
CAPÍTULO 1
NÚMEROS INTEIROS – NOÇÕES FUNDAMENTAIS
1.2 – Propriedades dos Inteiros
O conjunto Z dos inteiros munido das operações de adição (+) e multiplicação ( . )
possui as propriedades fundamentais que a seguir enumeramos, onde a, b e c são inteiros
quaisquer, isto é, elementos de Z:
1) a + b = b + a e
ab = ba;
2) (a + b) + c = a + (b + c)
e (ab) c = a (bc);
3) 0 + a = a e 1.a = a;
4) –a = (-1) a e
a – a = a + (-a) = 0;
5) a (b + c) = ab + ac;
6) 0.a = 0, e se ab = 0, então a = 0 ou b = 0.
Também existe uma “relação de ordem” entre os inteiros, representada pelo sinal “<
(menor que)”, que possui as seguintes propriedades:
7) Se a
0 , então a > 0 ou a < 0;
8) Se a < b e b < c, então a < c;
9) Se a < b, então a + c < b + c;
10) Se a < b e 0 < c, então ac < bc;
11) Se a < b e c < 0, então bc < ac.
 Destas propriedades podem ser deduzidas muitas outras propriedades dos inteiros.
Exemplo 1.1: Demonstrar: -(a + b) = (-a) + (-b).
Com efeito, temos sucessivamente:
-(a + b) = (-1) (a + b) =
= (-1) a + (-1) b =
= (-a) + (-b)
Exemplo 1.2: Demonstrar que , se x
(Propriedade 4)
(Propriedade 5)
(Propriedade 4)
0 , então 0
x2 .
Com efeito:
1) Se x 0 , então
2) Se x 0 , então
3) Se 0
x 0 ou 0
0.x x.x
0 x2
x , então 0.x x.x
0 x2
x
(Propriedade 7)
(Propriedade 11)
(Propriedade 6)
(Propriedade 10)
(Propriedade 6)
11
CAPÍTULO 1
NÚMEROS INTEIROS – NOÇÕES FUNDAMENTAIS
Nota:
Com o mesmo significado de a < b, escreve-se b > a. Indica-se, de modo
abreviado, que a < b ou a = b por a b . Por exemplo, temos 2 3 , porque 2
< 3, e 2 2 , porque 2 - 2.
Com o mesmo significado de a b , escreve-se b a . Em lugar de a b e
b c também se escreve a b c .
1.3 – Valor absoluto de um Inteiro
Definição 1.1: Chama-se valor absoluto de um inteiro a, o inteiro que se indica por | a | , e tal
que:
a, se a
|a|
0
a, se a < 0
Assim, por exemplo:
e
|3| 3
| 5|
( 5)
5
Consoante a definição de | a | , para todo inteiro a, temos:
|a| 0, |a|²
a² , | a | | a | , a | a |
O valor absoluto | a | de um inteiro a também pode ser definido pelas igualdades:
|a|
a² , | a | = máx [-a, a]
onde a² denota a raiz quadrada não negativa de a² e máx [-a, a] indica o maior dos dois
inteiros –a e a.
Assim, por exemplo:
| 4|
( 4)²
16 4
| 6 | = máx [-6, 6] = 6
Teorema 1.1: Se a e b são dois inteiros, então:
| ab | | a | . | b |
12
CAPÍTULO 1
NÚMEROS INTEIROS – NOÇÕES FUNDAMENTAIS
Demonstração:
Com efeito:
| ab |
(ab)²
a²b²
a². b² | a | .| b |
Teorema 1.2: Se a e b são dois inteiros, então:
|a
b| |a| |b|
Demonstração:
Com efeito, pela definição de | a | , temos:
| a | a | a |,
|b| b |b|
Somando ordenadamente estas desigualdades, obtemos:
(| a | | b |)
a
b |a| |b|
*
o que implica:
|a
* Usou –se o fato de que x
a
a
b| |a| |b|
x
a.
Corolário 1.1: Se a e b são dois inteiros, então:
|a
b| |a| |b|
Demonstração:
Com efeito:
|a
b| |a
( b) | | a |
| b| |a|
|b|
13
CAPÍTULO 1
NÚMEROS INTEIROS – NOÇÕES FUNDAMENTAIS
1.4 – Representação dos Inteiros em outras Bases
Teorema 1.3: Dado um inteiro qualquer b
representação da forma:
2, todo inteiro positivo n admite uma única
n ambm am 1bm 1  a2b2 a1b a0
onde os ai são tais que 0
ai < b , i = 0, 1, ... , m
Demonstração:
Assim, dado um inteiro qualquer b 2 , todo inteiro positivo n pode ser representado por um
polinômio inteiro em b do grau m (porque am 0 ), ordenado segundo as potencias
decrescentes de b, e cujos coeficientes ai são inteiros que satisfaçam as condições:
0 ai
b(i 0,1, 2,, m) , sendo am
0
Este polinômio representa-se, de modo abreviado, pela notação:
n (amam 1 a2a1a0 )b
em que os coeficientes ai são indicados pela ordem respectiva, figurando o inteiro b como um
índice.
O inteiro b chama-se base e é costume dizer que n está escrito no sistema de base b.
Exemplos:
a) Escrever 105 no sistema binário
105 = 1.26 + 1.25 + 0.24 + 1.23 + 0.22 + 0.2 + 1 = (1101001)2
Por outro lado, (100111)2 = 1.25 + 0.24 + 0.23 + 1.22 + 1.2 + 1 = 39
b) Escrever 31415 no sistema de base 8
Temos, sucessivamente:
31415
3926
490
61
7
Portanto 31415 = 7.84 + 5.83 + 2.82 + 6.8 + 7
14
= 8.3926
= 8.490
= 8.61
= 8.7
= 8.0
= (75267)8
+
+
+
+
+
7
6
2
5
7
CAPÍTULO 1
NÚMEROS INTEIROS – NOÇÕES FUNDAMENTAIS
c) Escrever (3531)6 no sistema de base 10
Temos, (3531)6 = 3.63 + 5.62 + 3.6 + 1 = 847
d) Escrever (6165)7 no sistema de base 12
Temos, (6165)7 = 6.73 + 1.72 + 6.7 + 5 = 2154
Vamos escrever 2154 (base 10) na base 12:
2154
179
14
1
= 12.179
= 12.14
= 12.1
= 12.0
+6
+ 11
+2
+1
No sistema de base 12 é hábito designar 10 e 11 por a e b, respectivamente, de modo que os
algarismos deste sistema são: 0, 1, 2, 3, 4, 5, 6, 7, 8, 9, a, b. Portanto,
2154 = 1.123 + 2.122 + b.12 + 6 = (12b6)12
Assim, no sistema de numeração decimal, dado um inteiro n, temos que,
n = am. 10m + am-1. 10m-1 + ... + a1. 10 + a0, 0 ≤ ak ≤ 10
é a representação no sistema decimal do inteiro positivo n.
Podemos também dizer que todo inteiro positivo n pode ser expresso sob a forma:
n = 10k + a0
Onde a0 é o algarismo das unidades de n
1.5 – Fatorial e Princípio Fundamental da Contagem
Foi a necessidade de calcular o número de possibilidades existentes nos chamados jogos de
azar que levou ao desenvolvimento da Análise Combinatória, parte da Matemática que estuda
os métodos de contagem. Esses estudos foram iniciados já no século XVI, pelo matemático
italiano Niccollo Fontana (1500-1557), conhecido como Tartaglia. Depois vieram os
franceses Pierre de Fermat (1601-1665) e Blaise Pascal (1623-1662).
A Análise Combinatória visa desenvolver métodos que permitam contar - de uma forma
indireta - o número de elementos de um conjunto, estando esses elementos agrupados sob
certas condições.
Definição 1.2: Chama-se fatorial de um inteiro não negativo n ( n 0 ), o inteiro que se indica
por n!, e tal que:
n!
1, se n = 0 ou n = 1
n(n 1)(n 2)...3.2.1 se n
2
Assim, por exemplo:
15
CAPÍTULO 1
NÚMEROS INTEIROS – NOÇÕES FUNDAMENTAIS
7! = 7.6.5.4.3.2.1 = 5040
Observe-se que n! = n.(n-1)!.
Exemplo 1.4: Escrever, usando o símbolo de fatorial, o produto dos n primeiros inteiros
positivos pares e o produto dos n primeiros inteiros positivos ímpares.
Os n primeiros inteiros positivos pares são:
2,4,6, ..., 2n – 2, 2n
Isto é:
2.1,2.2,2.3, ..., 2 . (n – 1), 2n
Portanto:
2,4,6, ..., 2n – 2, 2n = 2n (1.2.3... (n -1).n) = 2n . n!
Os n primeiros inteiros positivos ímpares são:
1,3,5, ..., 2n – 3, 2n - 1
Portanto:
1.3.5...(2n
3).(2n 1)
1.2.3.4...(2n 2).(2n 1).2n
2.4.6...(2n 2).2n
(2n )!
2n.n !
Exemplo 1.5: Calcular a soma:
1.1! + 2.2 ! + 3.3! + ... + n.n!
Tomemos a igualdade:
k.k! = (k + 1)! – k!
e nela façamos sucessivamente k = 1, 2, 3,..., n, o que dá:
1.1! = 2! – 1
2.2! = 3! – 2!
3.3! = 4! – 3!
n.n! = (n + 1)! – n!
Somando ordenadamente todas essas n igualdades e simplificando, obtemos:
1.1! + 2.2! + 3.3! +...+ n.n! = (n + 1)! – 1
1.6 – Princípio fundamental da contagem - PFC
16
CAPÍTULO 1
NÚMEROS INTEIROS – NOÇÕES FUNDAMENTAIS
Se determinado acontecimento ocorre em n etapas diferentes, e se a primeira etapa pode
ocorrer de k1 maneiras diferentes, a segunda de k2 maneiras diferentes, e assim
sucessivamente , então o número total T de maneiras de ocorrer o acontecimento é dado por:
T = k1. k2 . k3 . ... . kn
1.7 – Número Binomial
Definição 1.3: Sejam n > 0 e k dois inteiros tais que 0 k
de numerador n e classe k, o inteiro que se indica por
n
n
k
n . Chama-se número binomial
, e tal que:
n!
k!(n k)!
k
Obviamente, também podemos escrever:
n
n(n 1)...(k 1)
(n k)!
k
n(n 1)...(n k 1)
k!
Em particular, para k = 0 ou k = n, temos:
n
n
0
n
1
Assim, por exemplo:
8
3
8! 8.7.6.5.4.3.2.1 8.7.6
56
3!5! 3.2.1.5.4.3.2.1 3.2.1
7
7.6.5 7.6.5
35
4 (7 4)! 3.2.1
17
CAPÍTULO 1
NÚMEROS INTEIROS – NOÇÕES FUNDAMENTAIS
1.8 – Números Binomiais Complementares
Definição 1.4: Chamam-se números binomiais complementares dois números binomiais que
têm o mesmo numerador e cuja soma das suas classes respectivas é igual ao numerador
comum.
Assim, por exemplo,
20
20
e
7
são números binomiais complementares, pois, têm o
13
mesmo numerador 20 e 7 + 13 = 20.
Teorema 1.4: Dois números binomiais complementares são iguais.
Demonstração:
Sejam
n
k
e
n
h
dois números binomiais complementares. Então, k + h = n e k = n – h.
Portanto:
n
n
k
n h
n
n!
n!
(n h)!(n (n h))! (n h)!h!
h
1.9 – Números Binomiais Consecutivos
Definição 1.5: Chamam-se números binomiais consecutivos dois números binomiais que têm
o mesmo numerador e cujas classes respectivas são inteiros consecutivos.
Assim, por exemplo,
18
9
e
18
10
são números binomiais consecutivos, pois, têm o mesmo
numerador 18 e as suas classes respectivas são os inteiros consecutivos 9 e 10.
Teorema 1.5: Entre dois números binomiais consecutivos
subsiste a relação de Stifel:
18
n
n
n 1
k 1
k
k
n
k 1
e
n
k
, com 1 k
n,
CAPÍTULO 1
NÚMEROS INTEIROS – NOÇÕES FUNDAMENTAIS
Demonstração: Com efeito:
n
n
k 1
k
n!
n!
(k 1)!(n k 1)! k!(n k)!
n!
n!
(k 1)!(n k 1)(n k)! k(k 1)!(n k)!
n!
1
1
(k 1)!(n k)! n k 1 k
n!
n 1
(k 1)!(n k)! k(n k 1)
n 1
(n 1)!
k!(n 1 k)!
k
Assim, por exemplo:
Corolário 1.2:
n
n 1
n 2
k
k 1
k 1
18
18
19
9
10
10
13
12
12
8
8
7
...
k
k 1
k 1
k 1
Demonstração:
Com efeito, mudando na relação de Stifel n sucessivamente por n – 1,
obtemos:
n
n 1
n 1
k
k 1
k
n 1
n 2
n 2
k
k 1
k
n 2
n 3
n 3
k
k 1
k
n – 2, n – 3,..., k,
...........................................
n 1
k
k
k
k 1
k
19
CAPÍTULO 1
NÚMEROS INTEIROS – NOÇÕES FUNDAMENTAIS
Além disso, é evidente:
k
k 1
k
k 1
.
Somando ordenadamente todas essas igualdades e suprimindo os termos comuns aos dois
membros acha-se a relação desejada.
Substituindo, nesta relação, cada número binomial pelo seu complementar, obtemos:
Corolário 1.3:
n
n 1
n 2
n k
n k
n k 1
n
n 1
n 2
k
k
k 1
...
...
k
k 1
1
0
n k
n k 1
1
0
Demonstração: Consoante a relação de Stifel, temos:
n
k
n 1
k 1
n 1
k
n 1
n 2
n 2
k 1
k 2
k 1
n 2
n 3
n 3
k 2
k 3
k 2
...........................................
n k 1
1
n k
0
n k
1
Além disso, temos:
n k
n k 1
0
0
Somando ordenadamente todas essas igualdades e suprimindo os termos comuns aos dois
membros acha-se a relação desejada.
1. 10 – Piso, Teto e Nint de um número real.
É fácil perceber que qualquer número real está entre dois números inteiros, um inteiro menor
que o dado número real e um inteiro maior que esse número real. Por exemplo, o número real
3
5 3 ); o número real
, está entre os inteiros -5 e 5 , está entre os inteiros 2 e 3 ( 2
2
3
4( 5
4 ), etc.. Veremos a seguir que o inteiro à esquerda será chamado de Piso
2
(floor) e o inteiro à direita será chamado de Teto(ceiling).
20
CAPÍTULO 1
NÚMEROS INTEIROS – NOÇÕES FUNDAMENTAIS
Definição 1.6: Chamam-se partes inteiras de um número real r, os inteiros n e n+1 que
verificam às condições:
n
r
n 1
A todo número real r podemos associar dois números inteiros chamados piso e teto. Keneth
Iverson introduziu esses nomes, assim como a notação que será usada, no início da década de
1960.
Definição 1.7: Chama-se piso de um número real r, ao maior número inteiro menor ou igual a r.
Definição 1.8: Chama-se teto de um número real r, ao menor número inteiro maior ou igual a r.
Definição 1.9: Chama-se nint de um número real r, o valor inteiro mais próximo de r. Para
evitar ambigüidades, no caso de valores de r iguais à metade de um inteiro, convenciona-se
arredondar o valor de nint sempre para o inteiro par.
Notação: Usaremos as seguintes notações:
r = piso de r
r = teto de r
r
= nint de r
Assim, o piso e o teto de um número real r são os inteiros definidos pelas desigualdades:
r 1
r
r
r
r 1
Em linguagem da Teoria dos Conjuntos:
r
max{n

| n
r} e
r
min{n
| n
r}
Observe que r
r
r se, e somente se, r é um número inteiro, e que todo número real r
pode ser escrito sob a forma:
r
r
k , onde 0
r
r
1 k , onde 0
k
r
r
1
e
k
r
r
1 1
O número real k chama-se parte não-inteira de r.
21
CAPÍTULO 1
NÚMEROS INTEIROS – NOÇÕES FUNDAMENTAIS
Exemplos: piso e teto de r.
a)
2
b)
1 e
3 e
3
2
c)
2
d)
2
3
2
1
f)
1
3
0 e
1
2
e)
4
2 e
1
3
1 e
7
7 e
1
1
2
7
0
7
Exemplos: nint de r.
a) [2,3] = 2 e [2,7] = 3
b)
1
3
c) [ ] = 3
22
0
e
23
6
e [e] = 3
d) [3,5] = 4 e [4,5] = 4
4
e)
1
2
0
e
1,5
2
f) [-3, 4] = -3 e [-3, 7] = -4
CAPÍTULO 1
NÚMEROS INTEIROS – NOÇÕES FUNDAMENTAIS
Abaixo, estão ilustrados os gráficos das funções piso, teto e nint, respectivamente.
f ( x)
x
f ( x)
x
23
CAPÍTULO 1
NÚMEROS INTEIROS – NOÇÕES FUNDAMENTAIS
f ( x)
x
1.11 – O Princípio da Casa dos Pombos (Princípio das Gavetas de Dirichlet)
O princípio da Casa dos Pombos é a afirmação de que se n pombos devem ser postos em m
casas, sendo n > m então pelo menos uma casa irá conter mais de um pombo.
É também conhecido como Princípio das Gavetas de Dirichlet, acredita-se que o primeiro
relato deste principio foi feito pôr Dirichlet em 1834, com o nome de Schubfachprinzip
("Princípio das Gavetas").
O princípio da casa do pombo é um exemplo de um argumento de calcular que pode ser
aplicado em muitos problemas formais, incluíndo aqueles que envolvem um conjunto infinito.
Exemplo: Quantas pessoas são necessárias para se ter certeza que haverá pelo menos duas
delas façam aniversário no mesmo mês?
Resposta: 13 pessoas. Pelo princípio da casa dos pombos se houver mais pessoas (13) do que
meses (12) é certo que pelos menos duas pessoas terão nascido no mesmo mês.
Embora o princípio da casa dos pombos seja uma observação trivial, pode ser usado para
demonstrar resultados possivelmente inesperados . Por exemplo, em toda grande cidade,
digamos com mais de 1 milhão de habitantes existem pessoas com o mesmo número de fios de
cabelo. Demonstração: Tipicamente uma pessoa tem cerca de 150 mil fios de cabelo. É
razoavel supor que ninguém tem mais de 1.000.000 de fios de cabelo em sua cabeça. Se há
mais habitantes do que o número máximo de fios de cabelo, necessariamente pelo menos duas
pessoas terão exatamente o mesmo número de fios de cabelo.
24
CAPÍTULO 1
NÚMEROS INTEIROS – NOÇÕES FUNDAMENTAIS
1.12 – Caos Fatorial: !n.
Suponha que queremos calcular todos os anagramas da palavra ESCOLA, de modo que
nenhuma letra ocupe o seu lugar original, ou primitivo. Um deles seria SEOCAL, uma vez
que nenhuma letra ocupa seu lugar inicial. Esse tipo de permutação é chamada de caótica ou
desordenada e o caos fatorial n ( também chamado de subfatorial ou derangements em
Inglês), simbolizado por !n , é usado para calcular o número dessas permutações caóticas.
Lembre-se que o fatorial calcula o total de permutações de um conjunto.
Definição 1.11.: Chama-se caos fatorial de um inteiro não negativo n ( n 0 ), o inteiro que
se indica por !n, e tal que:
!n
n!
1 1 1 1
( 1) n
...
0! 1! 2! 3!
n!
n
n!
k
( 1) n
k!
0
Para n 1 , temos:
!n
Pode-se provar que !n
n!
1 1
( 1) n
...
2! 3!
n!
n
n!
k
( 1) n
k!
2
n!
.
e
M. Hassani deu outras formas para o caos fatorial:
!n
n! 1
,n 1
e
!n
e e
e
1
n!
en! , n 1
Os 10 primeiros valores !n, são:
n
0
1
2
3
4
5
6
7
8
9
10
n!
1
0
1
2
9
44
265
1854
14833
133496
1334961
25
CAPÍTULO 1
NÚMEROS INTEIROS – NOÇÕES FUNDAMENTAIS
Voltando ao problema comentando no início, podemos afirmar que o número de permutações
caóticas da palavra ESCOLA é !6 = 265.
Exemplos:
1 1 1
2! 3! 4!
9 1 1
!6 6!
24 5! 6!
6.5.4!.9 6.5!
!6
4!
5!
a) !6 6!
!6
265
b) !6
6!
e
c) !6
6! 1
e
1 1
5! 6!
9
6!
4!
6!
270
6!
720
2,718...
d) !6
2, 718...
!6
2221,92
1 1
2 6
1 1
5! 6!
265
265, 241...
1
.720
2, 718...
1956,96
1 1
5! 6!
6 1
264,87...
721
2,718...
1
24
6!
265
2, 718... .720
2221 1956
3, 086... .720
2,718... .720
265
1.13 – Left Fatorial: L!n
Dura Kurepa , em 1971 publicou a o conceito de L!n, o left factorial, definido como
n 1
L !n
0! 1! 2! ... (n 1)!
k!
k 0
Um famoso problema em aberto na Teoria dos Números, é uma conjectura feita por Kurepa
de que o MDC (n!, L!n) = 2 para todo n maior que 1.
Abaixo colocamos os 10 primeiros valores do left fatorial. Por definição, L!0 = 0.
n
0
1
2
26
L!n
0
1
2
CAPÍTULO 1
NÚMEROS INTEIROS – NOÇÕES FUNDAMENTAIS
3
4
5
6
7
8
9
10
4
10
34
154
874
5914
46234
409114
O left fatorial é sempre par para qualquer inteiro maior que 1. Se dividirmos o left fatorial por
2, obtemos alguns valores primos. Veja
L !n
n
2
3
2
4
5
5
17
8
2957
9
23117
10
204557
L !n
Uma questão em aberto é saber se existem infinitos primos da forma
.
2
EXERCÍCIOS
1) Sem usar P.A., calcule a soma dos “n”
primeiros inteiros positivos.
2)
Calcular o inteiro positivo n, sabendo que
3n+2 . 2n+3 = 2592.
8) Decompor o inteiro 565 numa soma de cinco
inteiros ímpares consecutivos.
9) Achar todas as soluções inteiras e positivas da
equação (x + 1)(y + 2) = 2xy.
3) Calcule o inteiro positivo n, sabendo-se que:
3n + 3n+1 + 3n+2 + 3n+3 = 1080.
10) Determinar todos os inteiros positivos de dois
algarismos que sejam igual ao quádruplo da
soma dos seus algarismos.
4) Com uma calculadora, achar os valores de n <
10 para os quais n! + 1 é um quadrado perfeito.
11) Achar o menor e o maior inteiro positivo de n
algarismos.
5) Sendo m e n inteiros positivos, dizer se é
verdadeiro ou falso:
12) Resolva a equação: (x + 2)! = 72.x!
a) (mn)! = m!. n!
x
b) (m + n)! = m! + n!
6) Demonstrar: (n – 1)! [(n + 1)! – n!] = (n!)2
7
13) Resolver a equação:
14) Demonstrar :
n
k
2
7
x
2x 2
n k 1 n
k 1
k
7) Sendo n > 2, demonstrar: (n2)! > (n!)2.
27
CAPÍTULO 1
NÚMEROS INTEIROS – NOÇÕES FUNDAMENTAIS
15) Achar todas as soluções inteiras e positivas da
equação: x2 – y2 = 88.;
Determine a posição (linha e coluna) ocupada
pelo número 107.
16) Verificar se o quadrado de um inteiro pode
terminar em 2, 3, 7 ou 8.
17) Hilbert escreveu os inteiros de 1 até 1000
(inclusive), em ordem decrescente. Sem usar
P.A, determine qual foi o 3330 inteiro
escrito?
26) Mostrar que o produto de quatro algarismos
consecutivos, aumentado de 1, é um quadrado
perfeito.
18) Calcular o número de algarismos necessários
para ser escrever os números positivos de 1, 2.
3, 4, ......, n algarismos.
28) Escrever os inteiros de 1 a 1993, inclusive,
quantas vezes o algarismo 1 é escrito?
19) O produto de um inteiro positivo de três
algarismos por 7 termina à direita por 638.
Achar esse inteiro.
29) Determinar o inteiro n > 1 de modo que a soma
1! + 2! + 3! + ... + n! seja um quadrado
perfeito.
20) Determinar quantos algarismos se emprega
para numerar todas as páginas de um livro de
2748 páginas.
30) A média aritmética de dois inteiros positivos é
5 e a média geométrica é 4. Encontre esses
números.
21) Dois homens estavam conversando num bar
quando um virou para o outro e disse:
31) Achar cinco inteiros positivos consecutivos
cuja soma dos quadrados é igual a 2010.
-
-
Tenho três filhas a soma de suas idades é
igual ao número da casa em frente e o
produto é 36.
Posso determinar as idades de suas filhas
apenas com esses dados?
Não. Dar-lhe-ei um dado fundamental:
minha filha mais velha toca piano.
Determine as idades das filhas e o número da
casa em frente.
22) Calcular a soma dos três maiores números
inteiros de, respectivamente, três, quatro e
cinco algarismos.
23) Determinar a diferença entre o maior número
inteiro com seis algarismos diferentes e o
maior inteiro com cinco algarismos também
diferentes.s
24) Um livro tem 1235 páginas. Determinar o
número de vezes que o algarismo 1 aparece na
numeração da páginas deste livro.
25) Os números abaixo estão dispostos em linhas e
colunas.
1
2
8
9
15 16
22 23
29 30

28
27) A soma dos quadrados de dois inteiros é 3332 e
um deles é o quádruplo do outro. Achar os dois
inteiros.

32) O resto por falta da raiz quadrada de um inteiro
positivo é 135 e o resto por excesso é 38.
Achar esse inteiro.
33) Resolver a equação
x ! 3( x 2)!
x ! 3( x 2)!
31
29
34) Achar o inteiro que deve ser somado a cada um
dos inteiros 2, 6 e 14 para que, nesta ordem,
formem uma proporção contínua.
60
40
35) Coloque em ordem crescente: 2 ; 3 ;
.
720
36) Achar o valor mínimo de uma soma de 10
inteiros positivos distintos, cada um dos quais
se escreve com três algarismos.
37) O menor número natural n, diferente de zero,
que torna o produto de 3888 por n um cubo
perfeito é:
38) Um estudante ao efetuar a multiplicação de
7432 por um certo inteiro achou o produto
1731656, tendo trocado, por engano, o
algarismo das dezenas do multiplicador,
tomando 3 em vez de 8. Achar o verdadeiro
produto.
39) Achar o menor inteiro cujo produto por 21 é
um inteiro formado apenas por 4 algarismo.
CAPÍTULO 1
NÚMEROS INTEIROS – NOÇÕES FUNDAMENTAIS
40) Escreve-se a seqüência natural dos inteiros
positivos, sem separar os algarismos:
123456789101112131415...
Determinar:
a)
o 435º algarismo
Apertando um botão do bordo do retângulo,
trocam de cor ele e seus vizinhos (do lado ou
em diagonal). Apertando o botão do centro,
trocam de cor todos os seus 8 vizinhos porém
ele não.
Exemplos:
b) o 1756º algarismo.
c)
Apertando 1, trocam de cor 1, 2, 4 e 5.
Apertando 2, trocam de cor 1, 2, 3, 4, 5 e 6.
Apertando 5, trocam de cor 1, 2, 3, 4, 6, 7, 8 e 9.
o 12387º algarismo.
41) Escreve-se a seqüência natural dos inteiros
positivos pares, sem separar os algarismos:
24681012141618...
Determinar o 2574º algarismo que se escreve.
Inicialmente todos os botões estão verdes. É
possível, apertando sucessivamente alguns
botões, torná-los todos vermelhos?
50) Escrevemos abaixo os números naturais de 1 a
10.
1999
42) As representações decimais dos números 2
e 51999 são escritos lado a lado. O número de
dígitos escritos é igual a:
43) Mostrar que o produto de dois fatores entre 10
e 20 é o décuplo da soma do primeiro com as
unidades do segundo mais o produto das
unidades dos dois.
44) Achar o menor inteiro positivo que
multiplicado por 33 dá um produto cujos
algarismos são todos 7.
45) Os inteiros a e b são tais que 4 < a < 7 e 3 < b
< 4. Mostrar que 0 < a – b < 4.
46) Os inteiros a e b são tais que –1 < a < 3 e –2 <
b < 0. Mostrar que –1 < a – b < 5.
47) Os inteiros a e b são tais que -2 < a < 2 e 2 < b < 2. Mostrar que –4 < a – b < 4.
48) Em um quartel existem 100 soldados e, todas
as noites, três deles são escolhidos para
trabalhar de sentinela. É possível que após
certo tempo um dos soldados tenha trabalhado
com cada um dos outros exatamente uma vez?
49) Um jogo consiste de 9 botões luminosos (de
cor verde ou vermelha) dispostos da seguinte
forma:
1
2
3
4
5
6
7
8
9
1
2
3
4
5
6
7
8
9
10.
Antes de cada um deles, coloque sinais “+” ou
“–” de forma que a soma de todos seja zero.
51) Escrevemos abaixo os números naturais de 1 a
11.
1
2
3
4
5
6
7
8
9
10
11
Antes de cada um deles, coloque sinais “+” ou
“–” de forma que a soma de todos seja zero.
52) Para numerar as páginas de um livro foram
utilizados 663 algarismos. Quantas páginas
tinha o livro?
53) Seja Q = 1! + 2! + 3! + ... + n!. Para quantos
valores de n tem-se Q quadrado perfeito?
54) Quantos são os números naturais de 4 dígitos
que possuem pelo menos dois dígitos iguais?
55) Quantos são os números de 5 algarismos, na
base 10:
a) Nos quais o algarismo 2 figura?
b) Nos quais o algarismo 2 não figura?
56) Permutam-se de todos os modos possíveis os
algarismos 1, 2, 4, 6, 7 e escrevem-se os
números assim formados em ordem crescente.
a) Que lugar ocupa o número 62417?
b) Qual o número que ocupa o 66º lugar?
c) Qual o 200º algarismo escrito?
d) Qual a soma dos números assim
formados?
29
Capítulo 2:
INDUÇÃO MATEMÁTICA
INTRODUÇÃO
A
s ciências naturais utilizam o método chamado indução empírica para formular leis
que devem reger determinados fenômenos a partir de um grande número de
observações particulares, selecionadas adequadamente. Esse tipo de procedimento,
embora não seja uma demonstração de que um dado fato é logicamente verdadeiro, é
frequentemente satisfatório. Por exemplo: ninguém duvidaria de que quando um corpo é
liberado ao seu próprio peso, no vácuo, na superfície da terra, ele cai segundo a vertical do
local.
A validade de um teorema matemático se estabelece de forma totalmente diferente. Verificar
que uma certa afirmação é verdadeira num grande número de casos particulares não nos
permitirá concluir que ela é válida.
Para demonstrar a verdade de uma sequência infinita de proposições, uma para cada inteiro
positivo, introduziremos o chamado método de recorrência ou indução matemática.
2.1 – Elemento mínimo de um conjunto de inteiros
Definição 2.1: Seja A um conjunto de inteiros. Chama-se elemento mínimo de A um elemento
a A tal que a x para todo x A .
Representa-se pela notação “minA”, que se lê: “mínimo de A”. Portanto, simbolicamente:
minA = a
(a
A e( x
A) (a
x ))
Teorema 2.1: Se a é elemento mínimo de A, então esse elemento é único.
Demonstração:
Com efeito, se existisse um outro elemento mínimo b de A, teríamos:
i) a b , porque a = minA.
ii) b a , porque b = minA..
30
CAPÍTULO 2
INDUÇÃO MATEMÁTICA
Logo, pela propriedade anti-simétrica da relação de ordem natural “ ” em Z, temos a = b.
O elemento mínimo de A, se existe, denomina-se também primeiro elemento de A ou menor
elemento de A
Exemplo 2.1: O conjunto N = {1, 2, 3,...} dos inteiros positivos tem o elemento mínimo, que
é 1 (minN = 1), porque 1 N e 1 n para todo n N .
Exemplo 2.2: O conjunto A x  | x 12
13), porque 13 A e 13 x para todo x A .
tem o elemento mínimo, que é 13 (minA =
Exemplo 2.3: O conjunto  0, 1, 2, 3,... dos inteiros não positivos não tem o elemento
mínimo, porque não existe a
Z - tal que a
A (3 divide 9) e 3
para todo x
Z- .
x  |3divide x 2 tem o elemento mínimo 3 (min A = 3),
Exemplo 2.4: O conjunto A
porque 3
x
x para todo x
A (1
Ae2
A).
2.2 – Princípio da boa ordenação
Todo conjunto não vazio A de inteiros não negativos possui o elemento mínimo.
Em outros termos, todo subconjunto não vazio A do conjunto
Z+ ={0 ,1, 2, 3, ...}
dos inteiros não negativos (
A
Z + ) possui o elemento mínimo, isto é, simbolicamente:
( A
Z ,A
)
min A
Exemplo 2.5: O conjunto A = {1, 3, 5, 7,...} dos inteiros positivos ímpares é um subconjunto
não vazio de
Z+ (
A
Z + ).
Logo, pelo “Princípio da boa ordenação”, A possui o elemento mínimo (minA = 1).
Exemplo 2.6: O conjunto P = {2,3,5,7,11, ...} dos inteiros primos é um subconjunto não vazio
de Z+ (
P Z+). Logo, pelo “Principio da boa ordenação”, P possui o elemento mínimo
(minP = 2).
Teorema 2.2 (de Archimedes): Se a e b são dois inteiros positivos quaisquer, então existe um
inteiro positivo n tal que na b .
Demonstração:
31
CAPÍTULO 2
INDUÇÃO MATEMÁTICA
Suponhamos que a e b são dois inteiros positivos para os quais na b para todo inteiro
positivo n. Então, todos os elementos do conjunto:
S = {b – na | n N }
são inteiros positivos e, pelo “Princípio da boa ordenação”, S possui o elemento mínimo,
digamos minS = b – ka.
E como b – (k + 1)a pertence a S, porque S contém todos os inteiros positivos desta forma,
temos:
b – (k + 1) a = (b – ka) – a < b – ka
isto é, b – ka não é o elemento mínimo de S, o que é uma contradição. Logo, a propriedade
archimediana é verdadeira.
Assim, por exemplo:
i) se a = 2 e b = 11, então n = 6, porque 6.2 > 11;
ii) se a = 9 e b = 5, então n =1, porque 1.9 > 5.
2.3 – Princípio de Indução Finita.
Quando uma proposição é enunciada em termos de números naturais, o Princípio de indução
finita constitui um eficiente instrumento para demonstrar a proposição no caso geral.
Na prática, o método pode ser entendido por um artifício muito simples. Vamos supor que
temos uma série de dominós idênticos colocados em fila, que começa por um deles e
prossegue indefinidamente. Nosso objetivo é - empurrando apenas um dominó - garantir que
todos caiam. Como derrubar todos os dominós? Para isso, basta nos assegurarmos de que:
1) O primeiro dominó cai;
2) Os dominós estão dispostos de tal modo que qualquer um deles - toda vez que cai -,
automaticamente, empurra o dominó seguinte e o faz cair também.
Assim, mesmo que a fila se estenda indefinidamente, podemos afirmar que todos os
dominós cairão.
32
CAPÍTULO 2
INDUÇÃO MATEMÁTICA
 Vamos estabelecer matematicamente esses procedimentos.
Teorema 2.3 Seja S um subconjunto do conjunto N dos inteiros positivos ( S
satisfaz as duas seguintes condições:
N ) que
i) 1 pertence a S ( 1 S );
ii) para todo inteiro positivo k, se k S , então (k 1) S .
Nestas condições, S é o conjunto N dos inteiros positivo: S = N.
Demonstração:
Suponhamos, por absurdo, que S não é o conjunto N dos inteiros positivos ( S
conjunto de todos os inteiros positivos que não pertencem a S, isto é:
X = {x | x
N e x
N ) e seja X o
S}=N–S
Então, X é um subconjunto não vazio de N (
X
N ) e, pelo “Princípio da boa
ordenação”, existe o elemento mínimo x0 de X (minX = x0 ).
Pela primeira condição, 1 S , de modo que x0 > 1 e, portanto, x0 - 1 é um inteiro positivo que
não pertence a X. Logo, (x0 - 1) S e, pela segunda condição, segue-se que ( x0 - 1) + 1 = x0
S , o que é uma contradição, pois, x0
X
N S , isto é, x0
S . Assim sendo, X
eS
= N.
Consoante este “Princípio de indução finita”, o único subconjunto de N que satisfaz às duas
condições é o próprio N.
2.4 – Indução Matemática
Em matemática, conclusões como as que se obtêm a seguir são inadmissíveis. Por quê? Em
que pecam os raciocínios utilizados? Vamos examiná-los...
1) Suponha que desejemos obter uma fórmula que dá o valor da soma Sn = 1 + 3 + 5 + 7 + ...
+ (2n - 1), para qualquer inteiro positivo de n.
É fácil ver que:
n=1
S1 = 1 = 12;
n=2
S 2 = 1 + 3 = 4 = 22 ;
n=3
S 3 = 1 + 3 + 5 = 9 = 32
n=4
S4 = 1 + 3 + 5 + 7 = 16 = 42
33
CAPÍTULO 2
INDUÇÃO MATEMÁTICA
Por meio de um raciocínio indutivo, os resultados obtidos nos levam a afirmar que para
todo inteiro positivo n tem-se Sn = n2.
2) Consideremos o trinômio P(n) = n2 + n + 41. Considerando n = 0, obtemos P(0) = 41, que
é um número primo. Substituindo n por 1, chegamos a outro número primo, o 43.
Substituindo sucessivamente n por 2, 3, 4, 5, 6, 7, 8, 9 e 10, conseguimos como resultados
outros números primos (47, 53, 61, 71, 83, 97, 113, 131 e 151, respectivamente). Então, os
resultados obtidos nos induzem a afirmar que, para todo n natural, o trinômio P(n) = n2 + n
+ 41, sempre produz como resultado um número primo.
Nos dois exemplos, propôs-se um resultado geral, supostamente válido para todo n, com
base no fato de que ele é correto para alguns valores particulares de n: tal procedimento,
entretanto, pode conduzir a conclusões falsas.
Assim, ainda que em no primeiro caso a proposição geral enunciada resulte correta - por
mero acaso! -, a proposição geral do segundo exemplo é falsa. De fato, P(n) gera números
primos para n= 0, 1, 2, 3, ..., 39, mas para n = 40, ele vale 412, que não é um número
primo. Portanto, no exemplo 2), encontramos uma proposição que - apesar de válida em
40 casos particulares - não é válida em geral.
Note bem: Uma proposição pode ser válida em uma série de casos particulares, mas,
mesmo assim, não o ser de maneira geral.
Coloca-se, então, o seguinte problema: temos uma proposição que se mostrou correta em
muitos casos particulares. No entanto, é impossível verificar todos os casos particulares.
Assim sendo, como podemos saber se a proposição é correta de modo geral? O Teorema
abaixo, esclarece essa questão.
Teorema 2.4: Seja P(n) uma proposição associada a cada inteiro positivo n e que satisfaz às
duas seguintes condições:
i)
P(1) é verdadeira;
ii) para todo inteiro positivo k, se P(k) é verdadeira, então P(k + 1) também é verdadeira.
 Nestas condições, a proposição P(n) é verdadeira para todo inteiro positivo n.
Demonstração:
Seja S o conjunto de todos os inteiros positivos n para os quais a proposição P(n) é
verdadeira, isto é:
S = { n N | P(n) é verdadeira}
Pela primeira condição, P(1) é verdadeira e, portanto, 1 S . Pela segunda condição, para todo
inteiro positivo k, se k S , então (k 1) S . Logo, o conjunto S satisfaz às duas condições
do “Princípio de indução finita” e, portanto, S = N, isto é, a proposição P(n) é verdadeira para
todo inteiro positivo n.
34
CAPÍTULO 2
INDUÇÃO MATEMÁTICA
Nota: O teorema 2.4 é geralmente denominado “Teorema da indução matemática” ou
“Princípio de indução matemática”, e a demonstração de uma proposição usando-se este
teorema chama-se “demonstração por indução matemática” ou “demonstração por indução
sobre n”.
Na “demonstração por indução matemática” de uma dada proposição P(n) é obrigatório
verificar que as condições i e ii são ambas satisfeitas. A verificação da condição i é
geralmente muito fácil, mas a verificação da condição ii implica em demonstrar o teorema
auxiliar cuja hipótese é:
H: proposição P(k) é verdadeira, k
N.
denominada “hipótese de indução”, e cuja tese ou conclusão é:
T: proposição P(k + 1) é verdadeira.
2.5. Exemplos de demonstração por Indução Matemática
Exemplo 2.7: Demonstrar a proposição:
P(n): 1 + 3 + 5 + ... + (2n – 1) = n²,
n
N
Demonstração:
i)
P(1) é verdadeira, visto que 1 = 1².
ii) A hipótese de indução é que a proposição:
P(k): 1 + 3 + 5 + ... + (2k – 1) = k², k
N
é verdadeira.
Adicionando (2k + 1) a ambos os membros desta igualdade, obtemos:
1 + 3 + 5 + ... + (2k – 1) + (2k + 1) = k² + (2k + 1) = (k + 1)²
e isto significa que a proposição P(k + 1) é verdadeira.
Logo, pelo “Teorema da indução matemática”, a proposição P(n) é verdadeira para todo
inteiro positivo n.
35
CAPÍTULO 2
INDUÇÃO MATEMÁTICA
Exemplo 2.8: Demonstrar a proposição:
P(n) :
1
1
1
1
...
1.2 2.3 3.4
n(n 1)
n
n 1
, n N
Demonstração:
1
1
1.2 1 1
2) A hipótese de indução é que a proposição:
1) P(1) é verdadeira, visto que
P(k) :
1
1.2
1
1
1
...
2.3 3.4
k(k 1)
k
k 1
,k N
é verdadeira.
Adicionando
1
a ambos os membros desta igualdade, obtemos:
k 1 k 2
1
1.2
1
1
1
...
2.3 3.4
k(k 1)
k
1
k 1
k 1 k 2
1
k 1 k 2
k 2 2k 1
(k 1) k 2
k 1
k 2
e isto significa que a proposição P k 1 é verdadeira. Logo, pelo “Teorema da indução
matemática”, a proposição P n é verdadeira para todo inteiro positivo n.
Exemplo 2.9: Demonstrar a proposição:
P(n) : 3| 22n 1 ,
Demonstração:
1) P (1) é verdadeira, visto que 3 | 22 1 .
2) A hipótese de indução é que a proposição:
P k : 3 | 22k 1 , k N é verdadeira.
Portanto:
22k – 1 = 3q, com q
36
Z
n N
CAPÍTULO 2
INDUÇÃO MATEMÁTICA
o que implica:
22 k 1
1 22.22k 1 4.22k 1
4.22k
4 4 1 4 22k 1
3
4.3q 3 3(4q 1)
isto é, a proposição P k 1 é verdadeira. Logo, pelo “teorema da indução matemática”, a
proposição P n é verdadeira para todo inteiro positivo n.
Exemplo 2.10: Demonstrar a proposição:
P(n) : 2n n,
n N
Demonstração:
1) P(1) é verdadeira, visto que 2¹ = 2 > 1.
2) A hipótese de indução é que a proposição:
P(k): 2 k
k, k
N
é verdadeira. Portanto:
2.2k > 2k ou 2k+1 > k + k
k+1
o que implica: 2k 1 k 1 , isto é, a proposição P(k+1) é verdadeira. Logo, pelo “Teorema da
indução matemática”, a proposição P(n) é verdadeira para todo inteiro positivo n.
2.6 . Outras formas da indução matemática
Teorema 2.5 Seja r um inteiro positivo fixo e seja P(n) uma proposição associada a cada
inteiro n r e que satisfaz às duas seguintes condições:
i) P(r) é verdadeira;
ii) para todo inteiro k
r, se P(k) é verdadeira, então P(k + 1) também é verdadeira.
 Nestas condições, P(n) é verdadeira para todo inteiro n
r..
37
CAPÍTULO 2
INDUÇÃO MATEMÁTICA
Demonstração:
Seja S o conjunto de todos os inteiros positivos n para os quais a proposição P(r + n – 1) é
verdadeira, isto é:
S = {n
N | P(r + n – 1) é verdadeira}
Pela primeira condição, P(r) = P(r + 1 – 1) é verdadeira, isto é, 1
condição, se P(r + k – 1) é verdadeira, então:
S. E, pela segunda
P((r + k – 1) + 1) = P(r + (k + 1) – 1)
também é verdadeira, isto é, se k S, então (k + 1) S. Logo, pelo “Princípio da indução
finita”, S é o conjunto dos inteiros positivos: S = N, isto é, a proposição
P(r + n – 1) é
verdadeira para todo n N , ou seja, o que é a mesma coisa, a proposição P(n) é verdadeira
para todo inteiro n r .
Exemplo 2.11: Demonstrar a proposição:
P(n): 2 n
n!,
n
4
Demonstração:
1) P(4) é verdadeira, visto que 24 16 4! 24 .
2) Suponhamos, agora, que é verdadeira a proposição:
P(k): 2 k
k !, k
4 (I)
Então, por ser
2 < k + 1 para k
4 ( II ),
multiplicando termo a termo ( I ) e ( II ):
2k
1
k !.(k 1) ou 2k
1
(k 1)!
isto é, a proposição P(k + 1) é verdadeira. Logo, pelo teorema 2.5, a proposição P(n) é
verdadeira para todo inteiro n 4 .
Observe-se que a proposição P(n) é falsa para n = 1, 2, 3, pois, temos:
2¹ > 1! , 2² > 2! , 2³ > 3!
Exemplo 2.12: Demonstrar a proposição:
38
CAPÍTULO 2
INDUÇÃO MATEMÁTICA
P(n) : n2 > 2n + 1,
n
3
Demonstração:
1) P (3) é verdadeira, visto que 32 = 9 > 2. 3 + 1= 7.
2) Suponhamos, agora, que é verdadeira a proposição:
P(k) : k2 > 2k + 1, k
3
Então, temos:
k2 + (2k+ 1) > (2k+1) + (2k+1)
ou
(k +1)2 > 2 (k + 1) + 2k > 2 (k + 1) + 2 > 2 (k +1) + 1, k
3
e, portanto:
(k +1)2 > 2 (k +1) + 1, k
3.
Isto é, a proposição P k 1 é verdadeira. Logo, pelo teorema 2.5, a proposição P n é
verdadeira para todo inteiro n 3 .
Observa-se que a proposição P n é falsa para n = 1 e n = 2, pois, temos:
12 < 2.1+1 e 22 < 2.2 + 1
Teorema 2.6 Seja P(n) uma proposição associada a cada inteiro positivo n e que satisfaz às
duas seguintes condições:
i) P(1) é verdadeira;
ii) para todo inteiro positivo k, se
P(1), P(2),..., P(k)
são todas verdadeiras, então P(k + 1) também é verdadeira.
Nestas condições, a proposição P(n) é verdadeira para todo inteiro positivo n.
39
CAPÍTULO 2
INDUÇÃO MATEMÁTICA
Demonstração:
Seja S o conjunto de todos os inteiros positivos n para os quais a proposição P(n) é
verdadeira, isto é:
S = { n N | P(n) é verdadeira}
Suponhamos por absurdo, que S N e seja X o conjunto de todos os inteiros positivos que
na pertencem a S, isto é:
X = {x | x N e x S } = N – S
Então, X é um subconjunto não vazio de N e, pelo “Princípio da boa ordenação”, existe o
elemento mínimo j de X (minX = j).
Pela primeira condição, 1 S , de modo que j > 1, e como j é o menor inteiro positivo que não
pertence a S, segue-se que as proposições P(1), P(2),..., P(j – 1) são todas verdadeiras. Então,
pela segunda condição, a proposição P(j) é verdadeira e j S , o que é uma contradição, pois
j X , isto é, j S . Assim sendo, S = N e a proposição P(n) é verdadeira para todo inteiro
positivo n.
Teorema 2.7 Seja r um inteiro positivo fixo e seja P(n) uma proposição associada a cada
inteiro n r e que satisfaz às duas seguintes condições:
1) P(r) é verdadeira;
2) para todo inteiro k > r, se P(m) é verdadeira para todo inteiro m tal que r
P(k) é verdadeira.
m
k , então
 Nestas condições, a proposição P(n) é verdadeira para todo inteiro n r .
Demonstração:
Seja S o conjunto de todos os inteiros n r para os quais a proposição P(n) é falsa, isto é:
S = { n N | n r e P(n) é falsa}
Suponhamos, por absurdo, que S não é vazio ( S
). Então, pelo “Princípio da boa
ordenação”, existe o elemento mínimo j de S (minS = j).
Pela primeira condição, r S , de modo que j > r, e, por conseguinte P(m) é verdadeira para
todo inteiro m tal que r m j . Assim sendo, pela segunda condição, P(j) é verdadeira e
), e a
j S , o que é uma contradição, pois, j S . Logo, o conjunto S é vazio ( S
proposição P(n) é verdadeira para todo inteiro n r .
40
CAPÍTULO 2
INDUÇÃO MATEMÁTICA
Nota Histórica
Era ideia assente na comunidade matemática do século XIX, que a indução era obra do
matemático francês Blaise Pascal , tendo em conta diversas demonstrações que apresenta no
seu Traité du Triangle Arithmétique.
Essa situação seria integralmente modificada, vinte anos após a formulação moderna de
indução matemática fixada por Giuseppe Peano , quando Giovanni Vacca , em 1909, num
artigo de três páginas publicado no Bulletin of American Mathematical Society, vem defender
que o italiano Francesco Maurolico , pelos trabalhos que desenvolveu no primeiro livro de
aritmética incluído na sua Opuscula Mathematica, escrita em 1557 e publicado em Veneza no
ano de 1575, como "the first discoverer of the principle of mathematical induction".
O artigo de Vacca encontrou eco, ainda que eventualmente sem verificação posterior, em
autores importantes como Moritz Cantor ou Siegmund Günther . M. Cantor, por exemplo,
que atribuiu inicialmente a Pascal a principal origem do método de indução completa (em
Vorlesungen uber Geschichte der Mathematik, vol. 2, p. 749), viria a transferir esse atributo
para Maurolico (em Zeichrift fur Mathematischen und Naturwissenschaftlichen Unterricht,
vol 33, 1902, p. 536), segundo conta devido a uma informação oral que lhe foi prestada pelo
próprio Vacca.
Passar-se-iam mais de quarenta anos sem que o artigo de Vacca fosse alvo de qualquer crítica.
Até que Hans Freudenthal (em Zur Geschichte der vollständigen Induktion, Archive
Internationale d'Histoire des Sciences 6 (1953) 17-37) depois de um exame detalhado dos
trabalhos de Maurolico, vem sustentar que em apenas três pontos conseguiu reconhecer uma
certa forma de indução matemática: uma forma arcaica, contudo, ao contrário do que
observou em Pascal, onde a indução é formulada pela primeira vez de uma maneira abstrata.
41
CAPÍTULO 2
INDUÇÃO MATEMÁTICA
EXERCÍCIOS
1) Demonstrar por "indução matemática":
c)
n(n 1)( 2n 1)
a) 1 + 2 + 3 + ... + n =
6
2
2
n
2
2
b)
n (n 1)
1 + 2 + 3 + ... + n =
4
3
3
3
2
3
n
N
c)
n ( 4n
3
n
n
e)
7 | (2 – 1)
n
N
f)
8 | 32n + 7,
n
N.
4) Demonstrar que 10n + 1 – 9n – 10 é um múltiplo
de 81 para todo inteiro positivo n
5) Demonstrar que
1)
N
3n
12 + 32 + 52 + ... + (2n – 1)2 =
2
n
d) 24 | (52n – 1)
N
2
5 | (8n – 3n)
N
n3
3
n5
5
positivo para todo n
7n
é um inteiro
15
N
d) 13 + 33 + 53 + ... + (2n –1)3 = n2(2n2 – 1)
6) Prove que, para todo inteiro n
e)
f)
1.2 + 2.3 + 3.4 + ... + n(n + 1) =
n(n 1)(n 2)
3
1
1
1 1 1
1
2
3
, n
an
...
1
1
n
n 1
4
n
1
3
1 , o número
é inteiro e ímpar.
n
7) Para n
1 , mostre que Sn
k ! é um
k 1
N
inteiro ímpar.
g) a + aq + aq2 + ...+aqn =
a (q n 1 1)
,q
q 1
1
8) Para n 0 , mostre que an 11n
é um inteiro divisível por 133.
2
122n
2) Demonstrar por "indução matemática"
a)
2n < 2n+1 n
n
2
b) 2 > n
c)
N
n
5
4n > n4 n
5
n
3
d) 2 > n
2
n
10
e)
n!>n
n
4
f)
n! > n3
n
6
g)
1
1
4
1
...
9
9)
1
n2
2–
1
, n
n
3) Demonstrar por "indução matemática"
2 | (3n – 1) n
N
b) 6 | (n3 – n) n
N
a)
42
Para n
3 , mostre que
a)
n 1
b)
n!
2
n
nn
1
nn . .
10) Mostre que é sempre possível pagar, sem
receber troco, qualquer quantia inteira de $,
maior que $7, com notas de $3 e $5.
N
1
Capítulo 3:
SOMATÓRIOS E PRODUTÓRIOS
3.1 . Somatórios
Sejam os n > 1 inteiros a1 ,a 2 ,...,a n . Para indicar, de modo abreviado, a soma a1 a 2 ... a n
desses n inteiros usa-se a notação:
n
ai
i 1
que se lê: “somatório de a i de 1 a n”.
Em particular, para n = 2, 3,..., temos:
2
3
ai
a1 a 2 ,
i 1
ai
a1 a 2 a 3 , ...
i 1
A letra i chama-se o índice do somatório e pode ser substituída por qualquer outra diferente de
a e de n – é um índice mudo. E os inteiros 1 e n que figuram abaixo e acima da letra grega
maiúscula (sigma) chamam-se respectivamente limite inferior e limite superior do índice i.
O número de parcelas de um somatório é sempre igual à diferença entre os limites superior e
inferior do seu índice mais uma unidade.
Se m e n são dois inteiros, com m n , então, por definição:
n
ai
am am 1 am
2
...a n
i m
Exemplo 3.1: Temos:
7
5i
5.1 5.2 5.3 5.4 5.5 5.6 5.7
i 1
5 10 15 20 25 30 35 140
43
CAPÍTULO 3
SOMATÓRIOS E PRODUTÓRIOS
4
8j 3
8.1 3
8.2 3
8.3 3
8.4 3
j 1
5 13 21 29
68
8
k .2k
3.23
4.44
5.25
6.26
7.27
8.28
k 3
24 64 160 384 896 2048 3576
Exemplo 3.2: Temos:
6
2i
2 4 8 16 32 64
i 1
15
1 3 5 ... 29
2j 1
j 1
3.2. Propriedades dos somatórios
n
n
Teorema 3.1:
(a i
n
bi )
ai
i 1
i 1
bi
i 1
Demonstração:
Com efeito, desenvolvendo-se o primeiro membro, temos:
n
(a i bi ) (a1 b1 ) (a 2 b2 ) ... (a n bn )
i 1
n
(a1 a 2 ... a n ) (b1 b2 ... b n )
n
ai
i 1
bi
i 1
n
Teorema 3.2
a
na
i 1
Demonstração:
Seja a i
a para i = 1, 2,..., n. Então, temos:
n
n
a
ai
i 1
n
Teorema 3.3
Demonstração:
44
n
(a i
i 1
a1 a 2 ... a n
i 1
a)
ai
i 1
na
a a ... a
na
CAPÍTULO 3
SOMATÓRIOS E PRODUTÓRIOS
Consoante os dois teoremas anteriores, temos:
n
n
(a i
i 1
n
n
a)
ai
n
a
i 1
ai
i 1
na
i 1
n
Teorema 3.4
ka i
k
i 1
ai
i 1
Demonstração:
Com efeito, desenvolvendo o primeiro membro, temos:
n
n
ka i
ka1 ka 2 ... ka n
k(a1 a 2 ... a n )
i 1
k
ai
i 1
20
Exemplo 3.3: Calcular
(5i 2)
i 1
Consoante os teoremas anteriores temos, sucessivamente:
20
20
(5i 2)
i 1
20
5i
i 1
20
2 5
i 1
i 20.2 5(1 2 ... 20) 40
i 1
1
5. (1 20)20 40 5.210 40 1090
2
3.3. Produtórios
Sejam os n > 1 inteiros a1 ,a 2 ,...,a n . Para indicar, de modo abreviado, o produto a 1a 2 ...a n
desses n inteiros usa-se a notação:
n
ai
i 1
que se lê: “produtório de a i de 1 a n”.
Em particular, para n = 2, 3,..., temos:
3
2
ai
i 1
a1a 2 ,
ai
a1a 2a 3 , ...
i 1
45
CAPÍTULO 3
SOMATÓRIOS E PRODUTÓRIOS
A letra i chama-se o índice do produtório e pode ser substituída por qualquer outra diferente
de a e de n – é um índice mudo. E os inteiros 1 e n que figuram abaixo e acima da letra grega
maiúscula
(pi) chamam-se respectivamente limite inferior e limite superior do índice i.
O número de fatores de um produtório é sempre igual à diferença entre os limites superior e
inferior do seu índice mais uma unidade.
Se m e n são dois inteiros, m n , então, por definição:
n
ai
a m .a m 1.a m 2 ...a n
i m
Exemplo 3.4: Temos:
6
3i
3.1 3.2 3.3 3.4 3.5 3.6
i 1
=3.6.9.12.15.18=524880
4
5j 3
5.1 3 5.2 3 5.3 3 5.4 3
j 1
=2.7.12.17
2856
Exemplo 3.5: Temos:
6
3i
3.9.27.81.243.729
i 1
16
1.3.5.7....31
2j 1
j 1
n
1.2.3... n 1 n
n!=
i
i=1
3.4. Propriedades dos Produtórios
n
Teorema 3.5
n
a i bi
i 1
n
ai .
i 1
bi
i 1
Demonstração:
Com efeito, desenvolvendo o primeiro membro, temos:
n
n
a i bi
i 1
46
(a1b1 )(a 2 b2 )...(a n bn )
(a1a 2 ...a n )(b1b 2 ...b n )
n
ai.
i 1
bi
i 1
CAPÍTULO 3
SOMATÓRIOS E PRODUTÓRIOS
n
Teorema 3.6
an
a
i 1
Demonstração:
Seja a i
a para i = 1, 2,..., n. Então, temos:
n
n
a
ai
i 1
n
Teorema 3.7
a1a 2 ...a n
a.a...a
an
i 1
n
kn
ka i
ai
i 1
i 1
Demonstração:
Com efeito, desenvolvendo o primeiro membro, temos:
n
n
n
ka i
(ka1 )(ka 2 )...(ka n ) k (a1a 2 ...a n ) k
n
i 1
ai
i 1
4
Exemplo 3.6: Calcular
(2i 1)²
i 1
Consoante o teorema 3.7, temos:
4
4
(2i 1)²
(2i 1) ² (3.5.7.9)² 945² 893025
i 1
i 1
n
n
n
a ij
Exemplo 3.7: Demonstrar
a ij
i, j 1
i 1 j 1
Com efeito, desenvolvendo o primeiro membro, temos:
n
a ij (a11a12 ...a1n )(a 21a 22 ...a 2n )...(a n1a n 2 ...a nn )
i, j 1
n
n
a1j .
j 1
n
a 2 j ...
j 1
n
n
a nj
j 1
a ij
i 1 j 1
47
Capítulo 4
DIVISIBILIDADE
Um conceito chave em Teoria dos Números é o conceito de divisibilidade. Existem muitos
aspectos interessantes referentes à divisão de números inteiros. Antes que possam ser
analisados, é necessário que conceitos básicos como divisor e divide estejam bem
estabelecidos.
4.1. RELAÇÃO DE DIVISIBILIDADE EM Z
Definição 4.1: Sejam a e b dois inteiros, com a
existe um inteiro q tal que b = aq
0. Diz-se que a divide b se, e somente se,
Se a divide b também se diz que a é divisor de b, que b é múltiplo de a, que a é um fator de b
ou que b é divisível por a.
Notação: a | b ( a divide b)
Observação: Se a | b , então –a | b
Teorema 4.1: Quaisquer que sejam os inteiros a, b e c tem-se:
1) a | 0 a
0, 1|a e a|a a
2) Se a | 1 , então a =
0
1
3) Se a | b e se c | d , então ac | bd
4) Se a | b e se b | c , então a | c
5) Se a | b e se b | a , então a =
6) Se a | b com b
0 , então | a |
b
|b|
7) Se a | b e se a | c , então a |(bx + cy) para todo x e y em Z
48
CAPÍTULO 4
DIVISIBILIDADE
Demonstração:
Em todas as demonstrações estaremos aplicando a Definição 1 e considerando aceitas todas
as propriedades operatórias dentro do conjunto  .
1) De fato:
0 = a.0;
a = 1.a;
a = a.1
2) De fato, se a|1, então 1 = a.q, o que implica a = 1 e q = 1 ou a = -1 e q = -1, ou seja:
a = 1.
3) De fato,
a|b
b
a.q
c|d
d
c.q1
Portanto:
bd
ac.(q.q1 )
ac | bd
4) De fato:
Logo, c a.(q.q1 )
a|b
b
a.q
b|c
c b.q1
a|b
b
b| a
a b.q1
a|c.
5) De fato:
a.q
Logo:
a
a(qq1 )
qq1 1
q1 |1
q1
1
a
b
6) De fato, nas condições da proprieade, temos:
a|b
Como q
b
a.q, ou seja | b | | a | . | q |
0 , temos que | q | 1 , desse modo temos | b | | a | .
7) De fato:
a|b
b
a.q
a|c
c a.q1
49
CAPÍTULO 4
DIVISIBILIDADE
Logo, quaisquer que sejam os inteiros x e y:
bx cy
aqx aq1 y
a(qx q1 y)
a | (bx cy) 
Esta propriedade (7) pode ser generalizada; ou seja, se
a | bk , k 1,2,3,..., n
então, quaisquer que sejam os inteiros
x1, x2 ,..., xn
temos:
a | (bx1 bx2 ... bxn )
De acordo com as propriedades (1) e (4), a relação de divisibilidade em  é reflexiva e
transitiva, mas não é simétrica.
4.2. Conjunto dos divisores de um inteiro
O conjunto de todos os divisores de um inteiro qualquer a indica-se por
D(a) = {x
Z* | x | a }
É imediato que, para todo inteiro a, se tem D(a) = D(-a).
Qualquer que seja o inteiro a 0 , se x | a , então:
a
x
a
D(a )
[ | a |,| a |]
significando que qualquer inteiro a 0 tem um número finito de divisores.
4.3. Divisores comuns de dois inteiros
Definição 4.2 Chama-se divisor comum de dois inteiros a e b todo inteiro d
d | a e d | b.
50
0 tal que
CAPÍTULO 4
DIVISIBILIDADE
Notação: D(a,b) = { x
Obs.: D(a,b)

*
| x | a e x | b} ou seja, D(a,b) = D(a)
D(b)
; D(0,0) =  *
Exemplo 4.1: Sejam os inteiros a = 12 e b = -15. temos:
D 12
D 15
1, 2, 3, 4, 6, 12
1, 3, 5, 15
Portanto:
D 12, 15
D 12
D
15
1, 3
4.4. Teorema da Divisão
O Teorema da divisão, que veremos a seguir, usado por Euclides no seu livro Elementos,
estabelece uma divisão com resto. É um teorema que foi "provado" uma vez através de um
algoritmo que explica como se processa a divisão, por esse motivo ficou conhecido como
Algoritmo de Euclides.
Teorema 4.2 Se a e b são dois inteiros, com b > 0, então existem e são únicos os inteiros q e r
que satisfazem às condições: a = bq + r e 0 r < b
Demonstração:
Existência
Seja S o conjunto de todos os inteiros não-negativos que são da forma a – bx, com x  , isto
é:
S = {a – bx ; x  , a – bx 0 }
Este conjunto S não é vazio, porque, sendo b > 0, temos b
resulta:
a – bx = a + b |a |
a+|a|
1 e, portanto, para x = - | a |,
0
Assim sendo, pelo “Princípio da boa ordenação”, existe o elemento mínimo r de S tal que
0
r e r = a – bq ou a = bq + r, com q 
51
CAPÍTULO 4
DIVISIBILIDADE
Além disso, temos r < b, pois, se fosse r b, teríamos:
0
r – b = a – bq – b = a – b( q+1 ) < r
isto é, r não seria o elemento mínimo de S.
Unicidade
Para demonstrar a unicidade de q e r, suponhamos que existem dois outros inteiros q 1 e r1 tais
que
a = bq1 + r1 e 0
r1 < b
Então, teremos:
r1 – r = b(q – q1)
bq1 + r1 = bq + r
b | (r1 – r)
por outro lado, temos:
-b<-r
0 e 0
r1 < b
o que implica:
- b < r – r1 < b,
isto é
| r1 – r | < b
Assim, b | (r1 – r) e | r1 – r | < b e, portanto: r1 – r = 0, e como b
q – q1 0. Logo, r1 = r e q1 = q. 
0, também temos
Nota: Aqui cabe uma pergunta: Por que o resto deve ser positivo? A resposta é
simples: Quando se divide a por b, o que se procura é o maior múltiplo de b que é
menor do que a, de modo que se
a =b q + r, então r = a – bq é positivo
porque a.b é menor do que a. Mas, não poderíamos definir divisão de modo que o
resto fosse negativo? Neste caso b.q seria o menor múltiplo de b maior do que a, e
teríamos situações como a do seguinte exemplo: para dividir R$10,00 entre 3
pessoas, cada uma delas receberia 4 reais e haveria um resto de -2(dívida de 2
reais? Quem iria pagar?) O exemplo mostra que esta maneira de se fazer a
divisão não teria muito valor prático. (RPM 8)
Corolário 4.1 Se a e b são dois inteiros com b 0, existem e são únicos os inteiros q e r que
satisfazem às condições: a = bq + r , 0 r < | b |
52
CAPÍTULO 4
DIVISIBILIDADE
Demonstração:
Com efeito, se b > 0, nada há que demonstrar, e se b < 0, então | b | > 0, e por conseguinte
existem e são únicos os inteiros q1 e r tais que
a = | b |q1 + r e 0
r<|b|
a = b(- q1 ) + r e 0
r< |b|
ou seja, por ser | b | = - b:
Portanto, existem e são únicos os inteiros q = - q1 e r tais que
a = bq + r e 0
r < | b |. 
Os inteiros a, b, q e r chamam-se respectivamente o dividendo, o divisor, o quociente e o
resto na divisão de a por b.
Nota: As demonstrações acima garantem a validade do Teorema de
Eudoxius:
Sejam a e b 0 inteiros, então,
I ) a é um múltiplo de b e, portanto, a= bq, q  ;
II ) a está situado entre dois múltiplos consecutivos de b, isto é, existe um
inteiro q tal que, para b > 0, bq a < b(q+1) e, para b < 0, bq a <
b(q-1).
Exemplo 4.2: Achar o quociente q e o resto r na divisão de a = 59 por b = -14 que satisfazem
as condições do algoritmo da divisão.
Efetuamos a divisão usual dos valores absolutos de a e b, obtemos:
59 14.4 3
o que implica:
59
Logo, o quociente q
14
4 e o resto r
4
3
e
0
3
14
3.
Exemplo 4.3: Achar o quociente q e o resto r na divisão de a = -79 por b = 11 que satisfazem
as condições do algoritmo da divisão.
Efetuamos a divisão usual dos valores absolutos de a e b, obtemos:
79 = 11.7 + 2
o que implica:
-79 = 11
7
2
53
CAPÍTULO 4
DIVISIBILIDADE
Como o termo r = 2 < 0 não satisfaz a condição 0 r 11 , somando e subtraindo o valor 11
de b ao segundo membro da igualdade anterior, obtemos:
79 11 7
11 11 2 11 8
9
com 0 9 11 . Logo, o quociente q = -8 e o r = 9.
Exemplo 4.4: Sejam os inteiros a = 1, -2, 61, -59 e b = -7. Temos:
1
7 .0 1 e
0 1
7
q
0 e r =1
e0 5
7
q 1 e r=5
61
7
8 5 e 0 5
7
q
8 e r=5
59
7 .9 4 e 0 4
7
q 9 e r=4
2
7 .1 5
4.5. Paridade de um Inteiro
Na divisão de um inteiro qualquer a por 2 os possíveis restos são r = 1 e r = 0.
Se r = 0 , então o inteiro a = 2q é denominado par; e se r = 1, então o inteiro a = 2q + 1 é
denominado ímpar, q  .
Dois inteiros que são ambos pares ou ambos ímpares dizem-se de mesma paridade, a dois
inteiros tais que um é par e o outro é ímpar, dizemos que tem paridades diferentes.
De modo geral, dado um inteiro a 2 , pode-se sempre escrever um inteiro qualquer n, de
modo único, na forma n aq r , onde k , r  e r a .
Teorema 4.3
1) A soma ou a diferença de dois números pares é par.
2) A soma ou a diferença de dois números ímpares é par.
3) A soma ou a diferença de um número par com um número ímpar é ímpar.
Demonstração:
1) Sejam a = 2k1 e b = 2k2, então a
b = 2k1
2) Sejam a = 2k1 +1 e b = 2k2 +1, então a
2k2 = 2(k1
b = (2k1 +1)
k2).
(2k2 +1) = 2(k1 + k2 + 1) ou 2(k1-
k2).
3) Sejam a = 2k1 e b = 2k2 +1, então a
54
b = 2k1
(2k2 +1) = 2(k1
k2) +1.
CAPÍTULO 4
DIVISIBILIDADE
Exemplo 4.5: Mostrar que o quadrado de qualquer inteiro ímpar é da forma 8k+1.
Com efeito, pelo algoritmo da divisão, qualquer inteiro é de uma das seguintes formas:
4q, 4q 1, 4q
2, 4q
3
Nesta classificação, somente os inteiros das formas 4q +1 e 4q +3 são ímpares e , portanto, os
seus quadrados são da forma:
4q 1
4q 3
2
8 2q 2 q
2
8 2q 2
1 8k 1
3q 1
1 8k 1
Assim, por exemplo, 7 e 13 são inteiros ímpares, e temos:
72
49 8.6 1
132 169 8.21 1
55
CAPÍTULO 4
DIVISIBILIDADE
EXERCÍCIOS
1) Mostrar que se a | b, então (-a) | b, a | (-b) e
(-a) | (-b).
13) Sendo m e n dois inteiros quaisquer, mostrar
que os inteiros m + n e m – n têm sempre a
mesma paridade.
2) Sejam a, b e c inteiros. Mostrar que:
14) Demonstrar que: Se “a” e “b” são inteiros
ímpares, então 8 | a2 – b2.
a ) se a | b, então a | bc.
b ) se a | b e se a | c, então a2 | bc.
c ) a | b se e somente se ac | bc (c
0).
3) Verdadeiro ou falso:
se a | (b + c), então a | b ou a | c.
4) Mostrar que, se a é um número inteiro
qualquer, então um dos inteiros a, a + 2, a + 4 é
divisível por 3.
5) Sendo a um inteiro qualquer, mostrar:
a ) 2 | a(a + 1).
b ) 3 | a(a + 1)(a + 2) .
6) Mostrar que um inteiro qualquer da forma 6k +
5 também é da forma 3t + 2.
7) Mostrar que todo inteiro ímpar é da forma 4k +
1 ou 4k + 3.
16) Verdadeiro ou falso: se a | c e se b | c, então a
| b.
17) Mostrar que a diferença entre os cubos de dois
inteiros consecutivos nunca é divisível por 2.
18) Na divisão do inteiro a = 427 por um inteiro
positivo “b”, o quociente é 12 e o resto é r.
Achar o divisor “b” e o resto “r”.
19) Na divisão do inteiro 525 por um inteiro
positivo o resto é 27. Achar os inteiros que
podem ser o divisor e o quociente.
20) Na divisão de dois inteiros positivos o
quociente é 16 e o resto é o maior possível.
Achar os dois inteiros, sabendo-se que sua
soma é 341.
8) Mostrar que o quadrado de um inteiro qualquer
é da forma 3k ou 3k + 1.
21) Achar os inteiros positivos menores que 150 e
que divididos por 39 deixam um resto igual ao
quociente.
9) Mostrar que o cubo de um inteiro qualquer é de
uma das formas 9k, 9k + 1 ou 9k + 8.
22) Seja d um divisor de n (d | n). Mostrar que cd |
n se e somente se c | (n/d).
10) Mostrar que:
23) Sejam n, r e s inteiros tais que 0 < r < n e 0 < s
< n. Mostrar que se n | (r – s) então r = s.
a)
n(n + 1)(2n + 1)/6 é um inteiro, qualquer
que seja o inteiro positivo n.
b) Se “a “ é um inteiro ímpar, então 24 | a
(a2 – 1).
11) Mostrar que se a | (2x – 3y) e se a | (4x – 5y),
então a | y.
12) Sendo a e b dois inteiros quaisquer, mostrar
que os inteiros a e a + 2b têm sempre a mesma
paridade.
56
15) Determinar os inteiros positivos que divididos
por 17 deixam um resto igual ao quadrado do
quociente.
24) Mostrar que o produto de dois inteiros ímpares
é um inteiro ímpar.
25) Demonstrar que se m e n são inteiros ímpares,
então 8 | (m4 + n4 – 2).
26) Demonstrar que 30 | (n5 – n)
27) Mostrar que, para todo inteiro n, existem
inteiros k e r tais que n = 3k + r e r = -1, 0, 1.
CAPÍTULO 4
DIVISIBILIDADE
28) Mostrar que (1 + 2 + . . . + n) | 3(1 2 + 22 + . . .
+ n2) para todo n > 1.
29) Mostre que todo inteiro ímpar, quadrado
perfeito, é da forma 4n + 1.
30) Na divisão de 392 por 45, determinar:
a)
o maior inteiro que se pode somar ao
dividendo sem alterar o quociente.
b) o maior inteiro que se pode subtrair ao
dividendo sem alterar o quociente.
31) Numa divisão de dois inteiros, o quociente é 16
e o resto 167. Determinar o maior inteiro que
se pode somar ao dividendo e ao divisor sem
alterar o quociente.
32) Achar o maior inteiro de quatro algarismos
divisível por 13 e o menor inteiro de cinco
algarismos divisível por 15.
33) Achar um inteiro de quatro algarismos,
quadrado perfeito, divisível por 27 e terminado
em 6.
34) Mostre que se a, b e c são inteiros ímpares, a
equação ax
racional.
2
bx
c
0 não tem raiz
35) Um tabuleiro 6 6 está coberto com dominós
2 1. Mostre que existe uma reta que separa as
peças do tabuleiro sem cortar nenhum dominó.
36) Dividindo-se o número 245 por um número
natural b, obtém-se quociente 5 e resto r.
Determine o valor da soma dos valores
possíveis para b.
37) A divisão de um certo número inteiro N por
1994 deixa resto 148. Calcule o resto da
divisão de N + 2000 pelo mesmo número 1994.
38) Considere quatro números inteiros a, b, c e d.
Prove que o produto: (a-b) . (c-a) . (d-a) . (d-c).
(d-b). (c-b) é divisível por 12.
n
39) Prove que a
ímpar.
b n é divisível por a+b se n é
57
Capítulo 5
MÁXIMO DIVISOR COMUM
5.1. Máximo Divisor Comum de Dois Inteiros
Definição 5.1 Sejam a e b dois inteiros não conjuntamente nulos (a 0 ou b 0). Chama-se
máximo divisor comum de a e b o inteiro positivo d (d 0) que satisfaz às condições:
1) d | a e d | b;
2) se c | a e se c | b, então c
d.
Observe-se que, pela condição (1), d é um divisor comum de a e b, e pela condição
(2), d é o maior dentre todos os divisores comuns de a e b.
O máximo divisor comum de a e b indica-se pela notação mdc(a,b).
É imediato que o mdc(a,b) = mdc(b,a). Em particular:
(i) o mdc(0,0) não existe.
(ii) o mdc(a,1) = 1
(iii) se a
0, então o mdc(a,0) = | a |
(iv) se a | b, então o mdc(a,b) = | a |
Assim, por exemplo:
mdc(8,1) = 1
mdc(-3,0) = | -3 | = 3
mdc(-6,12) = | -6 | = 6.
Exemplo 5.1 Sejam os inteiros a = 16 e b = 24. Os divisores comuns positivos de 16 e 24 são
1, 2, 4 e 8, e como o maior é 8, segue-se que o mdc(16,24) = 8.
Observa-se que
mdc(-16,24) = mdc(16,-24) = mdc(-16,-24) = 8.
Exemplo 5.2 Sejam os inteiros a = -24 e b = 60. Os divisores comuns positivos de –24 e 60
são 1, 2, 3, 4, 6 e 12, e como o maior deles é 12, segue-se que o mdc(-24,60) = 12.
58
CAPÍTULO 5
MÁXIMO DIVISOR COMUM
5.2. Existência e unicidade do MDC.
Teorema 5.1 (Identidade de Bézout ) Se a e b são dois inteiros não conjuntamente nulos ( a
0 ou b 0), então existe e é único o mdc(a,b); além disso, existem inteiros x e y tais que
mdc(a,b) = ax + by
Isto é, o mdc(a,b) é uma combinação linear de a e b.
Nota: Algumas fontes creditam este teorema ao matemático francês Claude
Gaspard Bachet de Méziriac e não ao também francês, Etienne Bézout.
Demonstração:
Seja S o conjunto de todos os inteiros positivos da forma au + bv, com u, v
S = { au + bv | au + bv
Este conjunto S não vazio (S
0 e u, v
), porque, por exemplo, se a
a = a.1 + b.0
e
Z, isto é:
Z}
0, então um dos dois inteiros:
-a = a.(-1) + b.0
é positivo e pertence a S. Logo, pelo “Princípio da boa ordenação”, existe e é único o
elemento mínimo d de S: minS = d 0. E, consoante a definição de S, existem inteiros x e y
tais que d = ax + by.
Posto isto, vamos mostrar que d = mdc(a,b). Com efeito, pelo algoritmo da divisão, temos:
a = dq + r, com 0
r
d
O que dá:
r = a – dq = a – (ax + by)q = a(1 – qx) + d(-qy)
Isto é, o resto r é uma combinação linear de a e b. Como 0 r d e d 0 é o elemento
mínimo de S, segue-se que r = 0 e a = dq, isto é, d | a.
Com raciocínio inteiramente análogo se conclui que também d | b. Logo, d é um divisor
comum positivo de a e b.
Finalmente, se c é um divisor comum positivo qualquer de a e b ( c | b, c 0), então:
c | (ax + by)
c|d
c
d
Isto é, d é o maior divisor comum positivo de a e b, ou seja:
mdc(a,b) = d = ax + by
e o teorema fica demonstrado.
x, y
Z
59
CAPÍTULO 5
MÁXIMO DIVISOR COMUM
Nota: A demonstração do teorema 1 deixa ver que o mdc(a,b) é o menor
inteiro positivo da forma ax + by, isto é, que pode ser expresso como
combinação linear de a e b. Mas, esta representação do mdc(a,b) como
combinação linear de a e b não pe punica, pois, temos:
mdc(a,b) = d = a(x + bt) + b(y - at)
qualquer que seja o inteiro t.
Importa ainda notar que, se
d = ar + bs.
Para algum par de inteiros r e s, então d não é necessariamente o mdc(a,b). Assim, por
exemplo, se:
mdc(a,b) = ax + by
então
t.mdc(a,b) = atx + bty
Para todo inteiro t, isto é:
d = ar + bd
onde d = t.mdc(a,b), r = tx e s = ty.
Exemplo 5.3 Sejam os inteiros a = 6 e b = 27. Temos:
mdc(6,27) = 3 = 6(-4 + 27t) + 27(1 – 6t)
qualquer que seja o inteiro t.
Exemplo 5.4 Sejam os inteiros a = -8 e b = -36. Temos:
mdc(-8,-36) = 4 = (-8)4 + (-36)(-1)
Teorema 5.2 Se a e b são dois inteiros não conjuntamente nulos (a
conjunto de todos os múltiplos do mdc(a,b) = d é
T = { ax + by | x,y
60
Z}
0 ou b
0), então o
CAPÍTULO 5
MÁXIMO DIVISOR COMUM
Demonstração:
Como d | a e d | b, segue-se que d | (ax + by), quaisquer que sejam os inteiros x e y, e por
conseguinte todo elemento do conjunto T e um múltiplo de d.
Por outro lado, existem inteiros x0 e y0 tais que
d = ax0 + by0,
de modo que todo múltiplo kd de d é da forma:
kd = k(ax0 + by0) = a(kz0) + b(ky0)
isto é, kd é uma combinação linear de a e b e, portanto, kd é elemento do conjunto T.
5.3. Inteiros Relativamente Primos (coprimos ou primos entre si)
Definição: Sejam a e b dois inteiros não conjuntamente nulos (a
são relativamente primos se, e somente se, o mdc(a,b) = 1.
0eb
0). Diz-se que a e b
Assim, por exemplo, são relativamente primos os inteiros: 2 e 5, -9 e 16, -27 e –35, pois,
temos:
mdc(2,5) = mdc(-9,16) = mdc(-27,-35) = 1
Dois inteiros a e b coprimos admitem como únicos divisores comuns 1 e –1.
Teorema 5.3 Dois inteiros a e b, não conjuntamente nulos (a
se, e somente se, existem inteiros x e y tais que ax + by = 1.
0eb
0), são primos entre si
Demonstração:
( ) Se a e b são relativamente primos, então o mdc(a,b) = 1 e por conseguinte existem
inteiros x e y tais que
ax + by = 1
( ) Reciprocamente, se existem inteiros x e y tais que ax + by = 1 e se o mdc(a,b) = d, então
d | a e d | b. Logo, d | (ax + by) e d | 1, o que implica d = 1 ou mdc(a,b) =1, isto é, a e b são
primos entre si.
Corolário 5.1 Se o mdc(a,b) = d, então o mdc( a/d , b/d ) = 1.
Demonstração:
Preliminarmente, observa-se que a/d e b/d são inteiros, porque d é um divisor comum de a e b.
Posto isso, se o mdc(a,b) = d, então existem inteiros x e y tais que ax + by = d, ou seja,
dividindo ambos os membros desta igualdade por d:
61
CAPÍTULO 5
MÁXIMO DIVISOR COMUM
(a/d)x + (b/d)y = 1
Logo, pelo teorema anterior, os inteiros a/d e b/d são primos entre si, isto é, o mdc
(a/d ,b/d) = 1.
Assim, por exemplo:
mdc(-12,30) = 6 e mdc(-12/6 , 30/6) = mdc(-2,5) = 1.
Corolário 5.2 Se a | b e se o mdc(b,c) = 1, então o
mdc (a,c) = 1.
Demonstração:
Com efeito:
a | b b = aq,
com q Z
mdc(b,c) = 1 bx + cy = 1, com x, y
Z.
Portanto:
a(qx) + cy = 1
mdc(a,c) = 1
Corolário 5.3 Se a | c, se b | c e se o mdc(a,b) = 1, então ab | c.
Demonstração:
Com efeito:
a|c
b|c
mdc(a,b) = 1
c = aq1,
c = bq2,
ax + by = 1,
acx + bcy = c
com q1
com q2
Z
Z
com x,y
Z
Portanto:
c = a(nq2)x = b(aq1)y = ab(q2x + q1y)
ab | c
Observe-se que somente as condições a | c e b | c não implicam ab | c.
Assim, por exemplo, 6 | 24 e 8 | 24, mas 6.8 | 24 (o mdc(6,8) = 2 1).
Corolário 5.4 Se mdc(a,b) = 1 = mdc(a,c), então o mdc(a,bc) = 1.
62
CAPÍTULO 5
MÁXIMO DIVISOR COMUM
Demonstração:
Com efeito:
mdc(a,b) = 1
mdc(a,b) = 1
ax + by = 1, com x,y
az + ct = 1, com z,t
Z
Z
Portanto:
1 = ax + by(az + ct) = a(x + byz) + bc(yt)
o que implica mdc(a,bc) = 1.
Corolário 5.5 Se o mdc(a,bc) = 1, então mdc(a,b) = 1 = mdc(a,c).
Demonstração:
Com efeito:
mdc(a,bc) = 1
ax + (bc)y = 1, com x,y
Z.
Portanto:
ax + b(cy) = 1
ax + c(by) = 1
mdc(a,b) = 1
mdc(a,c) = 1
Note-se que esta proposição é a recíproca da anterior.
Teorema 5.4 (de Euclides) Se a | bc e se o mdc(a,b) = 1, então a | c.
Demonstração:
Com efeito:
a | bc bc = aq,
com q Z
mdc(a,b) = 1 ax + by = 1, com x, y
acx + bcy = c
Z
Portanto:
c = acx + aqy = a(cx + qy)
Note-se que somente a condição a | bc não implica que a | c.
Assim, por exemplo, 12 | 9.8, mas 12 | 9 e 12 | 8 mdc(12,9)
a|c
1 e mdc(12,8)
1.
63
CAPÍTULO 5
MÁXIMO DIVISOR COMUM
5.4. Caracterização do MDC de Dois Inteiros
Teorema 5.5 Sejam a e b dois inteiros não conjuntamente nulos (a
positivo d (d 0) é o mdc(a,b) se e somente se satisfaz às condições:
(1) d | a e d | b
0 ou b
0). Um inteiro
(2) se c | a e se c | b, então c | d
Demonstração:
( ) Suponhamos que o mdc (a, b) = d. Então, d | a e d | b, isto é, a condição (1) é satisfeita.
Por outra parte, existem inteiros x e y tais que ax + by = d e, portanto, se c | a e se c | b, então
c | (ax + by) e c | d, isto é, a condição (2) também é satisfeita.
( ) Reciprocamente, seja d um inteiro positivo qualquer que satisfaz às condições (1) e (2).
Então, pela condição (2), todo divisor comum c de a e b também é divisor de d, isto é, c | d, e
isto implica c d. Logo, d é o mdc(a,b).
5.5. MDC de vários Inteiros
O conceito de máximo divisor comum, definido para dois inteiros a e b, estende-se de maneira
natural a mais de dois inteiros. No caso de três inteiros a, b e c, não todos nulos, o mdc(a,b,c)
é o inteiro positivo d (d 0) que satisfaz às condições:
(1) d | a, d | b e d | c
(2) se e | a, se e | b e se e | c, então e
d
Assim, por exemplo:
mdc(39,42,54) = 3 e mdc(49,210,350) = 7
Importa notar que três inteiros a, b e c podem ser primos entre si, isto é, o mdc(a,b,c) = 1, sem
que sejam primos entre si dois a dois, que é o caso, por exemplo, dos inteiros 6, 10 e 15.
Teorema 5.6 O mdc(a,b,c) = mdc(mdc(a,b),c).
Demonstração:
Com efeito, seja mdc(a,b,c) = d e mdc(a,b) = e. Então, d | a, d | b e d | c, e como existem
inteiros x e y tais que ax + by = e, segue-se que d | (ax + by) ou d | e, isto é, d é um divisor
comum de e e c (d | e e d | c).
64
CAPÍTULO 5
MÁXIMO DIVISOR COMUM
Por outro lado, se f é um divisor comum qualquer de e e c (f | e e f | c), então f | a, f | b e f |
c, o que implica f d.
Assim sendo, o mdc(e,c) = d, isto é, p mdc(mdc(a,b),c) = mdc(a,b,c).
Exemplo 5.5 Determinar o mdc(570,810,495).
Pelo teorema anterior, temos:
mdc(570,810,495) = mdc(mdc(570,810),495)
e como o mdc(570,810) = 30, segue-se que o
mdc(570,810,495) = mdc(30,495) = 15
EXERCÍCIOS
1.
Determinar:
a)
mdc(11, 99)
9.
Sendo a e b dois inteiros não conjuntamente
nulos (a 0 ou b 0), mostrar: mdc (a, b) =
mdc (-a, b) = mdc (a, -b) = mdc (-a, -b).
b) mdc(-21,14)
c)
mdc(17, 18)
10. Sejam a, b e c inteiros. Demonstrar:
a)
1.
Achar os elementos do conjunto A = {1, 2, 3,
4, 5} que são relativamente primos com 8.
2.
Seja o conjunto A = {1, 2, 3, 4, 5, 6}.
Enumerar os elementos do conjunto X = {x
A | mdc(x, 6) = 1}.
3.
Sabendo que o mdc(a, 0) = 13, achar todos os
valores do inteiro a.
4.
Achar o menor inteiro positivo c, da forma c =
22x + 55y, onde x e y são dois inteiros.
Sendo n um inteiro qualquer, calcular o mdc(n,
n + 1).
5.
6.
Calcular
a)
mdc(n, n + 2), sendo n um inteiro par.
b) se existem inteiros x e y tais que ax + by =
mdc(a, b) então mdc(x, y) = 1.
11. Sejam a, b e c inteiros. Demonstrar:
a)
8.
Sendo n um inteiro qualquer,
achar os
possíveis valores do máximo divisor comum
dos inteiros n e n + 10.
Sendo n um inteiro qualquer, calcular o mdc(n
– 1, n2 + n + 1).
se o mdc(a, b) = 1 então o mdc (ac, b) =
mdc (b, c)
b) Se o mdc(a, b) = 1 e se c | (a + b), então o
mdc(a, c) = 1 e o mdc(b, c) = 1.
c)
se b | c, então o mdc(a, b) = mdc (a + c, b).
d) Se mdc (a, b) = 1, então mdc (am, bn) = 1.
e)
Se mdc (a, b)= 1, então
ab+b²) =1 ou 3
f)
O mdc (a,b) = mdc (a, b + ac), com c
positivo.
b) mdc(n, n + 2), sendo n um inteiro ímpar.
7.
existem inteiros x e y tais que c = ax + by
se e somente se o mdc(a, b) | c.
mdc (a+b, a²-
12. Calcular o mdc (a + b, a – b) sabendo que a e b
são inteiros primos entre si.
13. Seja 10 a 120 e mdc ( a, 120 ) =10.
Determine o valor de a.
65
CAPÍTULO 5
MÁXIMO DIVISOR COMUM
14. Achar o maior inteiro positivo pelo qual se
devem dividir os inteiros 160, 198 e 370 para
que os restos sejam respectivamente 7, 11 e 13.
31. O mdc(a, b) = p, sendo p um primo. Achar os
possíveis valores do
a) mdc (a2, b)
b) mdc(a3, b) = p, mesma conclusão acima.
15. Determinar os inteiros positivos a e b, sabendose que:
a)
a + b = 63 e mdc(a, b) = 9
b) ab = 756 e mdc(a, b) = 6.
16. Os restos das divisões dos inteiros 4933 e 4435
por um inteiro positivo n são respectivamente
37 e 19. Achar o inteiro n.
17. Demonstrar que se n = abc + 1, então o mdc(n,
a) = mdc(n, b) = mdc(n, c) = 1.
18. Demonstrar que mdc(mdc(a, b), b) = mdc(a, b)
19. Demonstrar que o mdc(n + k, k) = 1 se e
somente se o mdc(n, k) = 1.
20. Demonstrar que, se a | bc e se mdc(a, b) = d,
então a | cd.
21. Demonstrar que, se a | c, se b | c e se o mdc(a,
b) = d então ab | cd.
22. Demonstrar que se mdc(a, b) = 1 e se mdc(a,c)
= d,então mdc(a, bc) = d.
c)
mdc(a2, b3) = p2. Pois aparecem 2 fatores
iguais a p em a2 e 3 fatores iguais a p em
b3.
32. Sabendo que o mdc(a, p2) = p e que o mdc(b,
p3) = p2, onde p é um primo, calcular o mdc
(ab, p4) e o mdc(a + b, p4).
33. Demonstrar que se o mdc(a, b) = d então o mdc
(a2, b2) = d2.
34. Demonstrar que mdc(a, b) = mdc(a, c) implica
mdc(a2, b2) = mdc(a2, c2).
35. Sejam a e k inteiros não conjuntamente nulos.
Demonstrar que mdc(a, a + k) | k.
36. Demonstrar que mdc(a, b) = mdc(a, c) implica
mdc(a, b) = mdc(a, b, c).
37. Demonstrar que mdc(a, b, c) = mdc(mdc(a, b),
mdc(a, c).
38. Sejam a e b inteiros positivos tais que ab é um
quadrado perfeito e o mdc(a, b) = 1.
Demonstrar que a e b são quadrados perfeitos.
39. Demonstrar que mdc( a + b, a – b) > mdc(a, b)
23. O inteiro ímpar d é um divisor de a + b e de a –
b. Demontrar que d também é um divisor do
mdc(a, b).
24. Os inteiros positivos a, b e c são tais que o
mdc(a, b) = 1, a | c e c | b. Demonstrar que a =
1.
25. O mdc(n, n + k) = 1 para todo inteiro positivo
n. Demonstrar que k = 1 ou k = -1.
26. Demonstrar que mdc(a, b) = mdc(a + kb, b)
para todo inteiro k.
27. O mdc(a, 4) = 2 = mdc(b, 4). Demonstrar que o
mdc(a + b, 4) = 4.
28. Os inteiros positivos m e n são tais que o
mdc(m, n) = d. Mostrar que o mdc (2m – 1, 2n –
1) = 2d – 1.
29. Demonstrar que mdc(a, b) = mdc(a, b, a + b).
30. Demonstrar que mdc(a, b) = mdc(a, b, ax +
by), quaisquer que seja os inteiros x e y.
66
40. Mostrar que o mdc (5n + 6, 5n + 8) = 1 onde n é
um inteiro ímpar.
41. Sejam a, b, c, d (b d) inteiros tais que mdc(a,
b) = mdc(c, d) = 1. Mostrar que a soma a/b +
c/d não é um inteiro.
42. Determinar os inteiros positivos a e b, sabendo
que a2 – b2 = 7344 e mdc(a, b) = 12.
43. Dividindo-se dois inteiros positivos pelo seu
mdc, a soma dos quocientes é 8. Determinar os
dois inteiros, sabendo-se que sua soma é 384.
44. Um enxadrista quer decorar uma parede
retangular, dividindo-a em quadrados, como se
fosse um tabuleiro de xadrez. A parede mede
4,40 metros por 2,75 metros. Qual o menor
número de quadrados que ele pode colocar na
parede?
Capítulo 6
ALGORITMO DE EUCLIDES –
MÍNIMO MÚLTIPLO COMUM
P
ouco se sabe sobre a vida e a personalidade de Euclides e se desconhece a data de seu
nascimento. É provável que sua formação matemática tenha se dado na escola platônica
de Atenas. Ele foi professor do Museu em Alexandria.
Euclides escreveu cerca de uma dúzia de tratados, cobrindo tópicos desde óptica, astronomia,
música e mecânica até um livro sobre secções cônicas; porém, mais da metade do que ele
escreveu se perdeu. Entre as obras que sobreviveram até hoje temos: Os elementos, Os dados,
Divisão de figuras, Os fenômenos e Óptica.
Os elementos de Euclides não tratam apenas de geometria, mas também de teoria dos
números e álgebra elementar (geométrica). O livro se compõe de quatrocentos e sessenta e
cinco proposições distribuídas em treze livros ou capítulos, dos quais os seis primeiros são
sobre geometria plana elementar, os três seguintes sobre teoria dos números, o livro X sobre
incomensuráveis e os três últimos tratam sobre geometria no espaço.
O livro VII começa com o processo, hoje conhecido como algoritmo euclidiano, para achar o
máximo divisor comum de dois ou mais números inteiros e o usa para verificar se dois
inteiros são primos entre si; encontramos também uma exposição da teoria das proporções
numérica ou pitagórica.
http://www.matematica.br/historia/euclides.html
Comecemos com o seguinte Lema:
Lema: Se a = bq + r, então o mdc(a,b) = mdc(b,r).
Demonstração:
Se o mdc(a,b) = d, então d | a e d | b, o que implica d | (a - bq) ou d | r, isto é, d é um divisor
comum de b e r (d | b e d | r).
Por outro lado, se c é um divisor comum qualquer de b e r (c | b e c | r), então c | (bq + r) ou
c | a, isto é, c é um divisor comum de a e b, o que implica c d. Assim sendo, o mdc(b,r) = d.
6.1. Algoritmo de Euclides
Sejam a e b dois inteiros não conjuntamente nulos ( a
comum se deseja determinar.
0 ou b
0 ) cujo máximo divisor
67
CAPÍTULO 6
ALGORITMO DE EUCLIDES – MÍNIMO MÚLTIPLO COMUM
Sejam a e b dois inteiros não conjuntamente nulos ( a
comum se deseja determinar.
0 ou b
0 ) cujo máximo divisor
É imediato:
(1) se a 0, então o mdc(a,0) = | a |
(2) se a 0, então o mdc(a,a) = | a |
(3) se b | a , então o mdc(a,b) = | b |
Além disso, por ser mdc(a,b) = mdc( | a | , | b | ), a determinação do mdc(a,b) reduz-se ao caso
em que a e b são inteiros positivos distintos, por exemplo, com a b, tais que b não divide a,
isto é: a b 0 e b | a. nestas condições, a aplicação repetida do algoritmo da divisão dános as igualdades:
a = bq1 + r1,
b = r1q2 + r2,
r1 = r2q3 + r3,
r2 = r3q4 + r4,
0 r1 b
0 r2 r1
0 r3 r2
0 r4 r3
..............................
........................
Como os restos r1, r2, r3, r4, ... são todos inteiros positivos tais que
b
r1 r2 r3 r4 ...
e existem apenas b – 1 inteiros positivos menores que b, necessariamente se chega a uma
divisão cujo resto rn+1 = 0, isto é, finalmente, teremos:
rn-2 = rn-1qn + rn,
0 rn rn-1
rn-1 = rnqn+1 + rn+1,
rn-1 = 0
O último resto rn 0 que aparece nesta sequência de divisões é o máximo divisor comum
procurado de a e b, isto é, o mdc(a,b) = rn, visto que, pelo lema anterior, temos:
mdc(a,b) = mdc(b,r1) = mdc(r1,r2) = ... =
= mdc(rn-2,rn-1) = mdc(rn-1,rn) = rn
este processo prático para o cálculo do máximo divisor comum de dois inteiros positivos a e b
é denominado algoritmo de EUCLIDES ou processo das divisões sucessivas.
É usual o seguinte dispositivo de cálculo no emprego do algoritmo de EUCLIDES:
a
r1
68
q1
b
r2
q2
r1
r3
q3
r2
r4
...
qn
rn-1
0
qn+1
rn
CAPÍTULO 6
ALGORITMO DE EUCLIDES – MÍNIMO MÚLTIPLO COMUM
Que se traduz na seguinte REGRA: Para se “ achar” o mdc de dois inteiros positivos, dividi-se
o maior pelo menor, este pelo primeiro resto obtido, o segundo resto pelo primeiro, e assim
sucessivamente até encontrar um resto nulo. O último resto não nulo é o máximo divisor
comum procurado.
O algoritmo de EUCLIDES também pode ser usado para achar a expressão do mdc(a,b) = rn
como combinação linear de a e b, para o que basta eliminar sucessivamente os restos rn-1, rn-2,
..., r3, r2, r1 entre as n primeiras igualdades anteriores.
Exemplo 6.1 Achar o mdc(963,657) pelo algoritmo de EUCLIDES e a sua expressão como
combinação linear de 963 e 657,
Temos, sucessivamente:
963 = 657.1 + 306
657 = 306.2 + 45
306 = 45.6 + 36
45 = 36.1 + 9
36 = 9.4 + 0
963
1
657
306
2
306
45
6
45
36
1
36
9
4
9
0
Portanto, o mdc(963,657) = 9 e a sua expressão como combinação linear de 963 e 657 se
obtém eliminando os restos 36, 45 e 306 entre as quatro primeiras igualdades anteriores do
seguinte modo:
9 = 45 – 36 = 45 – (306 – 45.6) =
= - 306 + 7.45 = - 306 + 7(657 – 306.2) =
= 7.657 – 15.306 = 7.657 – 15(963 – 657) =
= 963(-15) + 657.2
isto é:
9 = mdc(963,657) = 963x + 657y
onde x = -15 e y = 22.
Esta respresentação do inteiro 9 = mdc(963,657) como combinação linear de 963 e 657 não é
única. Assim, por exemplo, somando e subtraindo o produto 963.657 ao segundo mebro da
igualdade:
9 = 963(-15) + 657.22
obtemos:
9 = 963(-15 + 657) + 657(22 - 963) =
= 963.642 + 657(-941)
69
CAPÍTULO 6
ALGORITMO DE EUCLIDES – MÍNIMO MÚLTIPLO COMUM
que é uma outra representação do inteiro 9 = mdc(963,657) como combinação linear de 963
e 657.
Exemplo 6.2 Achar o mdc(252,-180) pelo algoritmo de EUCLIDES e a sua expressão como
combinação linear de 252 e –180.
Temos, sucessivamente:
252 = 180.1 + 72
180 = 72.2 + 36
72 = 36.2
Portanto, o mdc(252,-180) = mdc(252,180) = 36 e como
36 = 180 – 72.2 = 180 – (252 – 180)2 =
= 252(-2) + (-180)(-3)
temos:
36 = mdc(252,-180) = 252x + (-180)y
onde x = -2 e y = -3, que é a expressão do mdc(252,-180) como combinação linear de 252 e –
180.
Outra representação do inteiro 36 = mdc(252,-180) como combinação linear de 252 e –180 é
a seguinte:
36 = 252(-2 + 180) + (-180)(-3 + 252) =
= 252.178 + (-180)249
Exemplo 6.3 O mdc de dois inteiros positivos a e b é 74 e na sua determinação pelo algoritmo
de EUCLIDES os quocientes obtidos foram 1, 2, 2, 5, 1 e 3. Calcular a e b.
a
1
b
r
2
r
r1
2
r1
r2
5
r2
r3
1 3
r3 74
74 0
Temos, sucessivamente:
a = b + r, b = 2r + r1, r = 2r1 + r2
r1 = 5r2 + r3, r1 = r2 + 74, r3 = 74.3 = 222
70
CAPÍTULO 6
ALGORITMO DE EUCLIDES – MÍNIMO MÚLTIPLO COMUM
Portanto:
r2 = 222 + 74 = 296, r1 = 5.296 + 222 = 1702
r = 2.1702 + 296 = 3700, b = 2.3700 + 1702 = 9102
a = 9102 + 3700 = 12802
Nota: Dispositivo prático para expressar o MDC...(José Paulo Q. Carneiro)
Antes de deduzir o dispositivo prático, vamos vê-lo funcionando no exemplo
citado. Forma-se uma matriz com duas colunas, intituladas “q” e “s, t”.
Primeiramente, preenche-se a coluna “q” com os quocientes obtidos no
algoritmo de Euclides (desprezando o último, correspondente ao resto zero),
na ordem contrária ao de seu aparecimento no algoritmo, e deixando em
branco a primeira linha.
Na coluna “s, t”, coloca-se o número 1 na primeira linha (é sempre 1
mesmo), e na segunda linha repete-se o valor do quociente que
aparece ao seu lado na primeira coluna. No nosso exemplo, fica:
A partir daí, cada valor seguinte da coluna “s, t” vai sendo obtido de acordo com o seguinte
esquema auto-explicativo:
Observa-se que os dois últimos valores obtidos na segunda
coluna são, justamente, em valor absoluto, s = 85e t = 539. Isso
não é coincidência. Na realidade, essa matriz resume as contas
que precisam ser feitas com os números relevantes que figuram
no processo.
Há a questão dos sinais, isto é, o esquema só fornece os valores absolutos de s e t. Para
decidir sobre os sinais, aplica-se a seguinte regra: se o número de quocientes aproveitados
(ou seja, o número de linhas preenchidas na coluna “q”) for ímpar, então s é positivo e t
negativo; se for par, ocorrerá justamente o contrário: s é negativo e t é positivo. Deve ser
notado que se pode também, em vez de decorar mais uma regra, experimentar, comparando
os valores de
e
. No exemplo, | s | a = 85 x 7248 = 616080 e | t | b = 539 x 1143 =
616077, ficando claro que, para obter o mdc 3, é necessário fazer s = 85 e t = –539.
Outro exemplo: a = 1741 e b = 85
1741
20
2
13
1
2
85
41
3
2
1
41
3
2
1
0
71
CAPÍTULO 6
ALGORITMO DE EUCLIDES – MÍNIMO MÚLTIPLO COMUM
Formando então a matriz:
Como agora o número de quocientes aproveitados é par, tomase: s = 29 s = –29 e t = 549, de modo que
( 29) x 1741 + (594) x 85 = 50 489 + 40 490 = 1
Justificativa do dispositivo prático
Vamos agora justificar o dispositivo prático,
por meio da formulação literal do problema.
Supondo
> b > 0 inteiros, aplica-se o
algoritmo de Euclides, encontrando:
onde rn = m = mdc (a, b). Os quocientes já foram numerados de trás para frente, isto é, o
último quociente (correspondente ao resto 0) é q0, o penúltimo é q1, etc., e o primeiro é qn,
para manter coerência com o dispositivo prático.
Desprezando, como se fez nos exemplos, o último quociente q0, vê-se, da primeira equação,
que r1 pode ser escrito como uma combinação linear de a e b, ou seja, r1 = a – qnb.
Substituindo esse valor na penúltima equação, vê-se que r2 também pode ser escrito como
uma combinação linear de a e b, a saber, r2 = –qn-1a + (1 + qnqn–1)b. E assim por diante,
para todos os r. Em particular, calcula-se m = n = sa+ tb.
Repare que acabamos de demonstrar o Teorema Fundamental da Teoria dos Números. Se o
leitor não estiver satisfeito com o “e assim por diante”, pode formalizar a demonstração por
indução. É realmente fácil. Basta observar que uma combinação linear de combinações
lineares de a e b é também uma combinação linear de a e b.
n
s
1
2
3
4
...
1
...
t
...
A lei de formação é clara (e pode ser verificada por indução):
para n 1:
com os valores iniciais: s1 = 1 e t1 = –q1.
72
sk
tk
tk
1
qk t k
1
Sk
1
CAPÍTULO 6
ALGORITMO DE EUCLIDES – MÍNIMO MÚLTIPLO COMUM
Observando que cada s repete o t anterior, basta trabalhar com os t e notar que, ao final do
processo, o último valor é o t procurado, enquanto o penúltimo é o s. Foi por isso que
introduzimos a coluna “s, t” na matriz da nossa regra prática. Além disso, levando em conta a
alternância dos sinais, é suficiente lidar com os valores absolutos de t, tomando o cuidado
de, no final, fazer a correção de sinal conveniente, conforme n seja par ou ímpar. Com essas
providências, a lei de formação fica:
| tx | = qx | tx -1 | + | tx-2 |,
que é justamente o que se fez na regra prática.
Alguém pode ainda perguntar: quando se escreve o máximo divisor comum de dois inteiros
como uma combinação linear deles, essa representação é única? A resposta é não (ver [4],
para maiores detalhes). Uma vez encontrados s e t tais que
, basta tomar um
inteiro qualquer u, e também será verdade que m = s’ a + t’b, onde s’ = s + bu e t’ = t – au,
como pode ser verificado diretamente por substituição.
Em nosso primeiro exemplo, tomando
, temos:
s' = 85 + 1143 = 1228, enquanto t’ = – 539 – 7248 = – 7728
Verificando: s'a + t'b= 1 228 x 7 248 + ( 7 787) x 1 143 = 8900544 – 8900541 = 3
Teorema 6.1 Se k
0, então o mdc(ka,kb) = k.mdc(a,b).
Demonstração:
Multiplicando ambos os membros de cada uma das n+1 igualdades que dão o mdc(a,b) = rn
pelo algoritmo de EUCLIDES por k, obtemos:
ak = (bk)q1 + r1k, 0 r1k bk
bk = (r1k)q2 + r2k, 0 r2k r1k
r1k = (r2k)q3 + r3k, 0 r3k r2k
............................. .....................
rn-2k = (rn-1k)qn + rnk, 0 rnk rn-1k
rn-1k = (rnk)qn+1 + 0
Obviamente, estas n+1 igualdades outra coisa não são que o algoritmo de EUCLIDES
aplicado aos inteiros ak e bk, e por conseguinte o mdc(ak,bk) é o último resto rnk 0, isto é:
mdc(ak,bk) = rnk = k.mdc(a,b)
73
CAPÍTULO 6
ALGORITMO DE EUCLIDES – MÍNIMO MÚLTIPLO COMUM
Assim, por exemplo:
mdc(12,30) = mdc(2.6,5.6) = 6.mdc(2,5) = 6.1 = 6
Corolário 6.1 Para todo k
0, o mdc(ka,kb) = | k |.mdc(a,b)
Demonstração:
Se k 0, nada há que demonstrar, e se k 0, então –k = | k | 0 e, pelo teorema anterior,
temos:
mdc(ak,bk) = mdc(-ak,-bk) =
= mdc(a.| k |, b.| k |) = | k |.mdc(a,b)
6.2 . Múltiplos comuns de dois inteiros
O conjunto de todos os múltiplos de um inteiro qualquer a
M(A) = {x
Z | a | x } = {aq | q
0 indica-se por M(a), isto é:
Z}
Assim, por exemplo:
M(-1) = M(1) = Z
M(5) = {5q | q
É imediato que, para todo inteiro a
Z} = {0, +5, +10, +15, +20, ...}
0, se tem M(a) = M(-a).
Definição 6.1 Sejam a e b dois inteiros diferentes de zero (a
comum de a e b todo inteiro x tal que a | x e b | x.
0eb
0). Chama-se múltiplo
Em outros termos, múltiplo comum de a e b é todo inteiro que pertence simultaneamente aos
conjuntos M(a) e M(b).
O conjunto de todos os múltiplos comuns de a e de b indica-se por M(a,b). Portanto,
simbolicamente:
M(a,b) = {x
Z | a | x e b | x}
Ou seja:
M(a,b) = {x
Z|x
M(a) e x
E, portanto:
M(a,b) = M(a)
74
M(b)
M(b)}
CAPÍTULO 6
ALGORITMO DE EUCLIDES – MÍNIMO MÚLTIPLO COMUM
A interseção ( ) é uma operação comutativa, de modo que M(a,b) não depende da ordem
dos inteiros dados a e b, isto é: M(a,b) = M(b,a).
Obviamente, 0 é um múltiplo comum de a e b: 0
são múltiplos comuns de a e b.
M(a,b). E os produtos ab e –(ab) também
Exemplo 6.4 Sejam os inteiros a = 12 e b = -18. Temos:
M(12) = {12q | q
M(-18) = {-18q | q
Z} = {0, +12, +24, +36, +48, +60, +72, ...}
Z} = {0, +18, +36, +54, +72, +90, ...}
Portanto:
M(12,-18) = M(12)
M(-18) = {0, +36, +72, ...}
6.3. Mínimo Múltiplo comum de dois inteiros
Definição 6.2 Sejam a e b dois inteiros diferentes de zero (a 0 e b 0). Chama-se mínimo
múltiplo comum de a e b o inteiro positivo m (m 0) que satisfaz às condições:
(1)
(2)
a|m e b|m
se a | c e se b | c, com c
0, então m
c.
Observe-se que, pela condição (1), m é um múltiplo comum de a e b, e pela condição (2), m é
o menor dentre todos os múltiplos comuns positivos de a e b.
O mínimo múltiplo comum de a e b indica-se pela notação mmc(a,b).
Pelo “Princípio da boa ordenação” , o conjunto dos múltiplos comuns positivos de a e b
possui o elemento mínimo e, portanto, o mmc(a,b) existe sempre e é único. Além disso, por
ser o prodtudo ab um múltiplo comum de a e b, segue-se que o mmc(a,b) |ab|.
Em particular, se a | b, então o mmc(a,b) = | b |.
Exemplo 6.5 Sejam os inteiros a = -12 e b = 30. Os múltiplos comuns positivos de –12 e 30
são 60, 120, 180, ..., e como o menor deles é 60, segue-se que o mmc(-12,30) = 60.
Relação entre mdc e o mmc
Teorema 6.2 Para todo par de inteiros positivos a e b subsiste a relação:
mdc(a,b).mmc(a,b) = |ab|
75
CAPÍTULO 6
ALGORITMO DE EUCLIDES – MÍNIMO MÚLTIPLO COMUM
Demonstração:
Seja mdc(a,b) = d e mmc(a,b) = m. Como a | a(b/d) e b | b(a/d), segue-se que ab/d é um
múltiplo comum de a e b. Portanto, existe um inteiro positivo k tal que
ab/d = mk,
k
N
o que implica:
a/d = (m/b)k
e
b/d = (m/a)k
isto é, k é um divisor comum dos inteiros a/d e b/d. Mas, a/d e b/d são primos entre si
(corolário 1 desta apostila), de modo que k = 1. Assim sendo, temos:
ab/d = m ou ab = dm
isto é:
ab = mdc(a,b).mmc(a,b)
Esta importante relação permite determinar o mmc de dois inteiros quando se conhece o seu
mdc, e vice-versa.
Exemplo 6.6 Determinar o mmc(963,657).
Pelo algoritmo de EUCLIDES, temos mdc(963,657) = 9. Portanto:
mmc(963,657) =
963 .657
=70299
9
Corolário 6.2 Para todo par de inteiros positivos a e b, o mmc(a,b) = ab se e somente se o
mdc(a,b) = 1.
Demonstração:
( ) Se o mdc(a,b) = 1, então:
ab = 1.mmc(a,b) = mmc(a,b)
( ) Reciprocamente, se o mmc(a,b) = ab, então:
mdc(a,b).ab = ab
mdc(a,b) = 1
6.5. MMC de vários inteiros
O conceito de mínimo múltiplo comum, definido para dois inteiros a e b, estende-se de
maneira natural a mais de dois inteiros. No caso de três inteiros a, b e c, diferentes de zero, o
mmc(a,b,c) é o inteiro positivo m (m 0) que satisfaz às condições:
76
CAPÍTULO 6
ALGORITMO DE EUCLIDES – MÍNIMO MÚLTIPLO COMUM
(1) a | m, b | m e c | m
(2) se a | e, se b | e e se c | e, com e
0, então m
e.
Assim, por exemplo: mmc (39,102,75) = 33150.
Exemplo 6.7
Achar inteiros x, y e z que verifiquem a seguinte igualdade 11x + 19y + 3z = 1.
Achando o mdc(11,19)
1
19
11
8
1
8
3
2
3
2
1
2
1
2
1
0
Usando o algoritmo da divisão, podemos escrever:
19 = 11 x 1 + 8
11 = 8 x 1 + 3
8=3x2+2
3=2x1+1
2=1x2
Desprezando a última igualdade, eliminemos os restos a partir da penúltima igualdade:
1=3–2
1 = 3 – (8 – 3 x 2)
1= 3x3–8
1 = (11 – 8) x 3 – 8
1 = 11 x 3 – 8 x 4
1 = 11 x 3 – (19 – 11) x 4
1 = 11 x 7 – 19 x 4
(I)
Achemos agora o mdc(3,1) . Como mdc(3,1) = 1 , vamos escrever este mdc como
combinação de 3 e 1:
1 = 3 x 1 + 1 x (-2) ( II )
Agora substituamos o valor de 1, dado na igualdade ( I ) , na igualdade ( II ):
1 = 3 x 1 + (11 x 7 – 19 x 4) (–2)
1 = 3 x 1 + 11 (–14) + 19 x 8
ou
1 = 11 (–14) + 19(8) + 3(1) .
Logo x = –14, y = 8 e z = 1
77
CAPÍTULO 6
ALGORITMO DE EUCLIDES – MÍNIMO MÚLTIPLO COMUM
EXERCÍCIOS
1.
Usando o algoritmo de Euclides, determinar:
a)
b)
c)
d)
e)
f)
2.
mdc(306, 657)
mdc(272, 1479)
mdc(884, 1292)
mdc(-816, 7209)
mdc(7469, 2387)
mdc(-5376,-3402)
Usando o algoritmo de Euclides, achar os
inteiros x e y que verifiquem cada uma das
seguintes igualdades:
a)
b)
c)
d)
4.
78
a)
b)
c)
d)
e)
f)
g)
h)
Usando o algoritmo de Euclides, determinar:
a) mdc(285, 675, 405)
b) mdc(209, 299, 102)
c) mdc(69, 398, 253)
3.
Calcula-se o mdc pelo algoritmo de Euclides e
a seguir divide o produto ab pelo mdc(a, b)
mdc(56, 72) = 56x + 72y
mdc(24, 138) = 24x + 138y
mdc(119, 272)
mdc(1769, 2378) = 1769x + 2378y
Achar os inteiros x e y que verifiquem cada
uma das seguintes igualdades:
a) 78x + 32y = 2.
b) 104x + 91y = 13
c) 31x + 19y = 7
d) 42x + 26y = 16.
e) 288x + 51x = 3.
f) 52x + 13y = 1
g) 145x + 58y = 87
h) 17x + 5y = -2
5.
Achar os inteiros x, y e z que verifiquem cada
uma das seguintes igualdades.
a) 11x + 19y +5z = 1.
b) 56x + 6y + 32z = 2.
c) 6x + 3y + 15z = 9.
d) 14x + 7y + 21z = 4.
6.
Achar inteiros x, y e z que verifiquem a
igualdade 198x + 288y + 512z = mdc(198,
288, 512)
7.
Calcular
As soluções de todos os itens podem ser
obtidas a partir da propriedade mdc(a,
b).mmc(a, b) = a . b.
mmc( 45, 21).
mmc(83, 68)
mmc( 120, 110)
mmc(86, 71)
mmc(224, 192)
mmc(1287, 507)
mmc(143, 227)
mmc(306, 657)
8.
O mdc de dois inteiros positivos a e b é 8 e na
sua determinação pelo algoritmo de Euclides os
quocientes sucessivamente obtidos foram 2, 1,
1 e 4. Calcular a e b.
9.
Determinar os inteiros positivos a e b sabendo:
a) ab = 4032 e mmc(a, b) = 336
b) mdc(a, b) = 8 e mmc(a, b) = 560
c) a + b = 589 e mmc(a, b)/mdc(a,b) = 84
10. Demonstrar que se a e b são inteiros positivos
tais que o mdc(a, b) = mmc(a, b) então a = b.
11. Sabendo que o mdc(a,b) = 1 , demonstrar:
a) mdc(2 a + b, a + 2 b) = 1 ou 3
b) mdc(a + b, a2 + b2) = 1 ou 2
c) mdc(a + b, a2 – ab + b2) = 1 ou 3
12. Sendo a e b inteiros positivos, demonstrar que
o mdc(a, b) sempre divide o mmc(a, b).
13. Quantos pares de inteiros positivos A e B
existem cujo mínimo múltiplo comum é
126000?
Considere o par (A,B) como sendo o mesmo
que (B,A)
14. Calcule a soma dos números entre 200 e 500
que são múltiplos de 6 ou de 14, mas não
simultaneamente múltiplos de ambos."
15. O máximo divisor comum, o menor divisor
comum e o mínimo múltiplo comum dos
números 4, 8 e 12, são, respectivamente.
Capítulo 7
NÚMEROS PRIMOS
7.1. Introdução
A noção de número primo foi, muito provavelmente, introduzida por Pitágoras,
530 AC,
sendo que a mesma desempenhou um papel central tanto na matemática como no misticismo
pitagórico.
A escola pitagórica dava grande importância ao número um, que era chamada de unidade (em
grego: Monad). Os demais números inteiros naturais – o 2, 3, 4, etc – tinham caráter
subalterno, sendo vistos como meras multiplicidades geradas pela unidade e por isso recebiam
a denominação de número (em grego: Arithmós).
Entre os pitagóricos a preocupação com a geração dos números não parava por aí. Já o próprio
Pitágoras teria atinado que existem dois tipos de arithmós:
•
Os protoi arithmós (números primários ou primos), que são aqueles que não podem ser
gerados – através da multiplicação – por outros arithmós, como é o caso de 2, 3, 5, 7...
•
Os deuterói arithmós (números secundários), podem ser gerados por outros arithmós, por
exemplo, 4 = 2.2, 6 = 3.2, etc.
Ainda por influência dos Pitagóricos , por muitos séculos houve polemica a respeito da
primalidade do número dois. Os primeiros pitagóricos chamavam-lhe Dyad, atribuíam-lhe
caráter especial – embora menos importante que a unidade Monad – e alguns deles não o
incluíam entre os arithmós. Consequentemente, muitos pitagóricos não consideravam o dois
como primo. É só pela época de Aristóteles,
350 AC, que passou a ser considerado como
primo, sendo que este costume foi consagrado pelo livro Elementos de Euclides em cerca de
300 AC. Cabe mencionar que entre os gregos, principalmente os pitagóricos de
várias gerações após Pitágoras, surgiram outras denominações para os númerosprimos,
como: retilíneos, lineares e eutimétricos. Contudo, esta nomenclatura teve uso muito restrito e
caíram em desuso.
79
CAPÍTULO 7
NÚMEROS PRIMOS
Registros de documentos gregos
Foi supracitado que a noção de primo fora, muito provavelmente, introduzida por Pitágoras.
Com efeito, é impossível ter completa segurança nessa atribuição, pois Pitágoras não deixou
nenhum registro escrito de seus trabalhos e os documentos mais antigos que temos falando de
suas idéias resumem-se a pequenos fragmentos de textos escritos várias gerações após ele.
Entretanto, esses fragmentos, apesar de conterem informações muito escassas, são unânimes
em afirmar que Pitágoras iniciou o estudo de números primos.
O mais antigo livro de matemática que chegou completo aos nossos dias e que desenvolve
sistematicamente o estudo de números primos é Os Elementos de Euclides. Como é sabido,
Euclides seguiu muito de perto as orientações matemáticas dos pitagóricos. Assim não é
surpreendente que, no capítulo em que trata da teoria dos números, ele defina número primo
de um modo absolutamente compatível com as idéias pitagóricas expostas acima. Elementos,
Vol. VII, def 11, temos:
“protós arithmós estin monadi mone metroymenos”.
Ou seja: Número primo é todo aquele que só pode ser medido através da unidade.
Surgimento da denominação latina
A arithmetiké do grego Nikomachos, 100 dC, é o mais antigo livro de Teoria dos Números,
posterior a Elementos de Euclides, que chegou aos nossos dias. Trata-se de uma visão de
filósofo e letrado em Elementos, sendo que não há uma única demonstração entre os poucos
tópicos abordados. Apesar disso, teve grande repercussão na época e foi a base do primeiro
livro em latim que se escreveu sobre Teoria do Números: o De Institutione Arithmetica, do
romano Boethius 500 dC.
No livro de Boethius é onde aparece, pela primeira vez, a nomenclatura numerus primus
como tradução do tradicional protós arithmós preservada de Euclides por Nikomachos. Além
disto, Boethius, sempre seguindo Nikomachos, usa a velha classificação pitagórica dos
números naturais: primos incompostos versus secundários ou compostos.
O livro de Boethius foi, durante cerca de seiscentos anos, a única fonte de estudos de Teoria
dos Números disponível na Idade Média. Em torno de 1200 dC iniciou o renascimento
científico e matemático pela Europa, com o afluxo das obras árabes e a tradução das obras
gregas preservadas no Mundo Islamita. É dessa época um dos mais influentes livros de todos
os tempos: o Liber Abacci, de Fibonacci. Esse grande matemático, que havia estudado entre
os muçulmanos do Norte da África, diz que acha melhor dizer primus em vez do incomposto
preferido pelos árabes. Ficou assim, definitivamente, consagrada a denominação número
primo na Europa. (http://www.mat.ufrgs.br/~portosil/pqprimo.html)
80
CAPÍTULO 7
NÚMEROS PRIMOS
7.2. Números Primos (do lat. primus, principal. Prime em inglês)
Definição 7.1: Diz-se que um número positivo p > 1 é um número primo ou apenas um primo
se, e somente se, 1 e p são seus únicos divisores positivos. Um inteiro maior que 1 e que não é
primo diz-se composto.
Teorema 7.1: Se um número primo p não divide um inteiro a, então a e p são relativamente
primos (primos entre si).
Demonstração:
Seja d o mdc de a e p. Então d | a e d | p. Da relação d | p, resulta que d = 1 ou d = p, porque p
é primos, e como a segunda igualdade é impossível, porque p não divide a, segue-se que d =
1, isto é, o mdc ( a , p ) = 1. Logo, a e p são relativamente primos.
Corolário 7.1: Propriedade Fundamental dos Números Primos.
Se p é um primo tal que p | ab, então p | a ou p | b (podendo ser fator de ambos, a e b).
Demonstração:
Se p | a, nada há que demonstrar, e se, ao invés, p não divide a, então, pelo teorema anterior, o
mdc (p, a) = 1. logo, pelo teorema 5.4, p | b.
Nota: Observemos que esta propriedade necessária dos números primos é
também suficiente para que um inteiro positivo n seja primo: Pois, se n = k. s
é composto (1< s k < n) , temos n| k.s porém tanto n | k e n | s .
Corolário 7.2: Se p é um primo tal que p | a1a2a3 ... an, então existe um índice k, com 1
n tal que p | ak.
k
Demonstração:
Usando Indução, a proposição é verdadeira para n = 1(imediato) e para n = 2 (pelo corolário
5.1). Supondo, pois, n > 2 e que, se p divide um produto com menos de n fatores, então p
divide pelo menos um dos fatores (hipótese de indução).
Pelo corolário 7.1, se p|a1 a2 . . . an-1, então p|an ou p|a1 a2 ... an-1.
Se p|an, a proposição está demonstrada, e se, ao invés, p|a1 a2 ... an-1, então a hipótese de
indução assegura que p|ak, com 1 k n - 1. Em qualquer dos casos, p divide um dos inteiros
a1, a2, a3, ..., an.
Corolário7.3: Se os inteiros p, q1,q2 ,..., qn são todos primos e se p | q1q2 ... qn, então existe
um índice k, com 1 k n tal que p = qk.
81
CAPÍTULO 7
NÚMEROS PRIMOS
Demonstração:
De fato, pelo corolário 7.2, existe um índice k, com 1 k n , tal que p|qk, como os únicos
divisores positivos de qk são 1 e qk, porque qk, segue-se que p = 1 ou p = qk. Mas, p > 1,
porque p é primo. Logo, p = qk.
Teorema 7.2: Todo inteiro composto possui um divisor primo.
Demonstração:
Seja a um inteiro composto. Consideremos o conjunto A de todos os divisores positivos de a,
exceto os divisores 1 e a, isto é:
A = { x | a; 1 < x < a }
Pelo “Princípio da Boa Ordenação” existe o elemento mínimo p de A. que vamos mostrar ser
primo. De fato, se p fosse composto admitiria pelo menos um divisor d tal que 1 < d < p, e
então d|p e d|a, o que implica d|a, isto é, p não seria o elemento mínimo de A, se fosse
composto. Logo, p é primo.
7. 3. Teorema Fundamental da Aritmética.
Todo inteiro positivo n > 1 é igual a um produto de fatores primos.
Demonstração:
Mostraremos a existência da fatoração por indução. Se n é primo não há o que provar
(escrevemos m = 1, p1 = n). Se n é composto podemos escrever n = ab, a, b N , 1 < a < n, 1
< b < n. Por hipótese de indução, a e b se decompõem como produto de primos. Juntando as
fatorações de a e b (e reordenando os fatores) obtemos uma fatoração de n.
Nota: Este teorema (como qualquer outro teorema chamado de
"fundamental") não deveria ser aplicado sem a devida precaução. Existem
inúmeros sistemas numéricos nos quais a fatoração não é única. Por
exemplo, imagine um sistema que tenha apenas inteiros pares com a adição
e multiplicação usual e denominemos um número de "e-primo" se ele não for
o produto de dois outros números pares. Neste caso, alguns "e-primos" são
2, 6, 10, 14, 18, ... Observe que neste sistema, 36 possui duas fatorações
diferentes, 6x 6 e 2 x 18. (http://primes.utm.edu/)
Corolário 7.4: A decomposição de um inteiro positivo n > 1 como produto de fatores primos
é única, a menos da ordem dos fatores.
82
CAPÍTULO 7
NÚMEROS PRIMOS
Demonstração:
Suponha que
n = p1 ... pm = q1 ... qr
com, p1 ... pm, q1 ... qr . Como p1| q1... qr temos p1 | qi para algum valor de i, donde,
como qi é primo, p1 = qi e p1 q1 . Analogamente temos q1 p1, donde p1 = q1. Mas por
hipótese de indução
admite uma única fatoração, donde m = r e pi = qi para todo i.
Corolário 7.5: Todo inteiro positivo n > 1 admite uma única decomposição da forma
onde, para i =1,2,... , r cada ki é um inteiro positivo e cada pi é um primo, com
p1 < p2 < ... < pr,
denominada decomposição canônica do inteiro positivo n > 1.
Demonstração:
Pelo teorema Fundamental da Aritmética, n é um produto de fatores primos q1 . q2 ... qm, com
q1 q2 ... qm (m 1). Agrupando-se os fatores primos repetidos na forma de potências de
primos, temos a representação enunciada neste corolário e, pelo Teorema Fundamental da
aritmética, tal representação é única.
Nota: Conhecidas as decomposições canônicas de dois inteiros positivos a
> 1 e b > 1, o mdc (a, b) é o produto dos fatores primos comuns às duas
decomposições canônicas tomados cada um com o menor expoente, e o mmc
(a, b) é o produto dos fatores primos comuns e não comuns às duas
decomposições canônicas tomados cada um com o maior expoente.
Corolário 7.6: Se p1.p2 ... pn divide ar, então p1 . p2 ... pn divide a, onde p1 p2 ... pn é o
produto de n primos e n e r são inteiros positivos.
83
CAPÍTULO 7
NÚMEROS PRIMOS
Demonstração:
Se p1 p2 ... pn não divide a, então a não é nenhum dos primos p1p2 ... pn Seja pi, 1 i n , um
desses primos. Então, pi também não é fator primo de ar e, desta forma, não existe pi, 1 i
n, que divida ar, o que implica que p1 . p2 ... pn não divide ar. Por contraposição, temos a
demonstração pedida.
Observação: O fato de que os expoentes dos primos pi sejam 1 é essencial. Por exemplo 4 =
22 divide 62 = 36 , mas 4 não divide 6.
7.4. A Seqüência dos Números Primos
Teorema 7.4: (de Euclides): Há um número infinito de primos.
Demonstração:
Suponha por absurdo que p1, p2, ..., pm fossem todos os primos. O número P = p1 . p2 ... pm +
1 > 1 não seria divisível por nenhum primo, o que contradiz o teorema fundamental da
aritmética.
Proposição 7.1. Para o n-ésimo número primo pn vale a estimativa
.
Demostração:
Para n = 1 é verdade que
desigualdades
. Suponhamos já provadas as
Se q é primo tal que q | p1 . p2 ... pn + 1, então q > pn, particularmente q
pn+1.
Então,
Esta estimativa é exageradamente fraca, no geral pn é significativamente menor que
por
exemplo
= 256 e p4 é apenas 7. Uma estimativa melhor para pn, postulada por Bertrand
e, demonstrada por Chebychev, é dada pelo teorema seguinte:
84
CAPÍTULO 7
NÚMEROS PRIMOS
Teorema 7.5 (de Chebychev): Para todo inteiro m 2 existe um primo p com m < p< 2m. A
demonstração deste teorema está fora de nosso contexto. Um outro fato provado é que entre
dois cubos consecutivos existe sempre um primo.
Com esse Teorema podemos afirmar que pn+1 < 2pn (para n
1)
Corolário 7.6. Para o n-ésimo número primo pn vale a estimativa pn
2n.
Demonstração:
Temos 2 = p1 21 e pelo teorema de Chebychev: Para todo inteiro positivo n, tem-se pn < pn+1
< 2 . pn. De pn 2n segue que pn + 1 2. 2n = 2n+1.
Ao estudarmos a sequência de números primos percebemos que existem infinitos primos em
subconjuntos particulares dos inteiros, como, por exemplo, na sucessão aritmética: {4q + 3; q
inteiro e q 0} = {3, 7, 11, 15, 19, ...}.
Esse resultado foi generalizado pelo matemático alemão Peter Gustav Le-jeune Dirichlet
(1805-1859).
Teorema 7.6 (de Dirichlet). Sejam a e b inteiros primos entre si, isto é, mdc (a, b) = 1.
Existem infinitos primos da forma an + b, onde n é inteiro positivo.
A demonstração deste Teorema exige avançados conhecimentos de Análise Matemática.
Exemplos:
Na sequência 3n + 1, temos os primos 7, 13, 19, 31, 37, 43, 61, 67, 73, 97, 103, ...
Na sequência 9n + 4 temos os primos 13, 31, 67, 103, 139, 157, 193, 211, ...
O resultado de Dirichlet diz não só que o número de primos é infinito, mas também que, se
considerarmos subconjuntos particulares de inteiros, como as sucessões aritméticas acima,
teremos já nesses subconjuntos uma infinidade de primos.
Uma aplicação do Teorema de Dirichlet leva-nos a um resultado obtido pelo matemático
polonês W. Sierpinski , que nos mostra, mais uma vez, a forma surpreendente como os primos
se distribuem nos inteiros.
Teorema 7.7 (de Sierpinski). Dado um inteiro m maior que 1, existe um primo p tal que |p
1|, |p 2|, ..., |p m| são compostos.
Exemplo: Seja m = 10 e p = 19. Temos:
19+1, 19+2, 19+3, 19-4, 19+5, 19+6, 19+7, 19+8, 19+9 e 19-10. Os resultados são todos
números compostos: 20, 21, 22, 15, 24, 25, 26, 27, 28 e 9.
Observe que se tivéssemos escolhido o primo 17, não seria possível construir uma sequência
de compostos com m = 10, pois 17 + 6 = 23 e 17 – 6 = 11, ambos primos.
Demonstração:
Vejamos, em primeiro lugar, que existe um primo p tal que p + 1, p + 2, ..., p + m sejam
compostos. Para cada m dado, o Teorema 1 garante, em particular, que existe um inteiro
primo q maior do que m. Seja a = (q + 1) • (q + 2) • (q + 3) ... (q + m).
Se q divide a, então q divide q + i, e, portanto, q divide i, o que é impossível para 0 < i m <
q. Então a e q são primos entre si. Pelo teorema de Dirichlet, existe um primo p na sequência
85
CAPÍTULO 7
NÚMEROS PRIMOS
an + q. Seja p = (q +1) • (q + 2) • ... • (q + m) • n + q este primo. Então os números p + 1, p +
2, ..., p + m são m números compostos. Para ampliar este resultado, observemos que, por
motivos análogos aos de cima,
a' = (q - m).[q - (m- 1)]...(q - 1)...(q + m)
é primo com q e se p' for um primo da sequência a'n + q, isto é,
p' = (q - m) ... (q - 1) . (q + 1) ... (q + m) . n + q
os números p' – m, ... p' – 1, p' + 1, ... p' + m serão compostos. Assim o número primo p' se
encontra na sucessão dos inteiros, "isolado" por m compostos de cada lado. (RPM 11)
7.5. O Crivo de Eratóstenes.
Eratóstenes, matemático, astrônomo, historiador, geógrafo e filósofo grego, nasceu em Cirene
por volta de 276 a.C. e passou grande parte de sua juventude em Atenas. Com
aproximadamente 40 anos, foi convidado pelo rei Ptolomeu III do Egito para ser bibliotecário
da Universidade de Alexandria.
Ficou conhecido como Beta, e a respeito dessa alcunha existem algumas hipóteses. Alguns
acreditam que, por causa de seu saber, foi elevado à condição de um segundo Platão. Outros,
dizem que tal apelido lhe fora dado por ter sido o segundo bibliotecário da Universidade de
Alexandria. Uma terceira explicação sugere que, apesar de ser talentoso, Eratóstenes não
conseguiu ser o primeiro de seu tempo em nenhum ramo de estudo, em outras palavras, foi
sempre o segundo. Por fim, o historiador James Gow sugeriu que talvez Beta indicasse
simplesmente o número (grego) 2 referente a um gabinete ou a uma sala de leitura da
universidade.
Escreveu diversas obras, mas muitas se perderam, inclusive o tratado Sobre a medida da
Terra. Eratóstenes morreu em Alexandria, em 194 a.C.
(http://www.moderna.com.br/moderna/didaticos/ef2/matematica/erato/bio_eratostenes.htm)
Teorema 7.8: Se um inteiro positivo a > 1 é composto, então a possui um divisor primo
p
.
Demonstração:
Com efeito, se o inteiro positivo a > 1 é composto, então:
a = bc, com 1 < b < a e 1 < c < a
Portanto, supondo b
c, teremos:
b2
86
bc = a
b
CAPÍTULO 7
NÚMEROS PRIMOS
O teorema 7.8 fornece um processo que permite reconhecer se um dado inteiro a >1 é primo
ou é composto, para o que basta dividir a sucessivamente pelos primos que não excedem o
valor
. Tal resultado é a base do chamado Crivo de Eratóstenes que veremos em seguida.
Uma questão natural sobre os números primos é a de determinar, dentre os inteiros positivos,
todos os números primos até certo número dado. Esta questão também foi resolvida na
antiguidade por Eratóstenes. A ele devemos o chamado Crivo de Eratóstenes. Com o crivo de
Eratóstenes podem-se determinar, sem auxílio de máquinas, todos os números primos até 200,
400 ou 500, por exemplo. Com o auxílio de computadores, o crivo de Eratóstenes,
convenientemente adaptado, permite determinar os números primos até limites bem altos.
Mesmo antes dos computadores, já haviam sido determinados os números primos até
10.000.000. Isto ocorreu por volta de 1914, por obra do matemático americano D. N. Lehmer.
Dois outros matemáticos (Bays e Hudson) calcularam, em 1976, (usando computadores,
evidentemente!), a tabela dos números primos até 12 x 1011. Além disso, há tabelas de
números primos em determinados intervalos de inteiros e conhecem-se também números
primos bem grandes, como o número 244497 – 1, que possui 13395 algarismos! (RPM 19)
A construção de uma tabela de números primos que não excedam um dado inteiro n usando o
Crivo de Eratóstenes consiste no seguinte: escrevem-se na ordem natural todos os inteiros a
partir de 2 até n e, em seguida, eliminam-se todos os inteiros compostos que são múltiplos dos
primos p tais que p
isto é, 2p, 3p, 4p, ...
Exemplo: Construir a tabela de números primos menores que 200.
Solução: Como
, basta eliminar sucessivamente da tabela os números que são
múltiplos dos primos p menores que 14, ou seja, 2, 3, 5, 7, 11 e 13.
.
2
3
4
5
6
7
8
9
10
11
12
13
14
15
16
17
18
19
20
21
22
23
24
25
26
27
28
29
30
31
32
33
34
35
36
37
38
39
40
41
42
43
44
45
46
47
48
49
50
51
52
53
54
55
56
57
58
59
60
61
62
63
64
65
66
67
68
69
70
71
72
73
74
75
76
77
78
79
80
81
82
83
84
85
86
87
88
89
90
91
92
93
94
95
96
97
98
99
100
101
102
103
104
105
106
107
108
109
110
111
112
113
114
115
116
117
118
119
120
121
122
123
124
125
126
127
128
129
130
131
132
133
134
135
136
137
138
139
140
87
CAPÍTULO 7
NÚMEROS PRIMOS
141
142
143
144
145
146
147
148
149
150
151
152
153
154
155
156
157
158
159
160
161
162
163
164
165
166
167
168
169
170
171
172
173
174
175
176
177
178
179
180
181
182
183
184
185
186
187
188
189
190
191
192
193
194
195
196
197
198
199
200
Os valores em vermelho são os números primos que não foram “riscados” da tabela.
Listamos a seguir a os 199 primeiros números primos:
2, 3, 5, 7, 11, 13, 17, 19, 23, 29, 31, 37, 41, 43, 47, 53, 59, 61, 67, 71, 73, 79, 83, 89, 97, 101,
103, 107, 109, 113, 127, 131, 137, 139, 149, 151, 157, 163, 167, 173, 179, 181, 191, 193, 197,
199, 211, 223, 227, 229, 233, 239, 241, 251, 257, 263, 269, 271, 277, 281, 283, 293, 307, 311,
313, 317, 331, 337, 347, 349, 353, 359, 367, 373, 379, 383, 389, 397, 401, 409, 419, 421, 431,
433, 439, 443, 449, 457, 461, 463, 467, 479, 487, 491, 499, 503, 509, 521, 523, 541, 547, 557,
563, 569, 571, 577, 587, 593, 599, 601, 607, 613, 617, 619, 631, 641, 643, 647, 653, 659, 661,
673, 677, 683, 691, 701, 709, 719, 727, 733, 739, 743, 751, 757, 761, 769, 773, 787, 797, 809,
811, 821, 823, 827, 829, 839, 853, 857, 859, 863, 877, 881, 883, 887, 907, 911, 919, 929, 937,
941, 947, 953, 967, 971, 977, 983, 991, 997, 1009, 1013, 1019, 1021, 1031, 1033, 1039, 1049,
1051, 1061, 1063, 1069, 1087, 1091, 1093, 1097, 1103, 1109, 1117, 1123, 1129, 1151, 1153,
1163, 1171, 1181, 1187, 1193, 1201, 1213 ,1217.
Nota: Podemos fazer um crivo mais econômico, já que não é possível evitar
completamente o fato de que alguns números são riscados várias vezes.
Podemos proceder da seguinte maneira: Primeiro escrevemos uma lista com
os ímpares de 3 até n. Como queremos riscar os números de p em p é claro
que os múltiplos de p que são múltiplos de primos menores que p já foram
riscados da lista. Então, nesta etapa, podemos começar a riscar de p em p a
partir do menor múltiplo de p, que não é múltiplo de um primo menor que p;
isto é, a partir de p2. Isto evita muitas duplicações.[Coutinho]
3, 5, 7, 32, 11, 13, 15, 17, 19, 21 23, 52, 27, 29, 31, 33, 35, 37, 39, 41,
43, 45, 47, 72, 51, 53, 55, 57, 59, 61, 63, 65, 67, 69, 71, 73, 75, 77, 79,
81, 83, 85, 87, 89, 91, 93, 95, 97, 99.
88
CAPÍTULO 7
NÚMEROS PRIMOS
Definição 7.2: Para qualquer número real x > 0, seja (x) o número de primos p
(x) é quantidade dos números primos menores que ou iguais a x.
x , isto é,
Tabela dos 20 primeiros valores inteiros da Função p (x)
x
1
2
3
4
5
6
7
8
9
10
11
12
13
14
15
16
17
18
19
20
(x)
0
1
2
2
3
3
4
4
4
4
5
5
6
6
6
6
7
7
8
8
De acordo com o Teorema de Chebychev podemos afirmar que
(2n) -
(n)
1 (para n
2)
Nota: Um problema prático, onde as propriedades dos números primos têm
reflexos importantes, é o problema do reconhecimento da fala por
computadores que exige o desenvolvimento de algoritmos tão rápidos quanto
possível para a decomposição de sons nas suas frequências fundamentais,
uma técnica conhecida como Análise de Fourier. A velocidade teórica
máxima desses algoritmos esta diretamente relacionada com a função (x)
que fornece o numero de primos menores que ou iguais a x.
Fórmula de Minác: Dado um inteiro m
contagem dos números primos (m) :
2 , J. Minác estabeleceu uma fórmula para a
Demonstração: Será vista após estudarmos o Teorema De Wilson.
89
CAPÍTULO 7
NÚMEROS PRIMOS
Exemplo:
(6) = 1 + 1 + 0 + 1 + 0 = 3. Resultado que nos diz que existem três primos antes do número
seis.
Fórmula Para o n-ésimo Número Primo
Devido ao resultado acima podemos escrever uma fórmula que nos retorna o n-ésimo número
primo estabelecida por C. P. Willans em 1964:
Exemplo:
Definição 7.3: Para todo número primo p, seja p# o produto de todos os números primos q
p . p# é chamado o primorial de p.
Tabela dos 17 primeiros Primoriais
P
2
3
5
7
11
13
17
23
29
90
p#
2
6
30
210
2310
30030
510510
9699690
223092870
CAPÍTULO 7
NÚMEROS PRIMOS
31
37
41
43
47
53
59
61
6469693230
200560490130
7420738134810
304250263527210
13082761331670030
614889782588491410
32589158477190044730
1922760350154212639070
Teorema 7.8: p# +1 não possui nenhum fator primo menor do que ou igual a p.
Demonstração: Suponhamos, por contradição, que p# + 1 seja divisível por um primo q p .
Ou seja, existe um inteiro positivo s tal que tal que p# + 1 = q.s, isto é q.s – p# =1. Como q
p , então q é necessariamente um fator de p#. Logo q divide ambas as parcelas da diferença
q.s – p#. Portanto q divide 1, o que é um absurdo uma vez que q é primo.
Nota: Veja que resultado interessante:
Leitura: A Distribuição dos Números Primos
Ao contemplar uma tabela de números primos, a primeira impressão que se tem é a de que
não há nenhuma ordem entre os números primos: às vezes eles aparecem próximos uns dos
outros, às vezes afastados, ora menos, ora mais afastados; enfim, analisando-os
individualmente ou em pequenos grupos, não divisamos qualquer regularidade em sua
distribuição. Entretanto, a sagacidade de inteligências privilegiadas consegue ver mais fundo,
e foi precisamente isso o que aconteceu por obra do matemático francês Adrien - Marie
Legendre (1752-1833). Ele se ocupou dessa questão e por volta de 1800 formulou uma
conjectura que revela certa ordem no que parecia ser um caos completo. Para explicarmos a
conjectura de Legendre, introduzimos o símbolo (x) como sendo o número de números
primos até certo valor x. Assim, (8) = 4, ou seja, o número de números primos até 8 é 4;
(11) = 5, pois há cinco números primos até 11, precisamente, 2, 3, 5, 7, 11; e assim por
diante. Pois bem, o que Legendre conjecturou, empiricamente, analisando tabelas de números
primos (em 1797 uma dessas tabelas foi publicada, contendo todos os números primos até
400031), é que (x) podia ser aproximado pela função
(o logaritmo que aqui aparece é o
logaritmo natural, isto é, na base e 2,718281...), e que essa aproximação seria tanto melhor
quanto maior fosse x. Mas isto deve ser entendido em termos relativos, isto é, o erro que se
comete tomando
em lugar de (x) torna-se tanto menor quanto maior for x, relativamente
a
Em outras palavras, seja
91
CAPÍTULO 7
NÚMEROS PRIMOS
o erro que se comete ao tomar
crescer de x é o erro relativo
em lugar. de (x). Pois bem, o que se torna pequeno com o
(2)
Este erro pode ser feito, em valor absoluto, tão pequeno quanto quisermos, desde que façamos
x suficientemente grande.
Carl Friedrich Gauss (1777-1855), que é considerado por muitos o maior matemático de todos
os tempos, conta, numa carta de 1849, publicada vários anos mais tarde, que quando ainda
bem jovem, com apenas 15 anos de idade, pensou muito sobre a distribuição dos números
primos, chegando a conjecturar algo equivalente ao que conjecturou Legendre.
Seja como for, essa conjectura logo impressionou os matemáticos como algo notável, pois
quem diria que a seqüência dos números primos pudesse ter algo a ver com a função
logaritmo!
A descoberta de Legendre e Gauss demorou a ser demonstrada. Embora ela tenha sido objeto
da atenção dos melhores matemáticos do século, desafiou a argúcia desses homens por cerca
de 100 anos. De fato, foi somente em 1896 que ela foi demonstrada pela primeira vez. E nesse
mesmo ano apareceram duas demonstrações, uma pelo matemático francês Jacques Hadamard
(1865-1963) e outra, pelo belga Charles de Ia Vallée Poussin (1866-1962). Essas
demonstrações, independentes uma da outra, baseavam-se nas idéias de um outro grande
matemático do século, Bernhard Riemann (1826-1866). Embora não tenha logrado
demonstrar a conjectura de Legendre e Gauss, Riemann, num memorável trabalho intitulado
Sobre o número de números primos menores que um certo número, deixou ideias notáveis
sobre teoria dos números, que vêm sendo exploradas pelos estudiosos do assunto até os dias
de hoje.
Antes mesmo das demonstrações de Hadamard e de la Vallée Poussin, o matemático russo
Pafnutii Chebyshev (1821-1894) provou, por volta de 1850, um resultado próximo à
conjectura de Legendre e Gauss. Segundo Chebyshev, existem constantes positivas c e C (c
0,92, C 1, 106) tais que
(3)
Para bem entendermos o significado da aproximação
(4)
vamos comparar os gráficos das funções y = x e y = log x. Eles nos revelam que ambas as
funções crescem com o crescer de x, tendendo a infinito.
92
CAPÍTULO 7
NÚMEROS PRIMOS
No entanto, como podemos ver, claramente, a primeira dessas funções cresce mais depressa
que a segunda, distanciando-se mais e mais desta última, à medida que x cresce acima de
qualquer número dado. Isto fica mais claro ainda quando levamos em conta que o gráfico do
logaritmo tem a concavidade voltada para baixo, significando que, embora esta função esteja
crescendo sempre com o crescer de x, trata-se de um crescimento cada vez mais lento, quanto
maior for x. Isto quer dizer que o quociente no segundo membro de (4) também cresce,
tendendo a infinito com o crescer de x, o que está de acordo com o fato de que existem
infinitos números primos, isto é, (x) cresce acima de qualquer número, desde que façamos x
suficientemente grande. Não obstante tudo isso, o erro absoluto expresso em (1) pode tornarse muito grande, mas não o erro relativo expresso em (2); este tende a zero, isto é, pode ser
feito menor do que qualquer número positivo dado, desde que façamos x suficientemente
grande.
Uma conclusão simples que podemos tirar de (4) é que, em certo sentido, os números primos
vão ficando cada vez mais raros, à medida que avançamos na seqüência dos números naturais.
Para bem entender o que estamos dizendo, observe que
significa
de sorte que
é a densidade média dos números primos no intervalo que vai de 1 até x. O
fato de que essa densidade decresce com o crescer de x significa precisamente o que dissemos
acima: os números primos vão ficando cada vez mais raros, à medida que avançamos na
seqüência dos números naturais. ( RPM19)
Definição 7.3 Chamam-se primos gêmeos dois inteiros positivos ímpares e consecutivos que
são ambos primos. Em outras palavras, dizemos que dois primos ímpares são gêmos quando a
diferença entre eles é igual a 2.
Assim, por exemplo, são pares de primos gêmeos:
3 e 5, 5 e 7, 11 e 13, 17 e 19, 29 e 31
Não se sabe até hoje se há um número infinito de pares de primos gêmeos, mas são
conhecidos primos gêmeos muito grandes, tais como:
140.737.488.353.507 e 140.737.488.353.509
140.737.488.353.699 e 140.737.488.353.701
Um fato interessante é a existência de apenas um terno de inteiros positivos ímpares e
consecutivos que são todos primos: 3, 5 e 7.
93
CAPÍTULO 7
NÚMEROS PRIMOS
7.6. Seqüência de Inteiros Consecutivos Compostos
Existem, na sequência dos primos, primos consecutivos “tão afastados quanto se deseje”. Ou
seja, existem “saltos’ arbitrariamente grandes na seqüência dos primos.
Teorema 7.9: Dado um inteiro positivo n >1, é possível determinar n inteiros consecutivos
tais que nenhum deles seja primo.
Demonstração:
De fato, é evidente que na sequência:
(n + 1)! + 2, (n + 1)! +3, (n + 1)! + 4, ..., (n + 1)! + (n + 1)
os seus n termos são inteiros positivos consecutivos, e cada um deles é composto, porque (n
+1)! + j é divisível por j se 2 j n + 1.
Assim, por exemplo, supondo n = 4, obtemos a sequência:
5! + 2, 5! + 3, 5! + 4, 5! + 5
Cujos termos são 4 inteiros positivos consecutivos, cada um dos quais é composto, pois,
temos:
5! + 2 = 122 = 2 . 61, 5! + 3 = 123 = 3 . 41
5! + 4 = 124 = 4 . 31, 5! + 5 = 125 = 5 . 25
Outras sequências de 4 inteiros consecutivos e compostos existem, tais como
24, 25, 26, 27 e 32, 33, 34, 35
54, 55, 56, 57 e 74, 75, 76, 77
Nota: Em 1984 Samuel Yates iniciou uma lista dos "Maiores Primos
Conhecidos" e criou o nome primo titânico para designar qualquer número
primo com 1.000 ou mais dígitos decimais. Denominou também de titãs
aqueles que provaram a sua primalidade.
A maioria dos primos são titânicos e dezenas de milhares deles são
"conhecidos". Entretanto, na época em que Yates definiu os primos titânicos,
tinha-se conhecimento de apenas alguns poucos.
Cerca de dez anos mais tarde, Yates designou como primo gigante todo
número primo que possuísse 10.000 ou mais dígitos decimais. E os
Megaprimos são números primos que possuam no mínimo um milhão de
dígitos decimais.
http://www.numaboa.com.br/criptologia/matematica/primos.php
94
CAPÍTULO 7
NÚMEROS PRIMOS
Corolário 7.7: Dado um inteiro positivo n, existem dois primos consecutivos ph, ph+1 tais que
ph+1 – ph > n.
Demonstração:
Seja ph o maior dos primos que são menores que ( n +1 )! + 2.
Então, ph
(n + 1)!+ 1. Do teorema anterior, temos ainda que
ph +1 > (n + 1)! + (n + 1)
Fazendo a diferença entre ambas as desigualdades, temos
ph +1 > (n + 1)! + (n + 1)
Exemplo: Seja n = 6, de acordo com a demonstração podemos considerar os primos p1 =
5039 e p2 = 5059. Assim, 5059 – 5039 > 6, isto é, 20 > 6.
Teorema 7.10: O produto de qualquer sequência de k inteiros consecutivos é divisível por k!.
Demonstração:
Vamos considerar n e k inteiros positivos com k
de n, tomadas k a k, é um inteiro dado por:
n. Sabemos que o número de combinações
Sendo o numerador o produto de k inteiros consecutivos temos o resultado para uma
sequência de k inteiros positivos. No caso de zero ser um elemento na seqüência o resultado é
trivial, uma vez que zero é divisível por qualquer inteiro não nulo.
Se a sequência contiver só números negativos, a fração do lado direito da igualdade acima
sofrerá, no máximo, uma mudança de sinal continuando a ser um inteiro, o que conclui a
demonstração.
95
CAPÍTULO 7
NÚMEROS PRIMOS
7.7 . Conjecturas
● Conjectura de Goldbach.
Em 1742, numa carta a Leonhard Euler (1707-1783), Christian Goldbach (1690-1764)
expressou a seguinte conjectura:
Todo inteiro n > 5 é a soma de três números primos.
Em resposta, Leonhard Euler observou que essa conjectura era equivalente à seguinte:
Todo inteiro par maior que ou igual a 4 é a soma de dois primos.
Esta conjectura é conhecida como conjectura de Goldbach. Um romance interessantíssimo
sobre a dificuldade desse assunto é “Tio Petros e a Conjectura de Goldbach” escrito por
Apostolos Doxiadis e publicado pela Editora 34.
Exemplos:
4=2+2
6=3+3
8=3+5
10=3+7, 5+5
12=5+7
14=3+11, 7+7
16=3+13, 5+11
18=5+13, 7+11
20=3+17, 7+13
22=3+19, 5+17, 11+11
24=5+19, 7+17, 11+13
26=3+23, 7+19, 13+13
28=5+23, 11+17
30=7+23, 11+19, 13+17
32=3+29, 13+19
34=3+31, 5+29, 11+23, 17+17
36=5+31, 7+29, 13+23, 17+19
38=7+31, 19+19
40=3+37, 11+29, 17+23
Muitos matemáticos continuam tentando encontrar um contra-exemplo ou uma demonstração
para essa conjectura. Por exemplo:
96
•
Georg Cantor (1845-1918), efetuou em 1894 todas as decomposições possíveis, como
soma de dois números primos, de todos os números pares inferiores a 1000.
•
Aubry estendeu a lista de Cantor até 2000.
CAPÍTULO 7
NÚMEROS PRIMOS
•
R. Haussner em 1897 estendeu essa tabela até 5000.
•
Em 1937 o matemático soviético I.M.Vinogradov demonstrou, usando somas
trigonométricas adequadas, que qualquer número ímpar suficientemente grande é soma de
três números primos.
•
Em 1966 o matemático chinês Jeng-Run Chen provou que a partir de algum número n,
todo par maior que 2 ou é soma de dois primos, ou a soma de um primo com o produto de
dois primos. O argumento de Chen não diz qual é esse n; apenas demonstra que ele existe.
Além da Conjectura de Goldbach, em Teoria dos Números, particularmente em Números
Primos, existem muitos problemas em aberto. Segue uma lista com algumas conjecturas que,
embora já tenham sido testadas para inúmeros casos, ainda não foram demonstradas. Eis
algumas:
•
Todo número ímpar maior que cinco é a soma de três primos. Esse fato já foi provado,
por Vinogradov, para números suficientemente grandes. Em 1956, Borodzkin mostrou que
n > 314348907 é suficiente. Esse número foi diminuído, em 1989, para 1043000, por Chen e
Wang, mas ainda é muito grande para que os casos menores possam ser testados com o
uso de um computador.
Exemplos:
7 = 3 + 2 + 2; 21 = 11 +7 + 3 ; 41 = 11 + 13 + 17; 49 = 13 + 17 + 19
•
Existem infinitos primos da forma k2 + 1.
Exemplos:
5 = 22 +1; 17 = 42 +1; 37= 62 + 1.
•
Existem infinitos pares de primos consecutivos (Primos Gêmeos) .
Exemplos: (3 e 5), (5 e 7), (11 e 13), (17 e 19), (29.879 e 29.881), ...
Em 2000, foi apresentado um par de primos gêmeos cada um com 18075 dígitos. É o par
4 648 619 711 505. 260000 ± 1
•
Existe sempre um primo entre dois quadrados consecutivos.
Exemplos: 3 entre 1 e 4; 5 e 7 entre 4 e 9; 11 e 13 entre 9 e 16, ....
•
Primos de Sophie Germain. Um número primo p é um número primo de Sophie
Germain se 2p + 1 é também primo. São famosos porque Sophie Germain provou que o
Último Teorema de Fermat é verdadeiro para estes números. A existência de um número
infinito de tais números primos é uma uma afirmação ainda não provada. Os primeiros
primos de Sophie Germain são 2, 3, 5, 11, 23, 29, 41, 53, 83, 89, 113, 131, 173, 179, 191,
233 ...
97
CAPÍTULO 7
NÚMEROS PRIMOS
Nota: Primos em Progressão Aritmética.
Um problema famoso que permaneceu por muito tempo em aberto, era o de
provar se existiam progressões aritméticas arbitrariamente longas formadas
exclusivamente por primos. Van der Corput já havia provado em 1939 que há
uma infinidade de progressões aritméticas formadas por 3 primos. Ben Green
do Instituto de Matemática de Vancouver e Terence Tao da Universidade da
Califórnia, provaram em 2006, que tais sequências existem. Mas a prova não
especifica como encontrá-las ou entre quais primos tais sequências se
encontram.
A mais longa progressão aritmética de números primos conhecida até o
momento, tem 24 termos. Foi descoberta por Jaroslaw Wroblewski em
janeiro de 2007:
468395662504823 + 45872132836530.k, para k = 0, 1, ..., 23.
7.8. Fórmulas que geram alguns números primos
Muitas tentativas têm sido realizadas para encontrar fórmulas aritméticas simples que
forneçam somente primos. Nesta seção será apresentada algumas fórmulas famosas sobre
primos.
1) Fórmula de Fermat:
Fermat fez sua famosa conjectura de que os números da forma
são primos.
Para n = 1, 2, 3, 4 obtemos:
F1 = 22 + 1 = 5
F2 =
+ 1 = 24 + 1 = 17
F3 =
+ 1 = 28 + 1 = 257
F4 =
+ 1 = 216 + 1 = 65.537
todos primos. Porém em 1732, Euler descobriu a fatoração
+ 1 = 4294967297 = (641).(6700417)
portanto, F(5) não é primo. Até este momento (05 /2005) o maior primo de Fermat
conhecido é F4
98
CAPÍTULO 7
NÚMEROS PRIMOS
2) Fórmula de Euler:
Em 1772 Leonhard Euler descobriu um polinômio tendo uma longa sucessão de valores
primos, dado por
F(n) = n2 + n + 41
que fornece primos para n = 1, 2, ..., 39. Entretanto, para n = 40 o valor é composto:
F(40) = 402 + 40 + 41 = 40. (40 + 1) + 41 = 40.41 + 41 = 41.(40 + 1) = 41.41.
3) Fórmula de Mersenne:
Marin Mersenne em 1644 fez a seguinte afirmação: “Todo natural Mp = 2p – 1 é primo
para os primos p = 2, 3, 5, 7, 13, 17, 19, 31, 67, 127 e 257, e é composto para todos os
outros primos p < 257”.
Entretanto, esta afirmação é incorreta, pois, segundo o site http://www.mersenne.org/
prime.htm, até setembro de 2006 já eram conhecidos, 44 primos de Mersene, para os primos p
= 2, 3, 5, 7, 13, 17, 19, 31, 61, 89, 107, 127, 521, 607, 1279, 2203, 2281, 3217, 4253, 4423,
9689, 9941, 1213 ,19937, 21701, 23209, 44497, 86243, 110503, 132049, 216091, 756839,
859433, 1257787, 1398269, 2976221, 3021377, 6972593, 13466917, 20996011, 24036583,
30402457 e 32582657. Esse último
primo tem 9.808.358 dígitos.
Como se pode ver, Mersenne cometeu duas falhas: Incluiu p= 67, 257 na sua lista de primos e
excluiu dessa lista p= 61, 89, 107.
Somente em 1947 ( mais de 300 anos depois) a lista correta p = 2, 3, 5, 7, 13, 17, 19, 31, 61,
89, 107 e 127 onde p < 257, ficou pronta..
4) Outras fórmulas que geram alguns primos são:
F(n) = n2 - n + 41para n = 1, 2, 3, 4, ..., 40
F(n) = n2 - 79n + 1601para n = 0, 1, 2, ..., 79
F(n) = n2 + n + 17 para n = 0, 1, 2, ..., 15
F(n) = 3n2 + 3n + 23para n = 0, 1, 2, ..., 21
F(n) = 6n2 + 6n + 31para n = 0, 1, 2, ..., 28
Cabe agora a pergunta: Existe algum polinômio (não-constante), com coeficientes inteiros,
que forneça a sequência dos números primos ou apenas números primos? Infelizmente a
resposta é não!
Teorema 7.11: Não existe polinômio algum P(x) = anxn + an-1xx-1+ ... + a0, a0 0 com
coeficientes ak, 0 k n , todos inteiros, cujos valores numéricos sejam sempre primos para
valores inteiros da variável x. ( VER RPM 45)
99
CAPÍTULO 7
NÚMEROS PRIMOS
Demonstração: Suponhamos, por contradição, que o polinômio P(x), nas condições do
teorema, produz sempre primos para valores inteiros da variável x. Então, para x = j, sendo j
um inteiro fixo, P( j ) = p é um primo, e qualquer que seja o inteiro s, temos:
P(j + ps) = an (j + ps)n + na-1 (j + ps)n-1 + ... + a2 (j +ps)2 +a1 (j + ps) + a0
Desenvolvendo cada uma das potências pela fórmula do binômio e agrupando os primeiros
termos de cada desenvolvimento, temos:
P(j + ps) = (an jn + an-1 jn-1 + ... + a2j2 +a1j + a0) + pg(s) = P(j) + pg(s) = p + pg(s)
onde g(s) indica um certo polinômio não constante em s com coeficientes inteiros, de grau
n, logo:
P(j + ps) = p(1 + g(s))
Então, p| P(j + ps). Se P(j + ps) é primo devemos ter P( j + ps) = ± p, donde 1+ g(s) = ± 1,
para todo s. Temos uma contradição, pois g(s) não é constante.
Nota: O teorema anterior refere-se a polinômios numa variável. Os trabalho
de Putnam, Davis, Robison e Matijasevic conduziram a uma surpreendente
conclusão: Existe um polinômio de coeficientes inteiros, tal que o conjunto
dos números primos coincide com o conjunto dos valores positivos
assumidos por esse polinômio, quando as variáveis percorrem o conjunto
dos inteiros positivos.
Jones, Sato, Wada e Wiens (1976) foram os primeiros a escrever,
explicitamente, um polinômio desse tipo, de grau 25 e com 26 varáveis. [
Ribombim ]
Leitura: Uma Fórmula que Fornece todos os Números Primos
Sejam x e y números naturais, y
0 e a = x( y + 1) - ( y!+ 1).
A fórmula que dá todos os números primos e somente esses é:
f(x, y) =
Por exemplo:
Se x = 1 e y = 1, então a = 0 e f(1,1) = 2;
Se x = 1 e y = 2, então a = 0 e f(1,2) = 3;
Se x = 1 e y = 3. então a = –3 e f(1,3) = 2;
e, atribuindo-se a x e a y mais alguns valores, percebe-se logo que a função f tem uma
predileção muito grande pelo número primo 2. Mas ela fornece todos os números primos:
100
CAPÍTULO 7
NÚMEROS PRIMOS
5 = f (5,4); 7 = f (103,6); 11= f (329891,10); 13 = f (36846377, 12); ....
Como foram achados os pares (x,y) acima? A resposta é simples: para obter o número primo
p, calcule f(x,y) para
Assim, para obter 13, fizemos
Como se vê, a fórmula existe, mas não é nada prática, uma vez que envolve cálculos com
números muito grandes(RPM 37).
A demonstração dessa fórmula será vista após estudarmos o Teorema de Wilson.
7.9. Decomposição do Fatorial em Fatores Primos
Mostraremos como achar a fatoração em números primos de n! onde n é um número natural
arbitrário.
Proposição 7.2: Sejam a
0 e b, c > 0 . Temos que
Demonstração: Sejam.
Logo,
a = bq1 + r1 , com r1
b–1
e
portanto,
a = bq1 + r1 = b(cq2 + r2) + r1 = bcq2 + br2 + r1
como
br2 + r1
b(c - 1) + b - 1 = bc - 1
101
CAPÍTULO 7
NÚMEROS PRIMOS
segue-se que é o quociente da divisão de a por bc, ou seja,
q2 =
Dados um número primo p e um número natural m, vamos definir por Ep (m) o expoente da
maior potência de p que divide m, ou seja, é o expoente da potência de p que aparece na
fatoração de m em fatores primos.
Em particular, Ep (n!) representará a potência de p que aparece na fatoração de n! em fatores
primos.
Teorema de Legendre. Sejam m um número natural e p um número primo. Então
Ep (n!) =
Demonstração: Note, inicialmente, que a soma acima é finita, pois existe um número natural
r tal que pi > n para todo i > r portanto
, se i r
Vamos demonstrar o resultado por indução sobre n . A fórmula vale trivialmente para n = 0.
Suponha que o resultado vale para qualquer natural m com m < n Sabemos que os múltiplos
de p entre 1 e n são:
p, 2 p, ...,
Portanto, pela hipótese de indução, temos que
O resultado, agora, decorre da preposição 7.2.
Para calcular Ep (n!) faz-se uso do seguinte algoritmo:
n = pq1 + r1
q1 = pq2 + r2
.....
qs-1 = pqs + rs
Como q1 > q2 > ..., seguem-se que, para alguns s, tem-se que. Portanto, seguem-se que.
E (n!) = q1 + q2 + ... + qs
102
CAPÍTULO 7
NÚMEROS PRIMOS
Exemplo: Vamos determinar a decomposição de 10! Em fatores primos.
Para resolvermos o problema, devemos achar Ep (10!) para todo primo p
10. Sendo
Seguem-se que
10! = 2834527 .
Lema 7.1. Sejam a1,..., am ,b inteiros positivos. Tem–se que
Demonstração: Sejam qi e ri respectivamente o quociente e o resto da divisão de ai por b para
i = 1, ...., m. somando, membro a membro, as igualdade ai = bqi + ri temos que
a1 + ... + am = (q1 + ... + qm) b + r1 + ... + rm
Segue–se dai que o quociente da divisão de a1 + ... + am por b é maior ou igual do que q1 + ...
+ qm pois r1 + ... + rm poderia superar b – 1 . Isto é o que se queria provar.
Corolário 7.8. Se a1,..., am,b são números naturais com b
0 , então é natural o número
Demonstração: De fato, pelo Lema 7.1, para todo número primo P e todo número natural
i, temos que
Somando, membro a membro, as desigualdades acima, obtemos que
Ep ((a1 + ... + am)!)
Ep (a1 !) + ... + Ep (am)
103
CAPÍTULO 7
NÚMEROS PRIMOS
O que prova o resultado.
O próximo resultado relacionará Ep ( n!) e a representação p-ádica de n (i.e., a representação
relativa à base p)
Teorema 7.12. Sejam p,n inteiros positivos, com p primo. Suponha que.
n = nrpr + nr-1pr-1 + ... + n1p + no
Seja a representação p – ádica de n . Então.
Demonstração: Sendo 0
ni
p , temos que
Portanto,
Exemplo: Seja determinar a potência de 3 na decomposição de 53! em fatores primos.
Primeiramente escrevemos 53 na base 3, isto é:
53 = (1222)3
Aplicando o Teorema 7.12
104
CAPÍTULO 7
NÚMEROS PRIMOS
Verificando esse resultado pelo Teorema de Lagrange:
7.10. Método da Fatoração de Fermat
Até o momento, um dos procedimentos matemáticos mais difíceis é o de fatorar um número
arbitrariamente grande e isso às vezes requer um tempo razoável. Para os casos mais simples
podemos usar os conhecidos testes de divisibilidade, mas fatorar números grandes é objeto de
intensas pesquisas matemáticas. Damos uma aqui um uma ideia desse difícil problema
matemático, utilizando o chamado método da Fatoração de Fermat. Em cursos mais
avançados outros métodos são apresentados.
Proposição 7.3: Seja n > 1 um inteiro ímpar. Há uma correspondência biunívoca entre a
fatoração de n e a representação de n como diferença de dois quadrados.
Demonstração:
Se n = a.b, e n ímpar, então a e b são ímpares. Logo a+b e a-b são pares, então
são inteiros.
Então,
e
Expressa n como a diferença de dois quadrados.
Reciprocamente, suponha n escrito como a diferença de dois quadrados:
n = s2 – t2, então n = (s-t) . (s+t) é a forma fatorada de n.
Você pode ver que esses dois procedimentos – da fatoração para a diferença e da diferença
para a fatoração – determinam uma relação biunívoca.
7. 11 – Algoritmo de Fermat
A proposição acima nos permite descrever um algoritmo, que é muito eficiente quando n tem
um fator primo que não é muito menor que
.
Para começar vamos supor que n é ímpar, já que se n for par então 2 é um de seus fatores. A
idéia do algoritmo de Fermat é tentar achar números inteiros positivos x e y tais que n = x2 - y2
. Supondo que encontramos estes números, temos que
n = x2 - y2 = (x - y) (x + y).
105
CAPÍTULO 7
NÚMEROS PRIMOS
Logo x - y e x + y são fatores de n.
O caso mais fácil do algoritmo de Fermat ocorre quando n é um quadrado perfeito; isto é,
quando existe algum inteiro r tal que n = r2. Neste caso temos que r é fator de n. Além disso,
na notação acima x = r e y = 0. Observe que se y > 0 então
Isto sugere a seguinte estratégia para encontrar x e y.
Entrada: inteiro positivo ímpar n.
Saída: um fator de n ou uma mensagem indicando que n é primo.
Etapa 1: Comece com
; se n = x2 então x é fator de n e podemos parar.
Etapa 2. Caso contrário incremente x de uma unidade e calcule
.
Etapa 3. Repita a Etapa 2 até encontrar um valor inteiro para y, ou até que x seja igual a
no primeiro caso n tem fatores x+y e x-y, no segundo n é primo.
:
Exemplo: Seja n = 1342127 o número obtido como produto de dois primos. A variável x é
inicializada com a menor parte inteira da raiz quadrada de n
. Mas x2
= 11582 = 1340964 < 1342127 logo passamos a incrementar x de um em um. Fazemos isso
até que
seja inteiro, ou x seja igual a
, que neste caso valeria 671064. É mais
fácil resumir isto em uma tabela
x
1159
33,97
1160
58,93
1161
76,11
1162
90,09
1163
102,18
1164
113
Obtivemos assim um inteiro no sexto laço. Portanto x = 1164 e y = 113 são os valores
desejados. Os fatores correspondentes são x + y = 1277 e x – y = 1051. Logo, 1051 e 1277
são os dois números primos procurados.
106
CAPÍTULO 7
NÚMEROS PRIMOS
EXERCÍCIOS
1.
Com uma calculadora, achar todos os primos
da forma n – 2, para 25
2
n
35
2.
Determine todos os primos que são iguais a
diferença de quadrado entre dois primos.
3.
De quantos modos podem escrever 497 como a
soma de dois números primos?
4.
Mostrar que a soma de dois inteiros positivos
ímpares e consecutivos nunca é um primo.
5.
Em um quadro estão escritos alguns números
naturais. Dentre eles, há nove múltiplos de 4,
sete múltiplos
de 6, cinco múltiplos de 12,
três números primos e nada mais. Qual a
quantidade mínima de números escritos?
6.
Achar todos os primos p e q, tais que p – q =
3.
7.
Achar todos os primos que são iguais a um
quadrado menos 1.
8.
Achar todos os primos que são iguais a um
cubo menos 1.
9.
Escreva os números 55, 83 e 211 como uma
soma de três primos.
10. Determinar todos os inteiros positivos n tais
que n, n + 2 e n + 4 são todos primos.
11. Determinar todos os primos p tais que 3p + 1 é
um quadrado.
12. Com uma calculadora, determinar se são
primos os números
a)
b)
c)
d)
1699
7429
21793
1189
13. Encontre todos os primos p, tais que 17p + 1 é
um quadrado.
14. Usando a decomposição em fatores primos dos
inteiros 507 e 1287, achar o mdc (507, 1287) e
o mmc (507, 1287).
15. Achar o mdc(a, b) e mmc(a, b) sabendo a =
230 . 521 . 19 . 233 e b = 26 . 3 . 74 . 112 .
195 . 237
16. Achar o menor inteiro positivo pelo qual se
deve dividir 15! para se obter um quadrado.
Qual o menor valor do número natural n
que torna n! divisível por 1000?
17. Achar todos os primos que são divisores de
50!.
18. Verifique com uma calculadora, se são primos
gêmeos:
a) 1949 e 1951
b) 1997 e 1999
19. Achar uma sequência de quatro inteiros
positivos consecutivos e compostos.
20. Achar um sequência de 100 inteiros positivos
consecutivos e compostos.
21. Mostre que nenhum número inteiro da forma
1 4n é divisível pelo número primo 3.
22. Com uma calculadora, verificar a conjectura de
Goldbach para n par, 42
n
100 .
23. Determinar o menor valor positivo do inteiro n
tal que 2n2 + p, seja um número inteiro
composto e p um primo terminado em 7.
24. Demonstrar que todo primo, p 5 é da forma
6k – 1 ou 6k + 1, onde k é um inteiro positivo.
25. Demonstrar que todo primo p 3, é da forma
4k + 1 ou 4k – 1, onde k é um inteiro positivo.
26. Determine todos os primos p
3003 também seja primo.
5tais que 8p4 -
27. Mostrar que todo inteiro da forma n4 + 4, com
n > 1 não é primo.
28. Mostrar que todo inteiro da forma 8 n + 1, com
n > 1, não é primo.
29. Mostrar que se n2 + 2 é primo então 3 | n, para
todo n > 1.
107
CAPÍTULO 7
NÚMEROS PRIMOS
30. Se p > 5 é um primo, então p2 + 2 é composto.
45. Mostre que se n 1 é natural então, o número
22
31. Demonstrar as seguintes propriedades:
a)
Todo primo da forma 3n + 1 é também da
forma 6m + 1.
b) Todo inteiro n > 11 pode ser expresso
como a soma de dois inteiros não-primos.
c) Se p 5 é um primo ímpar, então p2 – 1 ou
p2 + 1 é divisível por 10.
d) Se p > q > 5 e se p e q são ambos primos,
então 24 | p2 – q2.
e) Todo inteiro da forma 3n + 2 tem um fator
primo desta forma.
f) Se p é um primo e se p | an , então pn | an.
32. Demonstrar que o inteiro positivo a > 1 é um
quadrado se e somente se todos os expoentes
dos fatores primos da sua decomposição são
inteiros pares.
33. Demonstrar que, se o inteiro k
2, não é
primo, então 2k – 1 nunca será primo.
não é primo.
46. Sendo n > 1 um inteiro, prove que 4n + n4 não
é primo.
47. Mostrar, mediante um exemplo, que a seguinte
conjectura é falsa:
“Todo inteiro positivo maior que 1, pode-se
escrever sob a forma a2 + p, com a > 0 e p é um
inteiro primo ou 1”.
48. Determine todos os números primos p e q, para
os quais os q números p, p + (q + 1), p + 2 (q
+ 1), p + 3 (q + 1), . . . , p + (q − 1) (q + 1),
também são primos.
49. Demonstrar que existem infinitos primos da
forma 4n + 3, com n inteiro positivo.
50. Seja m um intero positivo. Demonstre que não
existem números primos da forma 25m + 2m +1.
2) é primo,
51. Determinar o número inteiro positivo n que
que é produto dos primos p, q e r, sabendo que
r - q = 2p e rq + p2 = 676.
35. Seja p o maior fator primo do número 314 + 313
– 12, então p é igual a:
52. Mostre que existem infinitos valores primos p
para os quais 8.p2 + 5 é divisível por 77.
36. Sejam p, q inteiros positivos. Mostre que 2p + 1
= q2 implica p e q primos e p = q = 3.
53. Seja p > 2 um primo. Determine todos os
valores inteiros positivos de m e n,
tal que
(p – 1) (pn + 1) = 4m (m + 1).
34. Demonstrar que se 2k – 1, (k
então k também é primo.
37. Mostrar que um inteiro da forma 42n+1 + 1,
onde n 1, nunca é primo.
38. Sendo n um inteiro positivo, mostre que 24(n+1)
– 1 nunca será primo.
39. Mostrar que se n > 4, não é primo, então n
divide (n – 1)!.
40. Verificar que todo inteiro pode escrever-se sob
a forma 2k m, onde o inteiro k > 0 e m é um
inteiro ímpar.
41. Demonstrar que, se o inteiro n > 2, então existe
um primo p tal que n < p < n!.
42. Qual é o menor número primo que um fator da
soma 19992002 + 20012002?
43. Prove que um triângulo retângulo não pode
apresentar as medidas de seus lados sendo
números primos.
2
44. Se p e 8p + 1 são números primos, prove que
p = 3.
108
2n 1

Nos problemas que se seguem faça uso de uma
calculadora para verificar os resultados e
explicite bem os passos utilizados na resolução.
54. Segundo o Teorema de Chebychev, para um
inteiro m 2, existe um primo p tal que m < p
< 2 m. Determine todos os primos entre 600 e
1200.
55. Segundo o Teorema de Dirichlet, se o mdc (a,
b) =1, então existem infinitos primos da forma
an+b com n um inteiro positivo. Determine
todos os primos p da forma 4n+9, com
88 < 4n + 9 < 388.
56. Usando o Teorema de Sierpinski, determine
um primo p>19 e escreva 20 inteiros
compostos.
57. Usando a Fórmula de Minàc, determine
(12) .
CAPÍTULO 7
NÚMEROS PRIMOS
58. Usando a Fórmula do n-ésimo número primo,
determine o quarto número primo.
59. Calcule:
a)
3#.5 # 7 #
11#
b)
5 #.7 # 11#
13#
60. Verifique se existem primos gêmeos entre 600
e 700.
61. Determine dois números primos consecutivos
tais que a diferença entre eles seja maior que 7.
62. Decomponha 98! Em fatores primos.
63. Determine a potência de 5 na decomposição de
75!
em fatores primos,
fazendo a
decomposição p-ádica de 75.
64. Com quantos zeros termina o número 1000! ?
Qual é a potência de 3 que aparece na
decomposição de 1000! em fatores primos?
65. Justifique se o número
93.94. ... .112.113
21!
é inteiro. Em caso afirmativo, calcule o seu
valor.
66. Encontrar o maior valor do inteiro n
que
0 tal
10200!
seja inteiro.
504n
67. Utilizando o Teorema do Número Primo:
a)
Faça uma estimativa (sem muito rigor) de
quantos primos de 200 dígitos existem.
b) Mostre que entre os números de k-dígitos, um
em cada 2, 3k é primo.
68. Qual o menor valor do número natural n que
torna n! divisível por 1000?
109
Capítulo 8:
EQUAÇÕES DIOFANTINAS
LINEARES
UM POUCO DE HISTÓRIA SOBRE DIOFANTO
Diofanto tem o seu nome ligado à cidade que foi o maior centro de atividade matemática na
Grécia antiga. Pouco se sabe acerca da sua vida, o desconhecimento impede-nos mesmo de
fixar com segurança em que século viveu. Têm sido sugeridas datas distanciadas de um
século, antes ou depois do ano 250 d. C. Por uns versos encontrados no seu túmulo, escritos
em forma de um enigmático problema, deduz-se que viveu 84 anos. Positivamente, tal
problema não deve ser tomado como o paradigma dos problemas sobre os quais se interessou
Diofanto, pois ele pouca atenção deu a equações do 1º grau. Alexandria foi sempre um centro
muito cosmopolita e a matemática que se originou nela não era toda do mesmo tipo. Os
resultados de Heron eram bem diferentes dos de Euclides ou dos de Apolonios ou dos de
Arquimedes, e na obra de Diofanto há novamente uma quebra abrupta da tradição clássica
grega. Sabido é que os gregos, na época clássica, dividiram a aritmética em dois ramos: a
aritmética propriamente dita como "teoria dos números naturais". Frequentemente, tinha mais
em comum com a filosofia platônica e pitagórica do que com o que habitualmente se
considera como matemática, e logística ou cálculo prático que estabelecida as regras práticas
de cálculo que eram úteis à Astronomia, à Mecânica, etc. O principal tratado de Diofanto
conhecido, e que. ao que parece, só em parte chegou até nós, é a "Aritmética". Apenas seis
dos livros originais em grego sobreviveram, o número total (13) não passa de uma conjectura.
Era um tratado caracterizado por um alto grau de habilidade matemática e de engenho, pelo
que pode ser comparado aos grandes clássicos da "Primeira idade Alexandrina", ou seja, da
"época de ouro" da matemática grega, no entanto, quase nada têm em comum com esses ou,
na verdade, com qualquer matemática grega tradicional. Representa essencialmente um novo
ramo e usa um método diferente, dai a época em que possivelmente Diofanto viveu se chamar
"segunda idade Alexandrina", conhecida por sua vez por "época de prata" da matemática
grega. Diofanto, mais que um cultor da aritmética, e, sobretudo da geometria, como o foram
os matemáticos gregos anteriores, deve considerar-se um precursor da álgebra, e, em certo
sentido, mais vinculado com a matemática dos povos orientais (Babilônia, Índia,...) que com a
dos gregos. A sua "Aritmética” assemelha-se à álgebra babilônica em muitos aspectos, mas
enquanto os matemáticos babilônicos se ocupavam principalmente com soluções
“aproximadas" de equações "determinadas" e, sobretudo de equações "indeterminadas" do 2º
e do 3º graus das formas canônicas, em notação atual, Ax2+Bx+C = y2 e
Ax3+Bx2+Cx+D=y2, ou conjuntos (sistemas) destas equações. É exatamente, por esta razão
em homenagem a Diofanto -que a esta "Análise indeterminada" se chama “Análise
diofantina” ou “Análise diofântica". No desenvolvimento histórico da álgebra considera-se,
em geral, que podem ser reconhecidos três estádios: o primitivo ou retórico, em que tudo era
completamente escrito em palavras, um intermédio ou sincopado, em que foram adaptadas
algumas abreviaturas e convenções, e um final ou simbólico, em que são usados somente
símbolos. A "Aritmética" de Diofanto deve ser colocada no segundo estádio; nos seus seis
livros há um uso sistemático de abreviaturas para potências de números e para relações e
operações.
110
CAPÍTULO 8
EQUAÇÕES DIOFANTINAS LINEARES
3.1. Generalidades
A teoria das Equações Diofantinas é o ramo da Teoria dos Números que investiga as soluções
inteiras de equações polinomiais, como por exemplo:
x2 + y2 = z2, possui infinitas soluções representadas pelas ternas ordenadas (x,y,z)
conhecidas como ternos pitagóricos.
xn + yn = zn, que não possui soluções não nulas para para n > 2, e é conhecida como o
Último Teorema de Fermat.
y2 = x3 + 17, que é válida, por exemplo, para os seguintes valores positivos:
(4,9); (8,23); (43, 282); (52, 375); ...
Equação de Pell : x 2 dy 2 m , onde
quadrado e m é um inteiro qualquer.
Etc...
(2,5) ;
d um inteiro positivo que não seja um
O tipo mais simples de equação diofantina é a equação diofantina linear com duas incógnitas
x e y:
ax + by = c
onde a, b e c são inteiros dados, sendo ab 0 .
Todo par de inteiros x 0 , y 0 tais que a x 0 + b y 0 = c diz-se uma solução inteira ou apenas uma
solução da equação ax + by = c.
Consideremos, por exemplo, a equação diofantina linear com duas incógnitas:
3x + 6y = 18
Temos:
3.4 + 6.1 = 18
3(-6) + 6.6 = 18
3.10 + 6(-2) = 18
Logo, os pares de inteiros:
4 e 1, -6 e 6, 10 e -2
são soluções da equação 3x + 6y = 18
Existem equações diofantinas lineares com duas incógnitas que não têm solução. Assim, por
exemplo, a equação diofantina linear:
2x + 4y = 7
111
CAPÍTULO 8
EQUAÇÕES DIOFANTINAS LINEARES
não tem solução, porque 2x + 4y é um inteiro par quaisquer que sejam os valores inteiros de x
e y, enquanto que 7 é um inteiro ímpar (observe-se que 2 = mdc (2, 4) não divide 7).
De modo geral, a equação diofantina linear ax + by = c não tem solução todas as vezes que d
= mdc (a, b) não divide c, como é óbvio.
3.2. Condição de Existência de Solução
Teorema 3.1: A equação diofantina linear ax + by = c tem solução se e somente se d divide c,
sendo d = mdc (a, b).
Demonstração:
Suponhamos que a equação ax + by = c tem uma solução, isto é, que existe um par de
inteiros x 0 , y 0 tais que a x 0 + b y 0 = c.
Por ser o mdc (a, b) = d, existem inteiros r e s tais que a = dr e b = ds, e temos:
c = a x 0 + b y 0 = dr x 0 + ds y 0 = d(r x 0 + s y 0 )
e como r x 0 + s y 0 é um inteiro, segue-se que d divide c ( d | c ).
Reciprocamente, suponhamos que d divide c ( d | c ), isto é, que c = dt, onde t é um inteiro.
Por ser o mdc (a, b) = d, existem inteiros x 0 e y 0 tais que
d = a x 0 + b y0
o que implica:
c = dt = (a x 0 + b y 0 )t = a(t x 0 ) + b(t y 0 )
isto é, o par de inteiros:
x = t x 0 = (c/d) x 0 , y = t y 0 = (c/d) y 0
é uma solução da equação ax + by = c.
112
CAPÍTULO 8
EQUAÇÕES DIOFANTINAS LINEARES
3.3. Soluções da equação ax + by = c.
Teorema 3.2: Se d divide c ( d | c ), sendo d = mdc (a, b), e se o par de inteiros x 0 , y 0 é uma
solução particular da equação diofantina linear ax + by = c, então todas as outras soluções
desta equação são dadas pelas fórmulas:
x
x0
b
t, y
d
y0
a
t
d
onde t é um inteiro arbitrário
Demonstração: Suponhamos que o par de inteiros x 0 , y 0 é uma solução particular da
equação ax + by = c, e seja x1 , y1 uma outra solução qualquer desta equação. Então, temos:
a x 0 + b y 0 = c = a x 1 + b y1
e, portanto:
a( x1 - x 0 ) = b( y1 - y 0 )
Por ser o mdc (a, b) = d, existem inteiros r e s tais que a = dr e b = ds, com r e s primos entre
si. Substituindo estes valores de a e b na igualdade anterior e cancelando o fator com d,
obtemos:
r( x1 - x 0 ) = s( y 0 - y1 )
Assim sendo, r | s(y 0
y1 ) , e como o mdc (r, s) = 1, segue-se que r | (y0
y0
y1 = rt
e
x1
y1 ) , isto é:
x 0 = st
onde t é um inteiro. Portanto, temos as fórmulas:
x1 x0 st x0 (b / d)t
y1 y0 rt y0 (a / d)t
Estes valores de x1 e y1 satisfazem realmente a equação ax + by = c, qualquer que seja o
inteiro t, pois, temos:
a x1 + b y1 = a[ x0 (b / d)t ] + b[ y 0 (a / d)t ] = a x 0 + b y 0 + (ab/d – ab/d)t = c + 0.t = c
113
CAPÍTULO 8
EQUAÇÕES DIOFANTINAS LINEARES
Como se vê, se d = mdc (a, b) divide c ( d | c ), então a equação diofantina linear ax + by = c
admite um número infinito de soluções, uma para cada valor do inteiro arbitrário t.
Corolário 3.1: Se o mdc (a, b) = 1 e se x 0 , y 0 é uma solução particular da equação diofantina
linear ax + by = c, então todas as outras soluções desta equação são dadas pelas fórmulas:
x
x0 bt , y
y0
at
onde t é um inteiro arbitrário.
Nota: Uma solução particular da equação diofantina linear se obtém por tentativas ou pelo
algoritmo de Euclides. E em ambos os casos a solução geral se pode obter usando o teorema
3.2, conforme se vai ver nos exemplos a seguir.
Exemplo 3.1: Determinar todas as soluções inteiras e positivas da equação diofantina linear
18x + 5y = 48
Determinemos o mdc (18, 5) pelo algoritmo de Euclides:
18 = 5.3 + 3
5 = 3.1 + 2
3 = 2.1 + 1
2 = 1.2
18
3
1
1
2
5
3
3
2
2
1
1
0
Portanto, o mdc (18, 5) = 1 e a equação dada tem solução. e para exprimir 1 como
combinação linear de 18 e 5 basta eliminar os restos 2 e 3 entre as três primeiras igualdades
anteriores do seguinte modo:
1 = 3 – 2 = 3 – (5 – 3) = 2.3 – 5 = 2(18 – 5.3) – 5 = 18.2 + 5(-7)
isto é:
1 = 18.2 + 5(-7)
e
48 = 18.96 + 5(-336)
Logo, o par de inteiros x 0 = 96, y 0 = -335 é uma solução particular da equação proposta, e
todas as demais soluções são dadas pelas fórmulas:
x = 96 + 5t, y = -336 – 18t
114
CAPÍTULO 8
EQUAÇÕES DIOFANTINAS LINEARES
onde t é um inteiro arbitrário.
As soluções inteiras e positivas se acham escolhendo t de modo que sejam satisfeitas as
desigualdade:
96 + 5t > 0, -336 – 18t > 0
Isto é:
t > 19, 2
e t < 18, 6
o que implica t = -19 e, portanto:
x = 96 + 5(-19) = 1, y = -336 -18(-19) = 6
Assim, o par de inteiros x = 1, y = 6 é a única solução inteira e positiva da equação 18x + 5y =
48.
Exemplo 3.2: Resolver a equação diofantina linear
39x + 26y = 105
O mdc (a, b) = 13 e como 13 não divide 105, segue-se que a equação dada não tem solução.
EXERCÍCIOS
1)
Determinar todas as soluções inteiras das
seguintes equações diofantinas lineares:
a)
b)
c)
d)
e)
f)
g)
h)
i)
j)
k)
l)
56x + 72y = 40
24x + 138y = 18
221x + 91y = 117
84x – 438y = 156
48x + 7y = 5
57x – 99y = 77
11x + 30y = 31
27x – 18y = 54
13x – 7y = 21
44x + 66y = 11
21x – 12y = 72
17x + 54y = 8
2)
Determinar todas as soluções inteiras e
positivas das seguintes equações diofantinas
lineares:
a)
b)
c)
d)
e)
f)
g)
h)
3)
5x – 11y = 29.
32x + 55y = 771
58x – 87y = 290
62x + 11y = 788
30x + 17y = 300
54x + 21y = 906
123x + 360y = 99
158x – 57y = 7
Determinar o menor inteiro positivo que
dividido por 8 e por 15 deixa os restos 6 e 13,
respectivamente.
115
CAPÍTULO 8
EQUAÇÕES DIOFANTINAS LINEARES
4)
Exprimir 100 como soma de dois inteiros
positivos de modo que o primeiro seja
divisível por 7 e o segundo seja divisível por
11.
5)
Determinar as duas menores frações positivas
que tenham 13 e 17 para denominadores e
cuja soma seja igual a 305 .
221
6)
Determine todas as soluções inteiras do
sistema de equações
2 x 3 y 5 z 201
13) O laboratório Sangue Bom, dispõe de 2
máquinas para examinar amostras de sangue.
Uma delas examina 15 amostras de cada vez,
enquanto a outra examina 25. Quantas vezes
essas máquinas devem ser acionadas para
examinar exatamente 2 mil amostras?
14) Num determinado lugar a moeda é o mirrél.
Suponhamos que só existam moedas de 15 e
7 mirréis e que se queira pagar uma
determinada quantia em mirréis. Será que é
sempre possível? E se existirem moedas de
12 e 30 mirréis?
3x 5 y 7 z 315
Encontre todas as soluções da equação
Diofantina (6x+15y)(8x+7y) = 129.
8)
Uma pessoa foi ao banco para descontar um
cheque no valor de x reais e y centavos.
O caixa do banco errou na leitura do valor do
cheque e pagou y reais e x centavos. A pessoa
guardou o dinheiro no bolso sem verificar a
quantia. No caminho de casa, ela gastou cinco
centavos e quando chegou em casa verificou
que tinha exatamente o dobro do valor do
cheque. Sabendo-se que essa pessoa não levou
dinheiro nenhum consigo quando foi ao
banco, pergunta-se qual era o valor do cheque.
16) Para participar de um evento comemorativo
em um clube, não sócios pagavam R$ 12,00 e
sócios R$ 8,00. Sabendo-se que foram
arrecadados R$ 908,00 na portaria, quantos
sócios estiveram no evento?
Um grupo de pessoas gastou 1000 dólares
num hotel. Sabendo-se que apenas alguns dos
homens estavam acompanhados pelas esposas
e que cada homem gastou 19 dólares e cada
mulher gastou 13 dólares, pede-se determinar
quantas mulheres e quantos homens estavam
no hotel.
18) Demonstrar que se a e b são inteiros positivos
primos entre si, então a equação diofantina ax
– by = c têm um número infinito de soluções
inteiras e positivas.
9)
10) Um grupo de pessoas gastou 690 dólares num
hotel. Sabendo-se que apenas alguns dos
homens estavam acompanhados pelas esposas
e que cada homem gastou 18 dólares e cada
mulher gastou 15 dólares, pede-se determinar
quantas mulheres e quantos homens estavam
no hotel.
11) Ao entrar num bosque, alguns viajantes
avistam 37 montes de maçãs. Após serem
retiradas 17 frutas, o restante foi dividido
igualmente entre 79 pessoas. Qual pode ter
sido a parte de cada pessoa?
12) Sabendo que um time de basquete é composto
de 5 jogadores e um time de vôlei é formado
por 6 jogadores. Quantas quadras de basquete
e quantas de vôlei são necessárias para que 80
alunos joguem simultaneamente qualquer um
dos esportes? E se forem 77 alunos?
116
15) Para agrupar 13 aviões em filas de 3 ou de 5,
exatamente quantas filas serão formadas de
cada tipo?
7)
17) Um galo custa 5 mirréis, uma galinha 3
mirréis, e três pintinhos 1 mirrél. Com 100
mirréis um fazendeiro comprou 100 dessas
aves. Quantos galos, galinhas e pintinhos
foram comprados?
Capítulo 9
CONGRUÊNCIAS
9.1. Congruências
Definição 9.1 Sejam a e b inteiros quaisquer e seja m >1 um inteiro positivo fixo. Diz-se que
a é congruente a b módulo m se, e somente se, m divide a diferença a – b. Em outros termos a
é congruente a b módulo m se, e somente se, existe um inteiro k tal que a – b = km
Simbolicamente:
a
Exemplos 9.1 3
b (mod m)
m|(a–b)
24 (mod 7) ; –31
a - b = km
a = km + b
11 (mod 6) ; –15
–63 (mod 8)
Definição 9.2 Se m não divide a diferença a – b, então diz-se que a é incongruente a b
módulo m .
Notação: a b (mod m)
Observações:
1) Dois inteiros quaisquer são congruentes módulo 1
2) Dois inteiros são congruentes módulo 2, se ambos são pares ou ambos são
ímpares
3) a 0 (mod m) se, e somente se, m | a.
9.2. Caracterização de Inteiros Congruentes
Teorema 9.1 Dois inteiros a e b são congruentes módulo m se, e somente se, a e b deixam o
mesmo resto quando divididos por m.
Demonstração:
(
) Suponhamos que a
b ( mod m). Então, pela definição:
a – b = km, k
Z
117
CAPÍTULO 9
CONGRUÊNCIAS
Seja r o resto da divisão de b por m; então pelo algoritmo da divisão:
b = mq + r, 0
r<m
Portanto:
a = km + b = km + mq + r = (k + q)m + r
e isto significa que r também é o resto da divisão de a por m, isto é, os inteiros a e b divididos
por m deixam o mesmo resto r.
( ) Reciprocamente, suponhamos que a e b divididos por m deixam o mesmo resto r. Então,
podemos escrever:
a = mq1 + r e b = mq2 + r , 0
r<m
e, portanto:
a – b = ( q1 – q2) m
m (a – b)
a b ( mod m)

9.3. Propriedades das Congruências
Teorema 9.2 Seja m um inteiro positivo fixo (m > 1) e sejam a, b e c inteiros quaisquer.
Valem as propriedades:
1) a a (mod m) (Reflexiva)
2) Se a b (mod m), então b a (mod m) (Simétrica)
3) Se a b (mod m) e se b c(mod m), então a c (mod m) (Transitiva)
Demonstração:
(1)
(2)
a 0 ou seja, a (a –a), o que implica: a a (mod m)
Se a b (mod m), então a – b = km, k z.

Portanto:
b – a = -(km) = (-k)m
(3) Se a
b (mod m) e se b
b
a (mod m)

c (mod m), então existem inteiros h e k tais que
a– b = hm e b – c = km
Portanto:
a – c = (a - b) + (b - c) = hm + km = (h + k)m
118
CAPÍTULO 9
CONGRUÊNCIAS
e isto significa que a
c (mod m).

Nota: Consoante este teorema, as relação binária R no conjunto Z dos
inteiros definidas por
aRb
a b (mod m) é reflexiva, simétrica e transitiva, ou seja, R é uma
relação de equivalências em Z. Esta relação de equivalência R em z é
denominada “congruência módulo m” .
Teorema 9.3 Seja m um inteiro positivo fixo (m > 1) e sejam a, b dois inteiros quaisquer.
Valem as seguintes propriedades:
1) Se a
b (mod m) e se n | m, com n > 0, então a
b (mod n)
Demonstração: Com efeito:
a
b (mod m)
a – b =km e n m
m = nq
onde k e q >0 são inteiros.
Portanto:
a –b = (kq)n
2) Se a
b (mod m) e se c > 0 , então ac
Demonstração: Com efeito, se a
a – b = km
a
b (mod n)

bc (mod mc)
b (mod m), então:
ac – bc = k(mc)
ac
bc (mod mc)

3) Se a
b (mod m) e se a, b, m são todos divisíveis pelo inteiro d > 1, então
a b
m
(mod
)
d d
d
Demonstração:
Com efeito, se a
b (mod m), então:
a - b = km
a b
m
- = k( )
d d
d
a
d
b
m
(mod )
d
d

119
CAPÍTULO 9
CONGRUÊNCIAS
Teorema 9.4 Seja m um inteiro positivo fixo (m > 1) e sejam a, b, c, d inteiros quaisquer.
Valem as seguintes propriedades:
1) Se a b (mod m) e se c
(mod m).
d (mod m), então a + c
b + d (mod m)
e ac
bd
e ac
bc
Demonstração:
Se a b (mod m) e se c d (mod m), então existem inteiros h e k tais que
a – b = hm e c – d = km.Portanto:
(a + c) – (b + d) = (a - b)+ (c- d) = (h + k)m
e
ac – bd = (b + hm) (d + km) - bd = (bk + dh + hkm)m
o que implica:
a+c
b + d (mod m) e ac
bd (mod m)
2) Se a b (mod m) e c um inteiro qualquer, então a + c
(mod m).

b + c (mod m)
Demonstração: Temos:
a
b (mod m) e c
c (mod m)
Logo, pela propriedade anterior:
a+c
b + c (mod m) e ac
bc (mod m)

Em particular, se c = -1, então:
a (-1)
3)
Se a
b(-1) (mod m)
b (mod m), então an
ou -a -b (mod m)
bn (mod m) para todo inteiro positivo n.
Demonstração:
Usando o “Teorema da indução Matemática”, a proposição é verdadeira para
suposta verdadeira para o inteiro positivo k temos:
ak
b k (mod m) e a
Portanto, pela propriedade 1 acima
120
b (mod m)
n = 1, e
CAPÍTULO 9
CONGRUÊNCIAS
a k .a
b k .b (mod m) ou ak+1
bk+1 (mod m)
isto é, as proposição é verdadeira para o inteiro positivo k + 1. logo, a preposição é verdadeira
para todo inteiro positivo n. 
Teorema 9.5: Se ac
bc (mod m) e se o mdc(c,m) = d, então a
Demonstração: Com efeito, se ac
b (mod
m
)
d
bc (mod m), então:
ac – bc = (a – b)c = km , com k
Z.
Como o mdc (c,m) = d, existem inteiro r e s tais que c = dr e m = ds, onde r e s são primos
entre si. Portanto:
(a – b) dr = kds
ou
(a - b)r = ks
o que implica que s (a – b)r, com o mdc (r,s) = 1. Logo, pelo Teorema 5.4 ( de Euclides): s
m
m
(a – b) e a b (mod s) ou, por ser s = , a b (mod
). 
d
d
Corolário 9.1 Se ac bc ( mod m) e se o mdc (c,m) = 1, então a b (mod m).
Esta propriedade mostra que é permitido cancelar fatores de ambos os membros de uma
congruência que são primos com o módulo.
Corolário 9.2 Se ac
a
bc (mod p), com p primo, e se p não divide c, então
b (mod p)
Demonstração: Com efeito, as condições:
mdc(c, p) = 1. 
p não divide c e p é primo, implicam que o
9.4. Sistemas Completos de Restos
Definição 9.3 Chama-se sistema completo de restos módulo m todo conjunto
S = {r1 , r2 , ... , rm} de m inteiros tal que um inteiro qualquer a é congruente módulo m a um
único elemento de S.
Exemplo 9.2: Cada um dos conjuntos:
{1, 2, 3} , {0, 1, 2} , { –1, 0, 1} , {1, 5, 9}
é um sistema completo de restos módulo 3.
121
CAPÍTULO 9
CONGRUÊNCIAS
Teorema 9.6 O conjunto S = {0,1, 2, ..., m – 1} é um sistema completo de restos módulo m.
Demonstração: Com efeito, o conjunto S tem m elementos e, além disso, qualquer que seja o
inteiro a temos, pelo algoritmo da divisão:
a = mq + r, com 0 ≤ r ≤ m
o que implica a ≡ r (mod. m), e como o resto r só pode assumir os m valores 0, 1, 2, ..., m-1,
segue-se que o inteiro a é congruente módulo m a um único desses m inteiros. 
Corolário 9.3 Se S = {r1 , r2 , ... , rm} é um sistema completo de restos módulo m, então os
elementos de S são congruentes módulo m aos inteiros 0, 1, 2, ... , m – 1, tomados numa certa
ordem.
Demonstração: Com efeito, se a é um inteiro qualquer, então:
a ≡ r1 (mod. m), com r1 Є S
a ≡ k (mod. m), com 0 ≤ k ≤ m-1
o que implica: r1 ≡ k (mod. m).
9.5 – Aritmética Módulo m
Definição 9.4.: Seja a um inteiro. Chama-se classe de congruência de a módulo m (m > 1)
o conjunto formado por todos os inteiros que são congruentes a a módulo m. Denotamos esse
conjunto por a . Temos, então:
a ={x
Z;x
a (mod m) }
Como x a (mod m), se e somente se, x é da forma x = a + k.m, para algum
k Z, também podemos escrever:
a = { a + k.m | k
Z}
Mostraremos a seguir que a relação de congruência entre números se traduz em igualdade no
sentido estrito entre classes.
Proposição 9.1: Sejam a e b inteiros. Então a b (mod m), se e somente se, a b .
Demonstração: Suponhamos que a b (mod m), queremos provar que a = b , isto é, uma
igualdade entre conjuntos. Dado x
a (mod m). Da
a , temos por definição que x
propriedade transitiva de congruência e da hipótese, segue imediatamente que x b . Logo, a
b . A inclusão b
a em sentido contrário segue de forma análoga.
122
CAPÍTULO 9
CONGRUÊNCIAS
Reciprocamente, se a = b , como a
a , temos também que a
Corolário 9.4: Sejam a e b inteiros. Se a
b , então a
b =
b , logo, a
b (mod m).

.
Demonstração: Se a
b = , consideremos um inteiro c que pertença a ambas as classes.
Como c a , temos que c a (mod m) e, de forma análoga, c b (mod m). Portanto, a b
(mod m) e, da proposição acima, a = b . 
Note que, por exemplo, para as classes módulo 7, temos que
0 7 14
7 ... ou 4 11
3 ... etc.
Mais precisamente, dada uma classe a , para qualquer inteiro x tal que x a ,
temos que x = a . Por causa disto, cada inteiro pertencente a uma dada classe
diz-se um representante daquela classe. Por exemplo, 11 e – 3 são
representantes da classe 4 módulo 7.
Consideremos um sistema completo de classes ou resíduos módulo m, por exemplo, os
inteiros 0, 1, ..., m – 1 e suas respectivas classes:
0 = { 0,
m,
1 = { 1, 1
2.m,
m, 1
3.m, ... }
2.m, 1
3.m, ... }
...
m 1 = { m – 1, m – 1
m, m – 1
2.m, m – 1
3.m, ... }
Conforme já foi considerado, cada inteiro pertence a uma e apenas uma das m classes.
Por exemplo, se m = 7, todas as classes possíveis, módulo 7, são as seguintes:
0 = { 0,
7,
14,
21, ... }
1 = { 1, 1 7, 1
14, 1
21, ... }
2 = { 2, 2 7, 2
14, 2
21, ... }
3 = { 3, 3 7, 3
14, 3
21, ... }
4 = { 4, 4 7, 4
14, 4
21, ... }
5 = { 5, 5 7, 5
14, 5
21, ... }
6 = { 6, 6 7, 6
14, 6
21, ... }
Denotaremos pelo símbolo  m o conjunto das classes de congruências módulo m e o
chamaremos de Conjunto dos Inteiros Módulo m.
Assim, Z7 = { 0 , 1 , 2 , 3 , 4 , 5 , 6 }.
123
CAPÍTULO 9
CONGRUÊNCIAS
Note que, por exemplo,
0 = 7 , 1 = 15 , 2 = 9 , 3 =
6 =
11 , 4 = 25 , 5 =
16 ,
8 e também podemos escrever:
Z7 = { 7 , 15 , 9 ,
11 , 25 ,
16 ,
8 }.
Em geral, se { a1, a2, ..., am } é um sistema completo de restos módulo m, temos que:
 m = { a1 , a2 , ..., a m }
Tomando o sistema de restos mais simples, podemos escrever:
 m = { 0 , 1 , 2 , ..., m 1 }
Note que, conforme as observações acima, o conjunto  m tem precisamente
m elementos.
9.6. Adição e Multiplicação em  m
Agora gostaríamos de introduzir operações de soma e produto em  m e estudar suas
propriedades. Existe uma forma natural de fazê-lo. Por exemplo, para somar e multiplicar 3 e
6 em Z7, faríamos:
3 + 6 = 9 = 2
3 . 6 = 18 = 4
124
CAPÍTULO 9
CONGRUÊNCIAS
Exemplificando a adição e a multiplicação em uma “tabuada” módulo 7:
+
0
1
2
3
4
5
6
*
0
1
2
3
4
5
6
0
0
1
2
3
4
5
6
0
0
0
0
0
0
0
0
1
1
2
3
4
5
6
0
1
0
1
2
3
4
5
6
2
2
3
4
5
6
0
1
2
0
2
4
6
1
3
5
3
3
4
5
6
0
1
2
3
0
3
6
2
5
1
4
4
4
5
6
0
1
2
3
4
0
4
1
5
2
6
3
5
5
6
0
1
2
3
4
5
0
5
3
1
6
4
2
6
6
0
1
2
3
4
5
6
0
6
5
4
3
2
1
Observações:
Observe, na tabela de adição, o conceito de inverso aditivo módulo m. Dizemos
que dois elementos de  m são inversos aditivos, se e somente se, a b 0(mod m)
. Assim, por exemplo, 4 e 3 são inversos aditivos módulo 7, uma vez que
4 3 0(mod 7) .
Mais explicitamente, definimos soma e produto em Zm por:
3 + 6 = 9 = 2
3 . 6 = 18 = 4
Quer dizer, para efetuar a soma de duas classes módulo m, tomamos representantes
(quaisquer) a e b dessas classes, efetuamos a soma a + b em Z e consideramos como resultado
da soma a classe de a + b módulo m . A operação de produto se faz de forma análoga.
Surge agora uma pergunta natural: será que o resultado das operações não depende dos
representantes escolhidos? Voltando ao exemplo de Z7 , para somar 3 + 6 , poderíamos tomar
38 como um representante de 3 e 27 como representante de 6 . Será que 38 + 27 = 65 é o
mesmo resultado que aquele obtido acima, 3 + 6 = 2 ? A resposta é afirmativa. Como 65 2
(mod 7), felizmente o resultado é o mesmo. O lema abaixo mostra que isso não é uma mera
coincidência.
Lema 9.1: Sejam a, a’, b e b’ inteiros tais que a = a' e b = b' . Então,
a
b = a' b' e a . b = a' . b' .
Demonstração: É uma conseqüência imediata das propriedades.
Proposição 9.2: Em  m valem as seguintes propriedades:
125
CAPÍTULO 9
CONGRUÊNCIAS
(P1) Propriedade Associativa: Para toda terna a , b , c de inteiros módulo m, tem-se que: a + (
b + c )=( a + b )+ c
(P2) Existência do Elemento Neutro: Existe um único elemento em  m que é precisamente 0
a classe do elemento 0, tal que: a + 0 = a , para todo a  m
(P3) Existência do Elemento Oposto: Para cada inteiro módulo m, a , existe um único
elemento em Zm que chamaremos oposto de a e indicaremos por a , tal que: a + ( a ) = 0
(P4) Propriedade Comutativa: Para todo par a , b de elementos de Zm tem-se que: a + b = b
+ a.
Demonstração: As demonstrações são feitas apoiando-se nos axiomas para as operações com
números inteiros. A título de ilustração, provaremos P1 e P3.
(P1)Utilizando repetidamente a definição de soma em  m , temos:
a +( b + c )= a +( b
c )= a
(b c)
Agora, como vale a associativa da soma entre números inteiros, ou seja:
a+(b+c)=(a+b)+c
temos que: a ( b c ) = ( a b ) c
logo, a + ( b + c ) = a ( b c ) = ( a b ) c = ( a + b ) + c
Na última sequência de igualdade usamos, novamente, apenas a definição de soma em  m .
( P3) Dado a  m , basta tomar a classe a e verificar que:
a +(
a) = a
Para provar a unicidade, suponhamos que b
( a) = 0
 m também verifica a + b = 0 ou, usando da
comutatividade, b + a = 0 . Temos então:
b = b + 0 = b +( a +(
a) ) = ( b + a ) + (
a) = 0 + ( a) =
a. 
A verificação de P2 é imediata. Note, porém, que a classe do elemento neutro é formada
também pelos múltiplos de m. Temos, assim, que 0 = m .
Da demonstração de P3 vem que o oposto de a em  m é a classe
a = – a . É claro que, se
explicitamos  m na forma  m = { 0 , 1 , 2 , ..., m 1 } e a é um dos representantes utilizados,
então – a não é um deles. Para obter o menor representante positivo da classe de – a, fazemos:
a = 0 +(
126
a) = m + (
a) = m – a = m
a
CAPÍTULO 9
CONGRUÊNCIAS
Por exemplo, em Z7 temos que
2 =0 –2 =7 –2 = 7
2 = 5 (de fato, 2 + 5 = 7 = 0 ).
Listamos na próxima proposição as propriedades do produto e a propriedade distributiva, que
relaciona ambas as operações.
Proposição 9.3: Em  m valem as seguintes propriedades:
(P5) Propriedade Associativa: Para toda terna a , b , c de inteiros módulo m, tem-se que: a . ( b
. c )=( a . b ). c
(P6) Existência do Elemento Neutro: Existe o único elemento em  m que é precisamente 1 , tal
que: a . 1 = a .
(P7) Propriedade do Elemento Oposto: Comentário mais adiante.
(P8) Propriedade Comutativa: Para todo par a , b de elementos de  m tem-se que: a . b = b .
a.
(P9) Propriedade Distributiva: Para toda terna a , b , c de elementos de Zm tem-se que:
a . (b + c ) = a .b + a .c .
Demonstração: Tal como na proposição anterior, deixaremos as demonstrações como
exercício. São feitas reduzindo-as ao caso dos inteiros.
Você deve ter notado que não listamos uma propriedade P7 que, no paralelismo que
estávamos fazendo com as propriedades das operações nos inteiros, corresponderia à
propriedade cancelativa. Isso ocorreu porque ela não é válida em geral. Com efeito, por
exemplo, em Z6 temos que 3 . 2 = 6 = 0 , 3 . 4 = 12 = 0 ; logo, 3 . 2 = 3 . 4 , porém 2 4 .
No contra-exemplo acima, temos dois elementos não-nulos de Z6 cujo produto é zero,
situação que não acontece em  . Para melhor estudar a propriedade cancelativa,
começaremos formalizando esse conceito.
Definição 9.5: Um elemento não-nulo a
 m diz-se um divisor de zero se existe b
m ,
também não-nulo, tal que a . b = 0 . Agora, determinaremos quais são os divisores de zero em
m .
Lema 9.2: Um elemento não-nulo a  m é divisor de zero, se e somente se,
m.d.c.(a, m) 1.
Demonstração: Seja a um divisor de zero e b
0 um elemento de  m tal que a . b = 0 . Como
0 (mod m), isto é, m divide a.b (representamos por “m | a.b”).
Supondo por absurdo que m.d.c.(a, m) = 1, o Teorema de Euclides diz que para a e b inteiros
tais que a | b.c, se m.d.c.(a, b) = 1, então a | c , assim, vem que m | b, logo, b = 0 , uma
contradição. Reciprocamente, suponhamos que m.d.c.(a, m) = d > 1. Vamos determinar um
elemento b 0 em Zm tal que a . b = 0 .
Podemos escrever a = a1 . d , e m = m1 . d, em que 0 < m1 < m (já que d > 1).
a . b = b . a = 0 , temos que a.b
Logo, m1
0 . Agora, temos que:
127
CAPÍTULO 9
CONGRUÊNCIAS
a . m1 = ( a1 . d ) . m1 = a1 . ( d . m1 ) = a1 . m
Logo, em Zm temos:
a . m1 = a1 . m = 0
Assim, basta tomar b = m1.

Como consequência imediata deste Lema temos o Corolário abaixo.
Corolário 9.5: Seja p > 1 um inteiro primo. Então,  p não contém divisores de zero.
Vale também a recíproca.
Lema 9.3: Se  m não contém divisores de zero, então m é primo.
Demonstração: Suponhamos, por absurdo, que m seja composto, isto é da forma m = r . s
com 1 < r < m, 1 < s < m. Temos, então, que: 0 = m = r . s em que r 0 e s 0, uma
contradição. 
Proposição 9.4: A propriedade cancelativa do produto vale em  m , se e somente se, m é
primo.
Demonstração: Suponhamos inicialmente que m seja primo, e sejam a , b , c elementos de
 m , com a
Como a
0, tais que a . b = a . c . Então, a .( b – c ) = 0.
0 e  m não tem divisores de zero, deve ser b – c = 0, donde b = c .
Suponhamos que a propriedade cancelativa seja válida, mostraremos que nesse caso que  m
não contém divisores de zero. A tese seguirá então do Lema anterior.
Sejam a , b Zm tais que a . b = 0 . Se a
cancelar, temos que b = 0. 
0 , escrevemos a . b = a . 0 e, como podemos
Para continuar nosso estudo comparativo de  com  m , introduzimos ainda outro conceito.
Definição 9.6: Um elemento a  m diz-se inversível se existe a'  m tal que a . a' = 1. Um
elemento a' nessas condições diz-se um inverso de a .
O conjunto dos elementos de  m que têm inversos é muito importante. Vamos denotá-lo por
 (m). Em outras palavras,
128
CAPÍTULO 9
CONGRUÊNCIAS
 (m)
 m ; m.d.c.(a, m) = 1 }
={ a
No caso de m ser primo, todas as classes diferentes de 0 possuem inverso.
Os únicos elementos inversíveis de  são 1 e –1.
Obviamente, 1 e 1 são sempre inversíveis em  m . Porém há outros exemplos.
Em Z5 temos que 2 . 3 = 6 = 1 e 4 . 4 = 16 = 1 , logo 2 , 3 e 4 são também inversíveis de Z5, 2
é o inverso de 3 e, reciprocamente, 4 é o seu próprio inverso.
Em Z6 temos que 5 . 5 = 25 = 1 ; logo, 5 é um inversível de Z6.
Por outro lado, é claro que 0 não é inversível em  m , para nenhum valor de m. De fato, para
qualquer a  m temos que 0 . a = 0 1 .
Observe as tabelas de multiplicação em Z6 e em Z7.
*
0
1
2
3
4
5
6
*
0
1
2
3
4
5
0
0
0
0
0
0
0
0
0
0
0
0
0
0
0
1
0
1
2
3
4
5
6
1
0
1
2
3
4
5
2
0
2
4
6
1
3
5
2
0
2
4
0
2
4
3
0
3
6
2
5
1
4
3
0
3
0
3
0
3
4
0
4
1
5
2
6
3
4
0
4
2
0
4
2
5
0
5
3
1
6
4
2
5
0
5
4
3
2
1
6
0
6
5
4
3
2
1
Com exceção de 0, todos os elementos em Z7 (7 é primo) possuem inverso, enquanto que em
Z6 (6 não é primo), apenas 1 e 5 possuem inverso. Isso está de acordo com a proposição
abaixo.
Proposição 9.5: Seja a um elemento não-nulo de  m . Então, a é inversível, se e somente se,
m.d.c.(a,m) = 1.
Demonstração: Suponhamos que m.d.c.(a, m) = 1. O Teorema de Bézout afirma que sendo a
e b inteiros, d = m.d.c.(a, b). Então existem inteiros r e s tais
que d = r.a + s.b . Também,
existem inteiros r e s tais que a.r + m.s = 1.
Tomando classes temos que:
1 = a.r m.s = a.r + m.s = a . r + m . s = a . r + 0 . s = a . r
Logo, r é o inverso de a .
Reciprocamente, se m.d.c.(a, m) 1, então a é divisor de zero e existe b 0 tal que a . b = 0 .
Mostraremos que, nesse caso, a não pode ser inversível. Com efeito, suponhamos que existe
a' tal que a.a' = 1 . Teríamos, então:
b = b . 1 = b .( a.a' ) = ( b . a ). a' = ( a . b ). a' = 0 . a' = 0 , uma contradição. 
129
CAPÍTULO 9
CONGRUÊNCIAS
Uma consequência imediata da proposição anterior é a seguinte:
Corolário 9.6: Seja p > 0 um inteiro primo. Então, todo elemento não-nulo de  p é
inversível.
Exemplo 9.3: Veja na “tabuada” de Z11 que todo elemento não nulo, possui um inverso:
*
1
2
3
4
5
6
7
8
9
10
1
1
2
3
4
5
6
7
8
9
10
2
2
4
6
8
10
1
3
5
7
9
3
3
6
9
1
4
7
10
2
5
8
4
4
8
1
5
9
2
6
10
3
7
5
5
10
4
9
3
8
2
7
1
6
6
6
1
7
2
8
3
9
4
10
5
7
7
3
10
6
2
9
5
1
8
4
8
8
5
2
10
7
4
1
9
6
3
9
9
7
5
3
1
10
8
6
4
2
10
10
9
8
7
6
5
4
3
2
1
9.7. Subtração em  m
A operação de subtração no sistema aritmético módulo m, tal como na aritmética comum, é
definida em termos de adição.
Na aritmética comum, podemos achar a resposta para 13 – 4, procurando o número que
somado a quatro nos dê 13. Como 4 + 9 = 13, o número 9 é a resposta do problema
13 – 4 = 9.
Você deve ter usado muito este processo ao conferir resultados de subtrações. Na verificação
adicionamos a diferença ao subtraendo para ver se a soma se iguala ao minuendo. Dizemos
que a subtração é a operação inversa da adição.
Usando os elementos mód. 7, resolva o problema de subtração 6 – 5 = ? Qual é o elemento
que somado a 5 resulta em 6?
Da tabela da adição mód. 7, 5 + 1 = 6. Portanto, podemos substituir 1 no lugar do ponto de
interrogação, e 6 – 5 = 1.
130
CAPÍTULO 9
CONGRUÊNCIAS
Na aritmética comum, não podemos subtrair 5 – 6 sem termos um resultado negativo. Os
números negativos não são necessários na subtração mód. m. No problema 5 – 6 = ?,
procuramos um elemento que somado a 6 nos dê 5. A tabela da adição nos revela que a soma
de 6 + 6 é 5. Portanto, 6 pode substituir o? e 5 – 6 = 6.
Você encontrará outros problemas de subtração nesse sistema que são bem diferentes e
interessantes. Considere o problema 4 – 5 = n, onde n é a resposta. Pela nossa definição de
subtração n + 5 deve ser 4. Qual é o número que somado a 5 em mód. 7 é 4? Da tabela da
adição, vemos que n = 6 e, portanto, 4 – 5 = 6.
Neste sistema finito, 1 - 4 = 4 – 1? No primeiro membro, qual é o elemento que somado a 4
dá 1? Como 4 + 4 = 1, 1 – 4 = 4. Do mesmo modo, temos que achar um número que somado a
1 resulta em 4. Como 1 + 3 = 4, 4 – 1 deve ser 3. Você vê, portanto, que 1 – 4 ≠ 4 – 1 e a
propriedade comutativa não é válida para a operação subtração.
A tabela abaixo resume o que foi dito.
7
-
0
1
2
3
4
5
6
0
0
1
2
3
4
5
6
1
6
0
1
2
3
4
5
2
5
6
0
1
2
3
4
3
4
5
6
0
1
2
3
4
3
4
5
6
0
1
2
5
2
3
4
5
6
0
1
6
1
2
3
4
5
6
0
9.8. Divisão em  m
Não é necessário usar frações se você trabalhar no sistema aritmético de módulos. Como é
isto possível? Vamos considerar a operação de divisão. Na divisão de números naturais,
verificamos as respostas, multiplicando o quociente pelo divisor para chegarmos ao
dividendo. Assim, 125 ÷25 = 5 pois 5 . 25 = 125.
A operação divisão, na aritmética módulo m, também pode ser definida como operação
inversa da multiplicação. Por exemplo, usando o mód.7, o problema 5 ÷ 4 é resolvido
encontrando um número n, tal que n . 4 = 5. Já que completamos a tabela de multiplicação
mód. 7 podemos usá-la para achar a resposta. Temos que, 3 . 4 = 5 e, portanto, 5 ÷ 4 = 3. Qual
é a resposta de 6 ÷ 3? Como 2 . 3 = 6, a resposta ao problema 6 ÷ 3 é 2. Neste sistema a
divisão, quando existe, é sempre exata.
131
CAPÍTULO 9
CONGRUÊNCIAS
Se, consideramos a expressão 3 ÷ 2 como sendo o mesmo que
3
1
e 1 ÷ 3 o mesmo que ,
2
3
teremos uma interessante vantagem na aritmética de módulos.
3
1
3÷2=5 e também
1÷ 3 = 5
2
3
As frações são rapidamente substituídas por algum elemento do sistema mód. 7. Uma
expressão tal
1
2
como 4 é simplificada de modo análogo.
1
3
3
A fração
1
é equivalente a 2, pois em mód. 7, 4 . 2 = 1.
4
Portanto, 2
1
1
é equivalente a 2 + 2, ou 4. O denominador, 3 , é igual a 1, pois
4
3
1
4
5 e 3 + 5 = 1 em mód. 7. Finalmente
1
3
4 , pois 1 . 4 = 4.
Considere o problema 2 ÷ 6 em aritmética mód. 7. Neste caso, temos que achar a resposta n,
tal que n . 6 = 2. Como 5 . 6 = 2, então 2 ÷ 6 = 5. Entretanto, se mudarmos o problema para 6
÷ 2 teremos uma resposta totalmente diferente. No caso de 6 ÷ 2 a resposta é 3, pois 3 . 2 = 6.
É evidente que 6 ÷ 2 não é igual
a 2 ÷ 6, portanto, a propriedade comutativa não se verifica
na divisão módulo m.
A tabela abaixo resume o que foi dito.
7
÷
1
2
3
4
5
6
1
1
2
3
4
5
6
2
4
1
5
2
6
3
3
5
3
1
6
4
2
4
2
4
6
1
3
5
5
3
6
2
5
1
4
6
6
5
4
3
2
1
Podemos resolver facilmente o problema 3.x = 0, usando a multiplicação de números naturais
da aritmética comum. Sabemos que o produto é 0; portanto, o numero que devemos colocar
no lugar de x deve ser o zero. Você já deve ter visto muitas vezes a afirmação de que se um
produto é zero, pelo menos um dos fatores deve ser igual a zero. Isto não é sempre verdade!
Pelo menos não é verdade em alguns sistemas aritméticos finitos.
É possível um problema de divisão ter mais de uma resposta? Você deve estar imaginando
que isto nunca é possível. Entretanto, no sistema aritmético módulo m isso pode ocorrer.
132
CAPÍTULO 9
CONGRUÊNCIAS
Consideremos sistema módulo 6. Neste sistema, 4 ÷ 4 = 1. Isto não deve ser estranho para
você, mas acontece que 4 ÷ 4 também é igual a 4 na aritmética mód. 6. Vamos explorar este
sistema finito mais detalhadamente.
Já vimos que os sistemas aritméticos módulo m, podem ser divididos em duas categorias: o de
módulo primos e não- primos.
Consideremos os sistemas aritméticos onde m é não-primo.
Vamos usar como exemplo o módulo 6. Na tabela completa de multiplicação abaixo, notamos
alguns padrões um pouco estranhos.
Quando a tabela é simétrica com relação a diagonal, dizemos que a operação é comutativa.
Entretanto, repare nos produtos resultantes da multiplicação por 2. Além do estranho
resultado de 2 . 3 ser igual a 0, apesar de nenhum fator ser 0. E nesta multiplicação só temos
como respostas o 0, 2 e 4.
*
1
2
3
4
5
1
1
2
3
4
5
2
2
4
0
2
4
3
3
0
3
0
3
4
4
2
0
4
2
5
5
4
3
2
1
Do mesmo modo, somente teremos respostas 0 ou 3 na multiplicação por 3. Estes fatos tornarse-ão mais significativos quando estudarmos a divisão neste sistema.
Alguma vez você trabalhou num problema de matemática durante tanto tempo, sem conseguir
resolvê-lo? Você deve ter começado a desconfiar que talvez o problema não tivesse solução.
Isto pode parecer estranho, mas alguns problemas em matemática não tem solução para as
condições dadas.
Isto acontece com a divisão no sistema da aritmética de módulos não-primos.
Seja calcular 5 ÷ 1(mod 6), quer dizer, um número tal que, quando multiplicado por 1 dá o
produto 5. Em termos gerais, a ÷ b = c ,tal que c . b = a . Podemos ver na tabela de
multiplicação que 1 . 5 = 5, portanto, 5 ÷ 1 = 5.
Deparamos com uma situação diferente no problema 4 ÷ 2. A tabela de multiplicação mostra
claramente que 2 . 2 = 4 e também 2 . 5 = 4. Portanto, ambos os elementos 2 e 5 satisfazem
aos requisitos para o quociente 4 ÷ 2.
Mas há mais! No problema 5 ÷ 3, procuramos um quociente n, tal que n . 3 = 5. Olhe a tabela
de multiplicação para módulo 6. Não há um número n tal que n . 3 = 5. O problema 5 ÷ 3 não
tem resposta neste sistema.
Verifique as respostas na tabela da divisão módulo 6 completa absixo. Relacione estas
respostas com a tabela de multiplicação desenvolvida anteriormente. A divisão por zero é
133
CAPÍTULO 9
CONGRUÊNCIAS
excluída. Na tabela abaixo, e nas demais, estaremos considerando cada um dos elementos da
primeira linha como o dividendo e cada um dos elementos da primeira coluna como o divisor.
÷
Dividendo
1
2
3
4
5
1
1
2
3
4
5
2
-
1; 4
-
2; 5
-
3
-
-
1; 3; 5
-
-
4
-
2; 5
-
1; 4
-
5
5
4
3
2
1
Divisor
Exemplo 9.4: Construa as tabelas de multiplicação e divisão módulo 8.
134
*
1
2
3
4
5
6
7
1
1
2
3
4
5
6
7
2
2
4
6
0
2
4
6
3
3
6
1
4
7
2
5
÷
1
2
3
4
5
6
7
1
1
3
5
7
2
2
1; 5
6
2
3; 7
6
3
3
1
7
5
4
4
0
4
0
4
0
4
4
4
2; 6
4
1; 3; 5;7
4
2; 6
4
5
5
2
7
4
1
6
3
6
6
4
2
0
6
4
2
5
5
7
1
3
6
6
3; 7
2
6
1; 5
2
7
7
6
5
4
3
2
1
7
7
5
3
1
CAPÍTULO 9
CONGRUÊNCIAS
9.9. Potenciação em  m
Exponenciação Rápida
Sejam a e n inteiros positivos. Temos que
k
n i 2i
n=
i=0
é a expansão binária de n. Os coeficientes n i são 0 ou 1. Portanto
k
a
n
n i 2i
a
k
i
i
(a 2 ) n i
=
i=0
i=0
a2
0 i ki ni 1
Com isto, desenvolvemos a seguinte ideia:
1.
2.
i
Calculamos os quadrados sucessivos de a 2 , 0 i k .
i
Determinamos a n como o produto daqueles a 2 para os quais ni =1 .
i+1
Observe que a 2
i
(a 2 )2
i+1
i
Portanto, a 2 pode ser calculado de a 2 por uma elevação ao quadrado.
Vejamos um exemplo: Seja calcular 144823 (mod1037) .
Primeiro façamos a expansão binária de 823:
823 = (1100110111)2
Escrevamos essa expansão na base dez:
(1100110111)2 = 29 + 28 + 25 + 24 + 22 + 21 + 20
(1100110111)2 = 512 + 256 + 32 + 16 + 4 + 2 + 1
Logo,
144823 = 144512 . 144256 . 14432 . 14416 . 1444 . 1442 . 1441
Agora, calculemos cada uma das potências por uma elevação sucessiva á potência 2.
144 144 (mod 1037)
1442 1033 (mod 1037)
1444 16 (mod 1037)
135
CAPÍTULO 9
CONGRUÊNCIAS
1448 256(mod 1037)
14416 205(mod 1037)
14432 545(mod 1037)
14464 443(mod 1037)
144128 256(mod 1037)
144256 205(mod 1037)
144512 545
Assim,
144823
545.205.545.205.16.1033.144
mod1037
= 766.766.117 mod1037
= 851.117 mod1037
= 99567 mod1037
= 15 mod1037
Conclusão: 144823 15 (mod 1037).
Voltaremos a falar deste assunto, depois de estudarmos o Algoritmo Chinês do Resto e o
Pequeno Teorema de Fermat.
Leitura: Teste de Primalidade Circular
O Teste de Primalidade Circular é, em termos gerais, semelhante à verificação tradicional de
primalidade de dividir um inteiro N > 1 pelos primos menores ou iguais ao piso da raiz
quadrada de N. A principal diferença entre o Teste Circular e o método tradicional é que,
neste caso, os divisores não são todos primos. Podemos também perceber uma diferença no
que diz respeito ao desempenho. O Teste Circular, também pode ser considerada um tipo
especial de crivo.
136
CAPÍTULO 9
CONGRUÊNCIAS
Procedimento
O primeiro passo consiste em escolher uma pequena quantidade dos primeiros primos.
Por exemplo, escolhe-se os primos 2, 3 e 5.
As classes de congruências que serão consideradas, terão módulo m igual ao produto desses
primos: m = 2 . 3 . 5 = 30
Em seguida, toma-se todos os inteiros positivos menores que 30 que não sejam múltiplos de
2, 3, e 5 ( relativamente primo com 30). Elaborando uma pequena tabela, e removendo os
múltiplos, obtém-se:
1
7
13
19
25
2
8
14
20
26
3
9
15
21
27
4
10
16
22
28
5
11
17
23
29
6
12
18
24
30
Os números que não foram marcados formam a base do crivo: 1, 7, 11, 13, 17, 19, 23, 29
(módulo 30). Cada um desses números indicará as classes de congruências módulo 30 que
serão utilizadas, ou seja,
1,7,11,13,17,19,23 e 29 .
Finalmente, semelhante ao método tradicional, para se verificar que um dado número N é
primo, divide-se N pelos primos iniciais, usados para gerar m, e pelos elementos das classes
de congruências (com exceção do 1) que sejam maiores que, ou iguais a N . Se N não for
divisível por nenhum desses termos, então N é primo.
Um exemplo prático
3331 = 57. Em seguida,
Para verificar se 3331 é primo, define-se o limite superior
divide-se 3331 pelos primos iniciais 2, 3, 5 e por todos os elementos das classes
1,7,11,13,17,19,23 e 29
(mod 30), que não ultrapassam o valor de 57, até se obter um
resto zero ou a divisão chegar ao último elemento das classes (mod 30) sem se obter uma
divisão exata, o que signficará que 3331 é primo.
Sejam as classes com seus respectivos elementos menores que 57:
1 {1,31,...}
7 {7,37,...}
11 {11, 41,...}
13 {11, 41,...}
17 {17, 47,...}
19 {19, 49,...}
23 {23,53,...}
29 {29,...} .
137
CAPÍTULO 9
CONGRUÊNCIAS
Assim, os elementos das classes, que não ultrapassam o valor 57, com exceção do 1, serão
usados para o teste de primalidade de 3331 e mais os primos 2, 3 e 5, ou seja, dividiremos
3331 pelos seguintes números :
2, 3, 5, 7, 11, 13, 17, 19, 23, 29, 31, 37, 41, 43, 47, 49, 53. Observe que nesta lista, diferente
do método tradicional, temos o número 49 que não é primo!
Como 3331 não divisível por nenhum desses números, então ele é primo.
Nota: É claro que um inteiro maior que 5, que é “candidato” a primo não
pode ser par e nem terminar em 5. Logo, a divisão por 2 e 5 devem ser
descartadas, bem como por 1. Qualquer candidato a primo,obrigatoriamente
deverá ser um ímpar que termine em 1 ou 3 ou 7 ou 9.
Uma forma indireta de crivo
É claro que o aparecimento de números compostos na sequências dos prováveis divisores de
N é uma mera consequência teórica. Antes que um N seja divisível por um composto, já terá
sido divisível por algum primo que seja um divisor deste composto. Uma vantagem deste
método é não ser preciso saber se um número é ou não primo para usá-lo como provável
divisor, como acontece no método tradicional.
Pelo Teorema de Dirichlet, existem infinitos primos em cada classe de congruência a
módulo m. O que podemos afirmar é que, na sequência dos números a serem usados, se
houver divisores de N, o menor deles é necessariamente um primo de uma das classes de
congruências. Isso nos dá um método indireto para saber se um número é primo, ou seja, uma
forma indireta de crivo.
Vamos exemplificar: Queremos saber se N = 3127 é composto. Se existirem números que
dividam N em alguma das classes a serem usadas, então o menor desses divisores é primo.
De fato, seja 3127 55 . Consideremos m = 2.3 = 6, logo as classes módulo 6 são:
1={7,13,19,25,31,37,43,49,55}
5={11,17,23,29,35,41,47,53}
Da classe 5 , temos que 53 é o menor divisor de 3127, logo 53 é primo.
138
CAPÍTULO 9
CONGRUÊNCIAS
EXERCÍCIOS
1) Verdadeiro (V) ou falso (F)
a) 91 0 (mod.7).
b) 3 + 5 + 7 5 (mod.10).
c) –2 2 (mod.8).
d) 112 1 (mod.3).
e) 17
9 (mod.2).
f)
42
b)
c)
d)
e)
f)
-8 (mod.10).
2) Verdadeiro (V) ou falso (F)
a) x 3 (mod.5)
x { .... –7, -2, 3, 8,
13 .... }.
b) 5 -1 (mod.6) e -1 -7 (mod.6)
5 -7 (mod.6).
3) Achar o menor inteiro positivo que represente a
soma:
a) 5 + 3 + 2 + 1 + 8 (mod. 7)
b) 2 + 3 – 1 + 7 – 2 (mod.4)
4) Sabendo-se que 1766 1066 (mod. m), achar
todos os possíveis valores do módulo m.
5) Exprimir que “n é ímpar” de três outras
maneiras.
6) Achar todos os inteiros x tais que 0 < x < 15 e
3x 6 (mod. 15)
7) Achar todos os inteiros x tais que 1 < x < 100 e
x 7 (mod. 17)
8) Sabendo-se que k 1 (mod. 4), mostrar que
6k + 5 3 (mod. 4)
9) Mostrar, mediante um exemplo, que a2
(mod.m) não implica a b (mod.m).
b2
4165 por 7
25100 + 11500 por 3.
35555 por 80.
51000 por 126.
3399300 + 29 por 13
15) Mostrar que
a) 89 | 244 – 1.
b) 97 | (248 – 1)
c) 13| 270 + 370
d) n| 1n 2n ... (n 1)n ,, n ímpar.
16) Demonstrar que, se a b (mod. m) então
mdc(a, m) = mdc(b, m).
17) Mostrar, mediante um exemplo, que ak
(mod. m) e k j não implica aj bj.
bk
18) Demonstrar as seguintes proposições:
a) Se a é um inteiro ímpar então a2 1 (mod.
8)
b) Se a é um inteiro qualquer, então a3 0, 1
ou 8 (mod. 9).
c) Se a é um inteiro qualquer, então a3 a
(mod. 6).
19) Mostre que 7 divide 22225555+55552222.
20) Determine o resto da divisão de 61987 por 37.
21) Prove que 7 divide 32n+1+2n+2 para todo
natural n.
77
22) Determine o algarismo das unidades de 7 .
23) Determine os infinitos valores positivos de n
tal que 2n +27 seja divisível por 7.
10) Mostrar que todo primo ímpar é congruente
módulo 4 a 1 ou 3.
24) Prove que 2k −5, k 4 nunca deixa resto 1
quando dividido por 7.
11) Mostrar que todo primo maior que 3 é
congruente módulo 6 a 1 ou 5.
25) Prove que 4 divide 12233 . 455679+87653 3
26) Prove que 11. 31. 61 | 2015 - 1.
12) Mostrar que
a) 1110 1 (mod 100)
b) 31000 + 3 é divisível por 28.
13) Mostrar que 41 divide 220 – 1.
27) Calcular o resto da divisão do
n = (116 + 1717)21 por 8.
inteiro
28) Calcular o resto da divisão do inteiro
7100 + 11100 por 13
14) Achar os restos das divisões de:
a) 250 por 7
139
CAPÍTULO 9
CONGRUÊNCIAS
29) Mostrar que o inteiro n = 13
divisível por 3.
16
43
–2 .5
17
é
45) Determine para que inteiros positivos n, 2n –1 é
divisível por 7.
30) Mostrar que a expressão 3.5²n+1 + 2³n+1 é
divisível por 17, sendo n um inteiro positivo
qualquer.
46) Prove que 2n + 1 nunca é divisível por 7, para
qualquer inteiro positivo n.
31) Mostrar que o número de 2³² + 1 é divisível
por 641.
47) Demonstre que para todo inteiro n
um divisor de 34n + 2 + 26n + 3.
32) Demonstrar que, se o inteiro positivo n não é
divisível por 4, então a soma: S = 1n + 2n + 3n +
4n é divisível por 5.
33) Determinar todos os inteiros positivos n para os
quais 2n + 1 é divisível por 3.
34) Mostrar que 63!
35) Mostrar que não existe inteiro algum n que
verifique as condições: n 5 (mod 12) e n
(mod 15)
4
36) Na divisão do inteiro n por 10 o quociente é q e
o resto é r, tais que 2r – q
0 (mod 7).
Mostrar que 7 n.
37) Determinar quais dos seguintes conjuntos são
sistemas completos de restos módulo 4.
a) { -2, -1, 0, 1}
b) {0, 4, 8, 12}
c) { -13, 4, 17, 18 }
d) {–5, 0, 6, 22 }
38) Determinar quais dos seguintes conjuntos são
sistemas completos de restos módulo 6.
a) { 1, 2, 3, 4, 5}
b) {0, 5, 10, 15, 20, 25}
c) {-4, -3, -2, -1, 0, 1}
d) {17, -4, 6, 7, 10, 3}
39) Achar um sistema completo de restos {p1, p2,
...pi, ..., p7} módulo 7, tal que todo pi é primo.
40) Achar um sistema completo de restos módulo 7
formado só de múltiplos não negativos de 4.
41) Se n é um múltiplo positivo de 4, qual o resto
da divisão de 1n + 2n + ... + 8n + 9n por 10?
42) Mostre que 121n – 25n + 1900n – (–4)n é
divisível por 2000, para todo natural n..
43) Determinar o algarismo das unidades do
número 3100.
44) Que valores de a e b tornam o número
30a0b03 divisível por 13?
140
48) Determine o algarismo das dezenas do número
7999999.
49) Determine o resto da divisão por 7 do número
2
3
100
1010 1010 1010 ... 1010 .
50) Mostre que, para todo
102n + 1 + 1 0 (mod 11).
61! (mod 7)
0 , 17 é
n natural, temos
CAPÍTULO 10
FERMAT, WILSON E EULER
Capítulo 10
TEOREMAS DE FERMAT,
WILSON e EULER
Fermat foi um dos poucos matemáticos amadores famosos. Filho de um rico comerciante de
couro, pôde se dedicar completamente aos estudos. Por influência de sua mãe, descendente de
uma família de juristas, estudou leis na Universidade de Orleans e formou-se em advocacia.
Trabalhou durante toda sua vida na corte de justiça de Toulouse. Foi nomeado juiz e ocupava
os seus momentos de folga em diversos lazeres, entre os quais a poesia e a Matemática.
Seu interesse pela matemática iniciou-se em 1629 com o estudo dos trabalhos de Apolônio
(matemático grego, 260 A.D.) sobre curvas planas. Trocava correspondência com os maiores
matemáticos da época, como Torricelli, Roberval, Huyghens e Pascal, e, dessa forma relatava
suas descobertas. Jamais publicou seus trabalhos de nenhuma outra forma, mas o conteúdo
das cartas de Fermat é atualmente incluído em todos os textos usuais de teoria dos números.
Seu interesse na teoria dos números surgiu após ler o livro Aritmética de Diofanto
(matemático grego, 200 A.C.) e alguns dos problemas propostos por Fermat, nesta área, eram
tão difíceis que somente muitos anos mais tarde foram provados. Um desses problemas
afirmava que "todo número inteiro pode ser escrito como a soma de no máximo quatro
quadrados" e foi provado em 1770, pelo matemático francês Lagrange. Entretanto, seu
resultado mais famoso resistiu por mais de 350 anos e inspirou a publicação, em 1996, do
bestseller O Último Teorema de Fermat. Este teorema diz que “se n é um natural maior que 2,
então não existem números inteiros x, y e z que satisfaçam a equação xn + yn = zn”. Isto foi
provado definitivamente, em 1994, pelo matemático inglês Andrew Wiles (repare que no caso
n = 2 o teorema é satisfeito por todos os ternos pitagóricos, isto é, por inteiros que satisfaçam
o Teorema de Pitágoras).
10.1.
Teorema 10.1 (“Pequeno Teorema de Fermat” – PTF): se p é primo e se o MDC(p, a) = 1,
então:
ap–1
1 (mod p)
Demonstração: consideremos os (p – 1) primeiros positivos de s, isto é, os inteiros
141
CAPÍTULO 10
FERMAT, WILSON E EULER
a, 2 . a, 3 . a, ..., (p – 1) . a
Obviamente, nenhum desses (p – 1) inteiros é divisível por p e, além disso, dois quaisquer
deles
são
incongruentes
módulo
p,
pois,
se
fosse:
r.a
s.a (mod p), 1
p–1
r<s
então, o fator comum a poderia ser cancelado, visto que o MDC(a, p) = 1, e teríamos:
r
s (mod p), isto é p | (a – x)
o que é impossível, porque 0 < s – r < p.
Assim sendo, dois quaisquer dos (p – 1) inteiros a, 2.a, 3.a, ..., (p – 1).a divididos por p
deixam restos distintos, e por conseguinte cada um desses p – 1 inteiros é congruente módulo
p a um único dos inteiros 1, 2, 3, ..., p – 1, naturalmente numa certa ordem, multiplicando
ordenadamente essas p – 1 congruências, teremos:
a . 2a . 3a . ... . (p – 1)a
1 . 2 . 3 . ... . (p – 1) (mod p)
ou seja,
ap–1 (p – 1)!
(p – 1)! (mod p)
Como o MDC (p, (p – 1)!) = 1, porque p é primo e p não divide (p – 1)!, podemos cancelar o
fator (p – 1)!, o que dá a congruência de Fermat:
ap–1
1 (mod p)
Exemplo 10.1: seja o primo p = 7 e o inteiro a = 3 tais que 7 não divide 3, temos os p – 1 = 6
primeiros múltiplos positivos de 3: 3, 6, 9, 12, 15, 18. Nenhum desses 6 inteiros é divisível
por 7, todos são incongruentes módulo 7, e cada um deles é congruente módulo 7 a um único
dos inteiros 1, 2, 3, 4, 5, 6: 3 3 (mod 7), 6 6 (mod 7), 9 2 (mod 7), 12 5 (mod 7), 15
1 (mod 7), 18 4 (mod 7). Multiplicando ordenadamente essa 6 congruências, temos:
3 . 6 . 9 . 12 . 15 . 18
3 . 6 . 2 . 5 . 1 . 4 (mod p)
ou seja,
36 . 6!
6! (mod 7)
Como o MDC(7, 6!) = 1, podemos cancelar o fator comum 6!, que resulta em:
142
CAPÍTULO 10
FERMAT, WILSON E EULER
36
Corolário 1: se p é um primo, então ap
Demonstração: se p divide a, então a
1 (mod 7)
a (mod p), qualquer que seja o inteiro a.
0 (mod p) e ap
ap
0 (mod p), que implica em:
a (mod p)
Se, ao invés disto, p não dividisse a, então pelo PTF:
ap–1
1 (mod p), e ap
a (mod p)
Teorema 10.2: se p e q são primos distintos tais que ap
apq
a (mod q) e aq
a (mod p), então:
a (mod p.q)
Demonstração: pelo corolário 1 e da hipótese, temos:
(aq)p
(ap)q
aq (mod p)
ap (mod q)
aq
aq
a (mod p)
a (mod q)
apq
apq
a (mod p)
a (mod q)
Portanto, p | (apq – a), e q | (apq – a), que implica em:
(p.q) | (apq – a), isto é, apq
a (mod p.q)
Pelo corolário 1, se p é primo, então 2p 2 (mod p), isto é, p | (2p – 2).
Entretanto, a recíproca “se n | (2n – 2), então n é primo” não é uma proposição verdadeira, e os
inteiros positivos ímpares compostos que satisfazem essa condição são chamados de
pseudoprimos.
Por exemplo, 2341 2 (mod 341), isto é, 341 | (2341 – 2), onde o inteiro 341 = 11.31 é
composto. Com efeito, temos:
210 = 1024 = 3.11.31 + 1
que implica em
210
1 (mod 11), e 210
1 (mod 31)
Portanto,
211
2 (mod 31), e 231
2.(210)3 (mod 11)
2.(1)3 (mod 11)
2 (mod 11)
143
CAPÍTULO 10
FERMAT, WILSON E EULER
Como os inteiros 11 e 31 são primos distintos, temos, de acordo com o Teorema 2:
211.31
2 (mod 11.31), isto é, 2341
2 (mod 341), ou que 2340
1 (mod 341)
Congruência que mostra ser falsa a recíproca do PTF.
Logo, 341 é um pseudoprimo na base 2.
Definição 1 (Pseudoprimos): todo inteiro positivo composto n tal que 2n
chama um pseudoprimo para a base 2.
2 (mod n) se
Há um número infinito de pseudoprimos para a base 2, mas somente sete são menores que
2000, e todos ímpares:
341 = 11 . 31
645 = 3 . 5 . 43
1387 = 16 . 73
561 = 3 . 11 . 17 1105 = 5 . 13 . 17 1729 = 7 . 13 . 19
Nota: O primeiro exemplo de um pseudoprimo par para a base 2 foi dado
pelo matemático americano LEHMER em 1950. O menor deles é 161038 = 2
. 73 .1103 e, em 1951, BEEGER mostrou a existência da infinidade de
pseudoprimos pares para a base 2.
Teorema 10.3: seja um inteiro a > 1 e um primo p > 2.
n
a2p 1
a2 1
ap 1
a 1
ap 1
a
1
é um pseudoprimo na base a.
Demonstração: pelo PTF,
ap 1
a 1
ap 1
(mod p)
a
1
1 (mod p)
e se verifica facilmente que estes números são ímpares, de onde n 1 (mod 2.p), ou
n = 2.k.p + 1, para um k inteiro. Assim, como a2p 1 (mod n), temos:
an
a2kp+1 (mod n)
(a2p)k . a (mod n)
Existe uma infinidade de pseudoprimos de base a > 1.
Façamos a demonstração no caso de a = 2.
144
a (mod n)
CAPÍTULO 10
FERMAT, WILSON E EULER
Proposição 1: se n é um pseudoprimo de base 2, então 2n – 1 também é um pseudoprimo de
base 2.
Demonstração: dada a igualdade (xt – 1) = (x – 1).(x(t–1) + x(t–2) + ... + x2 + x + 1), sejam d, k
tais que n = d.k, d > 1, k < n, e seja m = 2n – 1. Então m é composto.Para isto, basta
considerar na igualdade acima x = 2d e t = k. Por hipótese 2n–1 1 (mod n) e, portanto, existe
k > 0 tal que 2n–1 – 1 = k.n. Assim,
2m
1
1
n
2(2
2)
1
n 1
22.(2
1)
1
22km
1
Voltando à igualdade acima e fazendo x = 2n e t = 2.k, concluímos que 2m–1 – 1 é um múltiplo
de m = 2n – 1, ou seja 2m–1 1 (mod m)
2m 2 (mod m). Concluímos assim que m é um
pseudoprimo de base 2.
Corolário 2: existe uma infinidade de pseudoprimos de base 2.
Demonstração: basta notar que 341 é um pseudoprimo de base 2 e aplicar sucessivamente a
proposição anterior.
Nota: O fato de existir uma infinidade de pseudoprimos em qualquer base
não implica a existência de uma infinidade de pseudoprimos.
EXERCÍCIOS
1) Verificar o PTF com a = 2 e p = 13.
7)
Sendo a um inteiro, mostrar que n5 e n têm o
mesmo algarismo das unidades.
2) Verificar o PTF com a = 3 e p = 17.
8) Demonstrar que 13 | (270 + 370) através do PTF.
3) Verificar utilizando o PTF:
a) 1850 2 (mod 7)
b) (b) 19933 8 (mod 31)
9) Demonstra que 22225555 + 55552222 é divisível
por 7 .
4) Mostrar que 538
10) Achar o resto da divisão de 21137 por 17 com a
ajuda do PTF.
5)
4 (mod 11) pelo PTF.
Mostrar que o inteiro 117 é composto usando
o PTF.
6) Achar o algarismo das unidades do inteiro 3 400
com o auxílio do PTF.
11) Mostrar que, se o MDC(a, 35) = 1,
a12 1 (mod 35).
então
12) Demonstrar que, para todo inteiro a, se tem:
a) a13 a (mod 7)
b) a37 a (mod 13)
145
CAPÍTULO 10
FERMAT, WILSON E EULER
c) a21 a (mod 15)
d) a7 a (mod 42)
19) Mostrar que 561 é um pseudoprimo para a base
2, 3 e 5.
13) Demonstrar que, para todo inteiro positivo n, se
tem:
a) 22n 1 (mod 3)
b) 23n 1 (mod 7)
20) Demonstrar que se 7 não divide n,
n6 1996 (mod 7).
então
21) Demonstrar que se MDC(n, 7) = 1
7 | (n12 – 1).
então,
14) Achar todos os primos p tais que p | (2p + 1).
22) Demonstrar que se MDC(a, 240) = 1, então
240 é um divisor de a4 – 1.
15) Demonstrar que 1105 | (31105 – 3) [sugestão:
PTF].
23) Demonstrar que todo primo maior que 5 divide
um inteiro formado só de algarismos 1.
16) Demonstrar que 161038 é um pseudoprimo
para a base 2, i.e., que 161038 | (2161038 – 2).
24) Demonstrar que para todo inteiro positivo n,
temos que (n3 – n).(58n+4 + 34n+2) é um múltiplo
de 3804.
17) Mostrar que 2047 é um pseudoprimo para a
base 2.
5
25) Prove que 15 divide 3n
18) Mostre que 341 não é um pseudoprimo para a
base 3.
5n3
7n .
26) Prove que 91 divide a12 b12 , onde a e b são
relativamente primo com 91.
10.2. TEOREMA DE WILSON
Teorema 10.4 (Teorema de Wilson): se p é um primo, então (p – 1)!
–1 (mod p).
Demonstração: o teorema é verdadeiro para p = 2 e para p = 3, pois:
(2 – 1)! = 1! = 1
(3 – 1)! = 2! = 2
–1 (mod 2)
–1 (mod 3)
de modo que vamos supor p 5. Consideremos a congruência linear a.x 1 (mod p), onde a é
um dos (p – 1) primeiros inteiros positivos 1, 2, 3,..., p – 1 de modo que o MDC(a, p) = 1.
Nestas condições, existe um único inteiro positivo a’, com 1
a.a’
a’
p – 1, tal que
1 (mod p)
Como p é primo, tem-se que a = a’ se e somente se a = 1, ou a = p – 1, visto que
a2 1 (mod p) implica em (a – 1).(a + 1) 0 (mod p) e, portanto,
a–1
146
0 (mod p), ou (a + 1)
0 (mod p)
CAPÍTULO 10
FERMAT, WILSON E EULER
isto é, a = 1 ou a = p – 1.
Nota: Este teorema foi descoberto primeiramente por John Wilson (1741 - 1793), estudante
do matemático inglês Edward Waring. Waring anunciou o teorema em 1770, embora nenhum
deles tenha conseguindo prová-lo. Lagrange deu a primeira prova em 1773.
Há uma evidência que Leibniz estava ciente do resultado um século antes, mas nunca o
publicou.
Exemplo 10.2: Com p = 13, existe um único inteiro positivo a’, com 1
a
a'
1
1
2
7
3
9
a.a’
1 (mod p)
5
8
6
11
4
10
7
2
8
5
9
3
a’
p – 1, tal que
10
4
11
6
12
12
p 3
2
1 (mod p). Então, multiplicando ordenadamente todas
Omitindo os inteiros 1 e p – 1, com os p – 3 restantes: 2, 3, ..., p – 2 podemos formar
pares (a, a’), com a a’ tais que a.a’
p 3
essas
congruências, obtemos a congruência:
2
2 . 3 . ... . (p – 2) 1 (mod p), ou (p – 2)!
1 (mod p)
ou, multiplicando por (p – 1):
(p – 1)!
(p – 1) (mod p)
–1 (mod p)
que é a própria congruência de Wilson.
Com p = 13, por exemplo, os 10 inteiros 2, 3,..., 11 dão lugar a 5 pares tais que o produto dos
inteiros de cada par é congruente a 1 módulo 13:
2.7 1 (mod 13)
4.10 1 (mod 13)
6.11 1 (mod 13)
3.9 1 (mod 13)
5.8 1 (mod 13)
multiplicando ordenadamente essas cinco congruências, obtemos:
(2.7) . (3.9) . (4.10) . (5.8) . (6.11)
11! (mod 13)
1 (mod 13)
e, portanto,
147
CAPÍTULO 10
FERMAT, WILSON E EULER
12!
12 (mod 13)
–1 (mod 13)
isto é,
(p – 1)!
–1 (mod p), com p = 13
Recíproca do Teorema de Wilson: “se (n – 1)!
–1 (mod p), então n é primo”.
Demonstração: a recíproca é verdadeira. Suponhamos, por absurdo, que o inteiro n é
composto. Então, n tem um divisor d tal que 1 < d < n, de modo que d é um dos fatores do
produto:
1 . 2 . 3 . ... . (n – 1) = (n – 1)!
e, portanto, d | (n – 1)!. Mas, por hipótese n | (n – 1)! + 1, e como d | n, segue-se que:
d | (n – 1)! + 1, e d | 1
o que é absurdo, visto que d > 1. Logo, n não possui divisores menores que ele mesmo e
diferentes de 1, ou seja, n é primo.
Nota: Este teorema recíproco dá um critério para se reconhecer se um inteiro dado é primo.
Mas, do ponto de vista prático, no momento ainda é de interesse apenas teórico, pois o cálculo
de fatoriais exigiria muito tempo.
Teorema 10.5 (Teorema de Leibniz): um inteiro n > 1 é primo, se e somente se,
(n – 2)!
1 (mod n)
Demonstração:
( ) suponhamos que o inteiro n > 1 é primo. Então, pelo teorema de Wilson:
(n – 1)!
–1 (mod n)
obviamente,
(n – 1)! = (n – 1). (n – 2)!
– (n – 2)! (mod n)
portanto,
(n – 2)!
( ) reciprocamente, se (n – 2)!
1 (mod n)
1 (mod n), então:
(n – 1)!
– (n – 2)!
–1 (mod n)
Logo, pelo recíproco do Teorema de Wilson, o inteiro n é primo.
148
CAPÍTULO 10
FERMAT, WILSON E EULER
EXERCÍCIOS
1) Verificar o Teorema de Wilson para p = 5 e para p = 7.
2) Mostrar que 11, 13, 17 e 19 são primos usando o Teorema de Wilson.
3) Mostrar que 8 é composto usando o Teorema de Wilson.
4) Achar o resto da divisão de 15! por 17.
5) Mostrar que 18! + 1
0 (mod 437).
6) Sendo p um primo ímpar, demonstrar que 2.(p – 3)!
–1 (mod p).
7) Verificar o Teorema de Leibniz com o primo p = 13.
8) Usando o Teorema de Leibniz, mostrar que 17 é primo.
9) Formar com os inteiros 2, 3, 4, ... , 21, todos os pares de a e b tais que a.b
1 (mod 23).
149
CAPÍTULO 10
FERMAT, WILSON E EULER
10.3. TEOREMA DE EULER
Leonhard Euler (15/04/1707 – 18/07/1783) foi um matemático e físico de origem suíça.
Nasceu na Basiléia, filho do pastor calvinista Paul Euler (lê-se “óiler”) que, desprezando seu
prodigioso talento matemático, determinou que ele estudasse Teologia e seguiria a carreira
religiosa. Daniel e Nikolaus Bernoulli convenceram o pai de Euler a permitir que seu filho
trocasse o hábito pelos números.
Euler, logo após, deixou a Suíça, indo para os palácios de Berlim e São Petersburgo, onde
passou os mais criativos anos de sua vida. Os governos da Europa estavam interessados em
usar a Matemática para resolver problemas práticos e competiam entre si para empregar os
melhores cérebros.
Durante sua vida resolveu enorme quantidade de problemas, da navegação às finanças, da
acústica à irrigação. A solução de tais problemas, que atendiam aos reclamos do mundo
prático, não o entediava, principalmente porque cada novo trabalho inspirava-o para criar uma
Matemática nova e engenhosa. Era capaz de escrever vários trabalhos em um único dia com
os cálculos completos e prontos para serem publicados.
Conseguiu provar o uma conjectura de Fermat relativa aos números primos. Fermat afirmava
que o primeiro tipo de número primo sempre era representado pela soma de números ao
quadrado enquanto que o segundo jamais o seria. Esta propriedade dos números primos é
extremamente simples, porém, ao tentar provar que isto é uma verdade para qualquer número
primo, torna-se extremamente difícil. Em 1749, depois de sete anos de trabalho e quase cem
anos após a morte de Fermat, conseguiu apresentar esta prova.
Ao final de vida estava completamente cego. Aliás, já era cego de um olho desde os vinte
anos, o que não o perturbara em nada. Certa vez disse: “agora eu terei menos distrações”. Aos
sessenta anos foi acometido por catarata que o cegou completamente. Apesar disto, continuou
a produzir Matemática por mais sete anos. Seu imenso conhecimento permitia-lhe criar
conceitos sem ter que colocá-los no papel e sua memória fenomenal permitia-lhe usar seu
cérebro como uma biblioteca mental.
Passou os anos finais de sua vida na Rússia, então sob a proteção de Catarina, a Grande.
Definição 2: chama-se função aritmética toda função f definida no conjunto  dos naturais e
com valores no conjunto  dos inteiros, i.e., toda função f de  em  (f : 
) .
Definição 3: uma função aritmética f se diz multiplicativa se f(r.s) = f(r) . f(s), para todo par
de inteiros positivos r e s, tais que o MDC(r, s) = 1.
150
CAPÍTULO 10
FERMAT, WILSON E EULER
10.4. FUNÇÃO TOTIENT (n)
Definição 4: Chama-se Função Totient a função aritmética assim definida para todo inteiro
positivo n:
(n) = quantidade de inteiros positivos menores que n relativamente primos a n.
Em outros termos, (n) é igual ao número de elementos do conjunto
#{ x

|1
x < n, MDC(x, n) = 1 }
Observação: (1) = 1, pois MDC(1, 1) = 1.
Exemplo 10.3: (30) = 8 e (12) = 4.
n
(n)
1
1
2
3
4
5
6
7
8
1
2
2
4
2
6
4
Tabela de (n) para os dez primeiros inteiros positivos.
9
6
10
4
Teorema 10.6: A Função Totient é uma função aritmética multiplicativa.
Demonstração: sejam r e s dois inteiros positivos tais que o MDC(r, s) = 1.
Cumpre demonstrar que (r.s) = (r) . (s).
A proposição é verdadeira se r ou s é igual a 1, pois, temos:
Se r = 1, (r.s) = (1.s) = (s) = 1 . (s) = (1) . (s) = (r) . (s)
Se s = 1, (r.s) = (r.1) = (r) = (r) . 1 = (r) . (1) = (r) . (s)
Suponhamos, pois, que r > 1 e s >1. Neste caso, todos os inteiros de 1 a r.s podem ser
dispostos em r colunas com s inteiros em cada uma delas, do seguinte modo:
1
2
r+1
r+2
2.r + 1
2.r + 2
...
...
(s – 1).r + 1 (s – 1).r + 2
...
h
...
r+h
...
2.r + h
...
...
... (s – 1).r + h
... r
... 2.r
... 3.r
... ...
... s.r
Como o MDC(q.r + h, r) = MDC(h, r), os inteiros da h-ésima coluna são primos com r, se e
somente se, h é primo com r. Além disto, como na primeira linha o número de inteiros que são
primos com r é igual a (r), segue que há somente (r) colunas formadas com inteiros que são
todos os primos com r. Por outro lado, em cada uma destas (r) colunas existe precisamente
(s) inteiros que são primos com s, porque na progressão aritmética:
h, r + h, 2.r + h, ..., (s – 1).r + h
151
CAPÍTULO 10
FERMAT, WILSON E EULER
onde o MDC(r, h) = 1, o número de termos que são primos com s é igual a (s).
Assim sendo, o número total de inteiros que são primos com r e s, isto é, que são primos a r.s,
é igual a (r). (s), e isto significa que (r.s) = (r) . (s).
10.5 – CÁLCULO DE (n)
Teorema 10.7: seja p um primo, então (p) = p – 1.
Demonstração: ( ) Se n > 1 é primo, então cada um dos inteiros positivos menores que n é
primo com n e, portanto, (n) = n – 1. ( ) Reciprocamente, se (n) = n – 1, com n > 1, então
n é primo, pois, se n fosse composto, teria pelo menos um divisor d tal que 1 < d < n, de modo
que pelo menos dois dos inteiros 1, 2, 3,..., n não seriam primos com n, d e n, isto é, (n) n –
2. Logo, n é primo.
Teorema 10.8: se p é primo e se k é um inteiro positivo, então
(pk) = pk – pk–1 = pk . 1
1
p
Demonstração: obviamente, o MDC(pk, n) = 1, se e somente se, p não divide n, e entre 1 e pk
existem pk–1 inteiros que não são primos com pk, que são todos os múltiplos de p:
p, 2.p, 3.p, ..., p2, ..., p3, ..., pk
segue-se que o conjunto {p, 2p, 3p, ..., p2, ..., p3, ..., pk} contém exatamente pk – pk–1 inteiros
que são relativamente primos a pk, de modo que pela definição da função (n) de Euler,
temos:
(pk) = pk – pk–1
Teorema 10.9: se n = p1k . pk2 . pk3 . ... . prk é a decomposição canônica do inteiro n > 1,
então:
1
(n) = p1k1 p1k1
1
. pk22
3
2
pk22
1
r
. ... . pkr r
pkr r
1
n. 1
1
1
1
. 1
. ... . 1
p1
p2
pr
ou seja,
r
(n) =
152
( piki
i 1
piki 1 )
r
n. (1
i 1
1
)
pi
CAPÍTULO 10
FERMAT, WILSON E EULER
Demonstração: usaremos o Teorema da Indução Matemática sobre r, número de fatores
primos distintos de n. A proposição é verdadeira para r = 1.
Suponhamos, então, a proposição verdadeira para r = i.
kr+1
Como o MDC p1k1 . pk22 . ... . prkr , pr+1
1, e (n) é uma função aritmética multiplicativa,
temos:
p1k1 . pk22 . ... . prkr
p1k1 . pk22 . ... . prkr
kr+1
. pr+1
r+1
pkr+1
.
ou seja,
p1k1 . pk22 . ... . prkr
p1k1 . pk22 . ... . prkr
kr+1
. pr+1
r+1
. pkr+1
r+1
pkr+1
1
ou, à vista da hipótese de indução:
kr+1
p1k1 . pk22 . ... . prkr . pr+1
p1k1
p1k1
1
. pk22
pk22
1
. ... . pkr r
pkr r
1
r+1
. pkr+1
r+1
pkr+1
1
e isto significa que a proposição é verdadeira para r = i + 1. Logo, a proposição é verdadeira
para todo inteiro positivo r.
Teorema 10.10: Para todo inteiro positivo n > 2 ,
Demonstração:
Se n é uma potência de 2, isto é, n
(n)
2 k , com k
(n) é um inteiro par
2 , então:
(2k ) 2k (1
1
) 2k
2
1
que é um inteiro par.
Se, ao invés, n não é uma potência de 2, então n é divisível por um primo impar p, isto é:
n p k m , onde k 1 e o mdc( pk , m) 1
E como (n) é uma função aritmética multiplicativa temos:
(n)
( pk ) (m)
pk 1( p 1) (m)
que é também um inteiro par, por que 2 | ( p 1) .
Assim, (n) é um inteiro ímpar somente para n = 1 e n = 2:
(1)
(2) 1
153
CAPÍTULO 10
FERMAT, WILSON E EULER
Teorema 10.11 (Teorema de Euler): Se n é um inteiro positivo e se MDC(a, n) = 1, então
a
(n)
1 (mod n)
Provaremos, primeiramente, o seguinte lema:
“Sejam a e n > 1 inteiros tais que o MDC(a, n) = 1. Se a1, a2, ..., a (n) são inteiros positivos
menores que n e que são relativamente primos com n, então cada um dos inteiros a.a1, a.a2, ...,
a.a (n) é congruente módulo n a um dos inteiros a1, a2, ..., a (n) (não necessariamente nesta
ordem em que aparecem).”
Demonstração: os inteiros a.a1, a.a2, ..., a.a (n) são mutuamente incongruentes módulo n, pois,
se a.ai a.aj (mod n), com 1 i j
(n), como o MDC(a, n) = 1, podemos cancelar o fator
comum a, o que dá ai aj (mod n)
n | (aj – ai). Isto é impossível, visto que (aj – ai) < n.
Por outro lado, como o MDC(ai, n) = i, i = 1, 2, ..., (n) e o MDC(a, n) = 1, segue que o
MDC(a.ai, n) = 1. Mas, pelo algoritmo da divisão, a.ai = n.qi + ri, 0 ri < n, que implica em
a.ai
ri (mod n), com 0
ri < n
portanto, MDC(ri, n) = MDC(a.ai, n) = 1, de modo que ri é um dos inteiros a1, a2, ..., a (n), isto
é, cada um dos inteiros a.a1, a.a2, ..., a.a (n) é congruente módulo n a um único dos inteiros a1,
a2, ..., a (n), em uma certa ordem.
Provemos, agora, o Teorema de Euler:
A proposição é verdadeira para n = 1, pois a (1) 1 (mod 1). Suponhamos, pois, n > 1, e sejam
a1, a2, ..., a (n) os inteiros positivos menores que n e relativamente primos a n.
Como o MDC(a, n) = 1, então, pelo Lema acima, os inteiros a.a1, a.a2, ..., a.a (n) são
congruentes módulo n aos inteiros a1, a2, ..., a (n), em uma certa ordem:
a.a1
onde a1*, a2*, ..., a
*
(n)
a1*, a.a2
a2*, ..., a.a
denotam os inteiros a1, a2, ..., a
(n)
(n)
a
*
(n)
em uma certa ordem.
Multiplicando ordenadamente todas essas (n) congruências, obtemos:
(a.a1).(a.a2). ... .(a. a
(n))
a1* . a2* . ... . a
*
(n)
(mod n)
ou seja,
a
(n)
. (a1 . a2 . ... . a
(n))
a1 . a2 . ... . a
(n)
(mod n)
Cada um dos inteiros a1, a2, ..., a (n) é relativamente primo a n, de modo que podem ser
sucessivamente cancelados, o que dá a congruência de Euler:
a
(n)
1 (mod n)
Nota: se p é um primo, (p) = p – 1, e se o MDC(a, p) = 1, então:
a
154
(p)
ap–1 (mod p)
1 (mod p)
CAPÍTULO 10
FERMAT, WILSON E EULER
que é a congruência de Fermat. Assim, o Teorema de Euler é uma generalização do teorema
de Fermat.
Corolário 3: Se m > 1, k 0, n 0 e a um inteiro qualquer são tais que, MDC(a, m) = 1 e
k n (mod (m)) então, ak an (mod m).
Demonstração: basta considerar o caso em que k > n. Como k
tal que k – n = q . (m) e, portanto,
ak = ak–n . an = aq.
(n)
. an = (a
(n) q
) . an
n (mod (m)) existe q
1
an (mod m)
Exemplo 10.4: sejam a = 5, m = 6, k = 8 e n = 2. Temos (6) = 2, e 8 2 (mod 2).
Como 52 1(mod 6), então 58 1 (mod 6) e desta forma, 58 52 (mod 6).
10.6. RESOLUÇÃO DE CONGRUÊNCIAS LINEARES PELO TEOREMA DE EULER
A congruência linear a.x b (mod m) no caso em que o MDC(a, m) = 1, admite uma única
solução módulo m, que se pode facilmente obter usando o Teorema de Euler. Realmente,
temos:
ak = ak–n . an = aq.
(n)
. an = (a
(n) q
) . an
an (mod m)
portanto,
a.x
b.a
(m)
(mod m)
Como o MDC(a,m) = 1, podemos cancelar o fator comum a, que resulta em:
x
b.a
(m) – 1
(mod m)
Exemplo 10.5: no caso da congruência linear 3.x
x
5. 3
(8)–1
(mod 8)
5.34–1 (mod 8)
5 (mod 8), onde o MDC(3, 8) = 1, temos:
5.27 (mod 8)
135 (mod 8)
7 (mod 8)
Em particular,
a.x
1 (mod n)
x
a
(n)
(mod n)
determina um inverso de a módulo n.
Exemplo 10.6: queremos determinar o inverso de 7 módulo 11, ou seja, queremos resolver a
congruência linear 7.x 1 (mod 11).
Queremos determinar o menor inteiro positivo em 11 que satisfaça a equação
x
7
(11) – 1
(mod 11)
710–1 (mod 11)
79 (mod 11)
8 (mod 11)
assim, temos que x = 8 é a menor solução positiva em 11 para o problema.
155
CAPÍTULO 10
FERMAT, WILSON E EULER
10. 7. RESOLUÇÃO DA EQUAÇÃO (n) = m.
Vejamos agora como resolver a equação (n) = m, quando m é um inteiro positivo dado.
Sabemos que não existe solução se é m é um ímpar maior que 1. Em geral não se conhece
uma fórmula para se resolver essa equação, quando m é qualquer inteiro positivo par. Mesmo
assim, existe um método com o qual podemos determinar todas as soluções dessa equação.
r
piki um inteiro qualquer, decomposto em fatores primos, que satisfaça a equação
Seja n
i=1
r
(n) = m. Como
r
(n) =
(n) =
( piki
piki 1 )
di
pi 1 ,
i 1
( piki
i 1
r
i 1
piki 1 ) , então
piki 1( pi
1)
m.
Façamos
i
1, 2,..., r
Assim,
r
i 1
r
i 1
m,
di
pi
piki
n.
n
m,
piki 1d i
di
pi
m,
m
r
i 1
di
r
.
i 1
pi
Todas essas equações estabelecem três condições para os d i , que nos permitem determinar
os valores de n que satisfazem a equação (n) = m:
1) cada d i 1 é um primo;
2) cada d i é um divisor positivo de m;
156
CAPÍTULO 10
FERMAT, WILSON E EULER
m
3)
i 1
r
é um inteiro positivo que contém somente fatores primos incluídos em
r
di
Exemplo 10.7: Resolver a equação
x
24
pi .
i 1
23.3
Os divisores positivos de 24 são 1,2,3,4,6,8,12 e 24. Sem dúvida, os possíveis d i são os
inteiros 1,2,4,6 e 12 já que eles são os únicos tais que d i 1 é um primo. Como todos os
produtos de quatro ou cinco dos d i são maiores que 24,podemos eliminar esses produtos
24
imediatamente; quer dizer, k
não e um inteiro quando quatro o mais d i são
di
i 1
k
considerados. As possíveis restantes expressões para
d i são os seguintes produtos abaixo:
i 1
1, 2 , 4, 6 , 12 , 1.2 , 1.4, 2.4
2.6 , 2. 12 , 4.6 , 4.12 , 6.12 ,1.2.4, 1.2.6,
1.2.12, 1.4.6 , 1.4.12 ,1.6.12 , 2.4.6 ,
2.4.12, 2.6.12 , e 4.6.12.
Os oito últimos produtos destacados, podem ser eliminados, já que em cada caso
24
k
di
i 1
não é um inteiro ; os sete primeiros produtos destacados, podem ser eliminados ,já que em
cada caso
24
k
di
k
, contém fatores primos não contidos em
k
(di 1) , isto é,
i 1
pi .
i 1
i 1
157
CAPÍTULO 10
FERMAT, WILSON E EULER
k
Agora, os produtos restantes para
d i conduzem as soluções:
i 1
24
k
di
di
i 1
i 1
1.2
1.6
1.12
2.4
2.12
4.6
1.2.4
1.2.6
1.2.12
1.4.6
22. 3
22
2
3
1
1
3
2
1
1
24
k
k
pi
i 1
2.3
2.7
2.13
3.5
3.13
5.7
2.3.5
2.3.7
2.3.13
25.7
k
di
k
.
pi
i 1
x
i 1
23.32
23.7
2
2 .13
32.5
3.13
5.7
2.32.5
22.3.7
2.3.13
2.5.7
72
56
52
45
39
35
90
84
78
70
Em conseqüência, as soluções da equação ( x) 24 são 35,39,45,52,56,70,72,78,84 e 90.
Exemplo 10.8: Usando o fato de que a função Totient é multiplicativa, encontrar as 10
soluções da equação ( x) 24 .
Consideremos ( x)
(n1 ). (n2 ). (n3 )... 24 ,onde n1, n2 , n3 são relativamente primos entre
si dois a dois. Investigando uma lista de valores de (n) , podemos determinar conjuntos de
fatores (n1 ), (n2 ), (n3 ) ,... onde n1, n2 , n3 ,... são relativamente primos dois a dois cujos
produtos são iguais a 24. Os produtos n1.n2 .n3 ... representam soluções da equação
x
158
24.Como
Produto
mdc
Solução
(3) .
(13) = 2.12
(3,13) = 1,
x = 39;
(4) .
(13) = 2.12
(4,13) = 1,
x = 52;
(5) .
(7) = 4.6
(5,7) = 1,
x = 35;
(5) .
(9) = 4.6
(5,9) = 1,
x = 45;
(6) .
(13) = 2.12
(6,13) = 1,
x = 78;
(7) .
(8) = 6.4
(7,8) = 1,
x = 56;
(7) .
(10) = 6.4
(7,10) = 1,
x = 70;
(7) .
(12) = 6.4
(7,12) = 1,
x = 84;
(8) .
(9) = 4.6
(8,9) = 1,
x = 72;
(9) .
(10) = 6.4
(9,10) = 1,
x = 90;
CAPÍTULO 10
FERMAT, WILSON E EULER
Assim, as dez soluções da equação x 24 são 35, 39, 45, 52, 56, 70, 72, 78, 84 e 90.
Este resultado esta de acordo com exemplo anterior.
Note que para usar este método e necessário conhecer primeiramente o número de soluções
desejadas ou limitar os valores das soluções desejadas com o fim de decidir quando termina a
investigação.
Kevin Ford provou em 1999 que para todo número inteiro k ≥ 2 há um número m para o qual
a equação ( x) m tem exatamente k soluções; este resultado já haviam sido conjecturado
por Sierpiński. No entanto, nenhum desses m é conhecido para k=1, e de acordo com a
conjectura da função totiente de Carmichael acredita-se que neste caso esse m não existe.
10.8 – VALÊNCIA DA FUNÇÃO TOTIENTE: N (m) .
A função N (m) , chamada de Valência da Função Totiente, é definida como o número de
inteiros positivos k, tal que
(k) = m, também chamada de multiplicidade de m.
Exemplo 1: Temos que N (8) 5 , porque existem apenas 5 inteiros, k = 15, 16, 20, 24 e 30,
tal que (k) = 8. A tabela abaixo mostra os valores de N (m) para n
m
N (m)
1
2
1, 2
2
3
3, 4, 6
4
4
5, 8, 10, 12
6
4
7, 9, 14, 18
8
5
15, 16, 20, 24, 30
10
2
11, 22
12
6
13, 21, 26, 28, 36, 42
12.
k, tal que (k) = m
Pode-se provar que existem inteiros pares m > 1 que não são valores assumidos pela função
Totiente, como por exemplo N (14) 0 . Andrzej Schinzel provou em 1956, que para todo
t
1 , o valor 2. 7 t não é valor da função Totiente. Veja que N (26)
0 e, 26 não é da
forma 2. 7t . Mas, em 1976, Nathan Mendelsohn provou a existência de uma infinidade de
números primos p, tais que para todo t 1 , o valor 2t . p não é assumido pela função Totiente
e, 26 é um número dessa forma.
Observe os seguintes valores da função Totiente:
159
CAPÍTULO 10
FERMAT, WILSON E EULER
m
(m)
2
1!
4
2!
9
3!
35 4!
231 5!
Himanshu Gupta provou em 1950, que para todo t 1 existe m tal que (m) = t!
EXERCÍCIOS
1.
Calcular (420), (1001), (5040) e (8316).
2.
Verificar que (n + 2) = (n) + 2, para n = 12,
14, 20.
3.
Verificar que (n) = (n + 1) = (n + 2), para n
= 5186.
4.
Verificar que (3k . 568) = (3k . 638).
5.
Verificar que (n) é uma função aritmética
multiplicativa para n = 144.
6.
Verificar o Teorema de Euler com a = 3 e n =
10; a = 7 e n = 12.
7.
Verifique que:
11. Usando o Teorema de Euler, resolver as
seguintes congruências lineares:
(a) 5x
(b) 2x
(c) 7x
(d) 8x
(e) 2x
(f) 5x
7 (mod 12)
3 (mod 9)
1 (mod 10)
4 (mod 5)
1 (mod 17)
– 3 (mod 8)
12. Demonstrar que se n é um inteiro positivo
ímpar, então:
11357 59(mod96)
31304
b) 6
84(mod101)
4205
c) 23
67(mod77)
12101
d) 20
20(mod57)
850
e) 17
100(mod143)
a)
8.
Achar os dois últimos algarismos da direita do
inteiro 3256.
9.
Calcular o menor inteiro positivo n tal que n
71015 (mod 31).
10. Achar o algarismo das unidades do inteiro 3145.
160
a)
b)
(2.n) = (n)
(4n) = 2. (n)
13. Resolva em  as seguintes equações:
a)
(n) = 12
b)
(n) = 18
c)
(n) = 20
d)
(n) = 30
e)
(n) = 4!
CAPÍTULO 10
FERMAT, WILSON E EULER
10.9. TEOREMA CHINÊS DO RESTO (TCR)
O livro “Manual Aritmético do Mestre Sol” foi escrito por Sun Zi Suanjing (ou Sun Tzu Suan
Ching), provavelmente entre 280 d.C. a 483 d.C. O livro está dividido em 3 capítulos.
O 1º capítulo, contém apenas dois problemas que dizem respeito sobretudo a métodos para
fazer multiplicações e divisões, utilizando “palitinhos chineses”. O segundo capítulo, contém
28 problemas, apresenta métodos para o cálculo com frações, extração da raiz quadrada,
determinação de áreas e volumes, proporções e regra de três simples. O terceiro capítulo
contém 36 problemas aritméticos.
No problema 26 (também conhecido como “problema do Mestre Sun”) do 3° capítulo, Sun
Tzu utiliza pela primeira vez o chamado Teorema Chinês do Resto (TCR). O problema está
enunciado abaixo:
“Temos coisas, mas não sabemos quantas; se as contarmos de três em três, o resto é 2; se as
contarmos de cinco em cinco, o resto é 3; se as contarmos de sete em sete, o resto é 2.
Quantas coisas temos?”
Em notação moderna, este problema equivale a procurar as soluções do seguinte sistema de
congruências:
x
2 (mod 3)
x
3 (mod 5)
x
2 (mod 7)
Resolver problemas como esse é um dos objetivos deste tópico.
Teorema 10.11 (Teorema Chinês do Resto – TCR): sejam m1, m2, ..., mk inteiros positivos
primos entre si dois a dois, isto é, tais que o MDC(mi, mj) = 1 se i j. Nestas condições, o
sistema de congruências lineares:
x
a1 (mod m1 )
x
a2 (mod m2 )
...
x
ak (mod mk )
tem única solução módulo m = m1, m2, ..., mk , dada por:
x a1.M1.x1 + a2.M2.x2 + ... + ak.Mk.xk (mod m)
Demonstração: para cada k = 1, 2, 3, ..., r, seja:
Seja Mk
m
mk
m1 . m2 . ... . mr
mk
. Como os inteiros mi são todos primos entre si dois a dois, o
MDC(Mr, mr) = 1, de modo que a congruência linear Mr . x
solução x xr (mod mr)
Posto isto, vamos mostrar que o inteiro x
solução do sistema considerado.
1 (mod Mr) tem única
a1.M1.x1 + a2.M2.x2 + ... + ak.Mk.xk (mod m) é uma
161
CAPÍTULO 10
FERMAT, WILSON E EULER
Com efeito se i
r, então mr | Mi e Mi
x
0 (mod mr), que implica em:
a1.M1.x1 + a2.M2.x2 + ... + ak.Mk.xk (mod m)
Para demonstrar a unicidade desta solução, suponhamos que x1 é uma outra solução
qualquer do sistema considerado. Então:
x
ar (mod mr)
x1 (mod mr), r = 1, 2, ..., k.
e, portanto, mr | (x – x1), r = 1, 2, ..., k.
Mas, o MDC(mi, mj) = 1 implica em (m1 . m2 . … . mk) | (x – x1), isto é, m | (x – x1) e x
(mod m), com o que termina a demonstração do TCR.
x1
Teorema 10.12: Sejam m1, m2, ..., mk inteiros positivos primos entre si dois a dois, e sejam
a1, a2, ..., ak inteiros tais que MDC(ar, mr) = 1 para r = 1, 2, ..., k. Nestas condições, o
sistema de congruências lineares:
a1.x
b1 (mod m1 )
a2 .x
b2 (mod m2 )
...
ak .x
bk (mod mk )
tem única solução módulo m = m1 . m2 . ... . mk.
Demonstração: como o MDC(ar, mr) = 1, a congruência linear ar.x
solução x ar (mod mr), de modo que:
ar.ar
1 (mod mr), e ar.ar.x
e, portanto, é equivalente a congruência x
1 (mod mr) tem única
x (mod mr)
ar . br (mod mr).
Assim sendo, o sistema considerado é equivalente ao seguinte sistema de congruências
lineares:
x
a1.b1 (mod m1 )
x
a2 .b2 (mod m2 )
...
x
ak .bk (mod mk )
o qual tem, pelo TCR, uma única solução módulo m = m1 . m2 . ... . mk:
xk . Mk 1 (mod mk)
xk
Mk (m) 1 (mod mk )
onde:
162
CAPÍTULO 10
FERMAT, WILSON E EULER
m
, k = 1, 2, 3, ...
mk
(ii) xk . Mk 1 (mod mk)
mk .
(i) Mk
xk
Mk (m) 1 (mod mk ) , ou seja, xk é o inverso de Mk módulo
Exemplo 10.7: utilizando o TCR, resolver o sistema de congruências lineares:
x
8 (mod 5)
x
5 (mod 3)
x
11 (mod 7)
x
2 (mod 4)
Resolução: os módulos 5, 3, 7 e 4 das congruências lineares que formam o sistema são
primos entre si dois a dois, de modo que pelo TCR este sistema tem uma única solução
módulo m =m1 . m2 . m3 . m4 = 5 . 3 . 7 . 4 = 420. Temos então:
M1
m
m1
420
5
84 , M2
m
m2
420
3
140 , M3
m
m3
420
7
60 , M4
m
m4
420
4
105
Os inversos xk dos Mk são dados por:
84 x1
140 x2
60 x3
105 x4
1 (mod 5)
1 (mod 3)
1 (mod 7)
1 (mod 4)
Aplicando o método de Euler nas equações acima obtemos as soluções respectivas:
x1 = 4, x2 = 2, x3 = 2, e x4 = 1.
Portanto, temos :
x
x
x
a1.M1.x1 + a2.M2.x2 + ... + ak.Mk.xk (mod m)
8 . 84 . 4 + 5 . 140 . 2 + 11 . 60 . 2 + 2 . 105 . 1 (mod 420)
5618 (mod 420) 158 (mod.420),
segue-se que x = 158 é a menor solução positiva módulo 420, do sistema de
congruências lineares dado. Qualquer outra solução é da forma:
x
158 (mod 420)
x = 158 + 420.k, k
.
Exemplo 10.8:
5.x
3.x
11.x
11 mod 17
19 mod 32
6 mod 37
163
CAPÍTULO 10
FERMAT, WILSON E EULER
Resolução: Como o MDC(17, 32) = MDC(17, 37) = MDC(32, 37) = 1, o sistema possui
solução.
De 5.x 11 (mod 17), obtemos x 9 (mod 17).
De 3.x 19 (mod 32), obtemos x 17 (mod 32)
De 11.x 6 (mod 37), obtemos x 14 (mod 37)
assim temos o seguinte sistema:
x 9(mod 17)
x 17(mod 32)
x 14(mod 37
Usando o TCR:
a1 = 9, a2 = 17, a3 = 14.
m1 = 17, m2 = 32, m3 = 37.
m = m1 . m2 . m3 = 17 . 32 . 37 = 20128.
m 20128
m 20128
629 , M3
M1
1184 , M2
m1
m2
17
32
m
m3
20128
37
544
Então 1184.b1 1 (mod 17), 629.b2 1 (mod 32) e 544.b3 1 (mod 37).
De onde concluímos: b1 = 14 , b2 = 29 e b3 = 10.
A solução geral será:
x
x
9 . 14 . 1184 + 17 . 29 . 629 + 14 . 10 . 544 (mod 20128)
535441 12113 (mod 20128)
EXERCÍCIOS
1. Resolver os seguintes sistemas de congruências lineares utilizando o TCR:
x 3 (mod 5)
x 1 (mod 5)
a) x 5 (mod 7)
b) x 5 (mod 7)
c) x
x 5 (mod11)
4 (mod11)
d) x 14 (mod 29)
x 7 (mod11)
x 7 (mod11)
x 3 (mod17)
x 15 (mod 31)
x 7 (mod 9)
x 28 (mod 29)
x
a (mod 3)
2 x 1 (mod 5)
f) x 30 (mod 31)
g) x b (mod 5)
h) 4 x 1 (mod 7)
x 10 (mod11)
x c (mod8)
e) x 10 (mod 4)
x 1 (mod 7)
164
x 5 (mod 6)
5 x 9 (mod11)
CAPÍTULO 10
FERMAT, WILSON E EULER
10.10. POTENCIAÇÃO: UMA APLICAÇÃO DO TEOREMA DE EULER
Seja m um inteiro positivo tal que mdc(a, m) = 1 . Então:
a
(m)
1 (mod m)
Aplicamos este resultado ao cálculo de potência módulo m. De uma maneira geral o problema
é o seguinte. São dados a, k e m três inteiros positivos, dos quais sabemos que m é co-primo
com a. Digamos que k é muito grande (o caso difícil) e queremos achar a forma reduzida de ak
(mod m).
Podemos simplificar as contas usando o teorema de Euler. Estamos supondo que k é grande,
na prática precisamos saber apenas que k
(m) . Dividindo k por (m) , obtemos
k = (m) .q + r, onde o resto r satisfaz 0 r
(m) – 1. Temos, então, que:
ak
a
(m).q+r
(mod m)
(a
(m) q
) . ar (mod m)
Mas pelo teorema de Euler, a (m) 1 (mod m) . Obtemos, portanto, da equação acima que ak
ar (mod p).
Um exemplo numérico para convencê-lo das vantagens deste resultado tão simples.
Queremos calcular 25432675 (mod 13). Da maneira como vínhamos procedendo teríamos que
efetuar uma quantidade enorme de potenciações módulo 13. Usando a idéia acima, obtemos o
resto da divisão de k = 5432675 por (13), que é r = 11, e assim:
25432675
Logo, 25432675
(2
(13) q
) . 211 (mod 13)
211 (mod 13)
7 (mod 13)
7 (mod 13).
10.11 – POTENCIAÇÃO: UMA APLICAÇÃO DO TEOREMA CHINÊS DO RESTO (TCR)
Podemos aplicar o TCR para simplificar o cálculo de potências módulo n em alguns casos
especiais. Suponhamos que n = p1 . p2 . ... . pk, onde p1 < p2 < ... < pk são números primos.
Assim, estamos supondo que, na fatoração de n, cada fator primo aparece com multiplicidade
1. Neste caso fica muito fácil calcular a forma reduzida de am (mod n).
Em primeiro lugar, usamos o PTF para achar a forma reduzida de am módulo cada um dos
primos p1, p2, ..., pk separadamente. Digamos que:
am
am
am
r1 (mod p1) e 0
r2 (mod p2) e 0
rk (mod pk) e 0
r1 < p1
r1 < p2
rk < pk
165
CAPÍTULO 10
FERMAT, WILSON E EULER
Para achar a forma reduzida de am (mod n), basta resolver o sistema:
x
x
x
r1 (mod p1)
r2 (mod p2)
...
rk (mod pk)
Observe que este sistema sempre tem solução, já que os módulos são primos distintos e,
portanto, o MDC entre dois quaisquer entre eles é sempre igual a 1. Além disto, o teorema nos
garante que o sistema tem uma única solução r (mod n), onde n = p1 . p2 . ... . pk.
Logo am r (mod n), e obtivemos o que queríamos.
Vejamos um exemplo numérico. Digamos que queremos calcular a forma reduzida de
26754 (mod 1155)
Fatorando 1155 vemos que é 1155 = 3 . 5 . 7 . 11, todos primos entre si com multiplicidade 1.
Aplicando o PTF a cada um destes primos, temos que:
26754
26754
26754
26754
2r1
2r2
2r3
2r4
(mod 3)
(mod 5)
(mod 7)
(mod 11)
Assim:
6754 = (3).q1 + r1
6754 = (5).q2 + r2
6754 = (7).q3 + r3
6754 = (11).q4 + r4
6754 = 2 . 3377 + 0, logo 20
6754 = 4 . 1688 + 2, logo 22
6754 = 6 . 1125 + 4, logo 24
6754 = 10 . 675 + 4, logo 24
1 (mod 3).
4 (mod 5).
2 (mod 7).
5 (mod 11).
Precisamos, portanto, resolver o sistema:
x
x
x
x
1 (mod 3)
4 (mod 5)
2 (mod 7)
5 (mod 11)
Solução:
m = m1 . m2 . m3 . m4 = 3 . 5 . 7 . 11 = 1155
M1
166
m
m1
1155
3
385 ; M2
m
m2
1155
5
231; M3
m
m3
1155
7
165 ; M4
m
m4
1155
11
105
CAPÍTULO 10
FERMAT, WILSON E EULER
M1.x1
M2.x2
M3.x3
M4.x4
1 (mod m1)
1 (mod m2)
1 (mod m3)
1 (mod m4)
385.x1
231.x2
165.x3
105.x4
1 (mod 3)
1 (mod 5)
1 (mod 7)
1 (mod 11)
x1 385 (3)–1 (mod 3) 1 (mod 3)
x2 231 (5)–1 (mod 5) 1 (mod 5)
x3 165 (7)–1 (mod 7) 2 (mod 7)
x4 105 (11)–1 (mod 11) 2 (mod 11)
Assim, temos:
x
x
x
a1.M1.x1 + a2.M2.x2 + a3.M3.x3 + a4.M4.x4 (mod m)
1 . 385 . 1 + 4 . 231 . 1 + 2. 165 . 2 + 5 . 105 . 2 (mod 1155)
3019 (mod 1155) 709 (mod 1155)
EXERCÍCIOS
1) Construa as tabelas de operações em
8 ,  9 ,
10 , 13 e 17 .
2) Encontre os dois números primos, cujos
produtos, geraram os números abaixo,
utilizando o método da Fatoração de Fermat:
a) 437623
b) 919199
c) 9797
d) 4061
e) 19109
3) Calcule a potências utilizando o método da
exponenciação rápida.
a) 33268 (mod 335)
b) 1774096 (mod 277)
c) 818192 (mod 92)
d) 482048 (mod 20)
4) Calcule as potências utilizando o Pequeno
Teorema de Fermat
a) 52349899 (mod 17)
b) 4511223311 (mod 19)
c) 1001002003007 (mod 281)
5) Calcule as potências utilizando o Pequeno
Teorema de Fermat e o Algoritmo Chinês do
Resto.
a) 4753 (mod 437623)
b) 51773 (mod 919199)
c) 8397 (mod 2926)
d) 313961 (mod 12369)
167
CAPÍTULO 10
FERMAT, WILSON E EULER
LEITURA COMPLEMENTAR: PSEUDOPRIMOS
De acordo com o teorema de Fermat, se p é primo e a é um inteiro qualquer não divisível por
p, então ap a (mod p). Imagine então a seguinte situação: temos um número ímpar n, e
desejamos saber se é, ou não, composto.
Digamos que descobrimos um inteiro b tal que bn
b (mod n). Pergunta-se: n pode ser
primo? A resposta é não, porque isto viola o teorema de Fermat. Logo, isto nos dá uma
maneira indireta de verificar se um número é composto.
Observe que, na prática, só precisamos considerar os inteiros b no intervalo 1 < b < n – 1.
Por quê? Em primeiro lugar, como estamos trabalhando módulo n, qualquer inteiro é
congruente a um inteiro no intervalo de 0 a n – 1. Além disso, a equação bn b (mod n) é
sempre satisfeita quando b é 0, 1 ou n – 1.
Este teste produz uma situação surpreendente. Com ele podemos chegar à conclusão de que
um número é composto mesmo que não nos seja possível determinar seus fatores. Antes de
fazer um exemplo, é conveniente formularmos um teste de uma maneira mais fácil de utilizar
na prática. Para isto usaremos a segunda versão do teorema de Fermat.
Eis o teste: se n > 0 e 1 < b < n – 1 são números inteiros, e bn–1 1 (mod n), então n é um
número composto. O número b é conhecido como uma “testemunha de fato” de n ser
composto.
Será que podemos inverter o teorema de Fermat para verificar se um número é primo? Na
verdade, estamos perguntando se um número ímpar n que satisfaz
bn–1 1 (mod n), para algum 1 < b < n – 1, é primo? Leibniz, famoso pela invenção do cálculo
(quase ao mesmo tempo que Newton), achava que sim, e usou isto como um critério de
primalidade. Na verdade, ele tomava apenas b = 2, que é o caso mais simples de calcular.
Infelizmente isto não é verdade. Por exemplo, 2340 1 (mod 341). Logo, segundo Leibniz,
341 seria um número primo. Mas, 341 = 11 . 31, é composto. Estes “falsos primos” são
conhecidos como pseudoprimos. Isto é, um inteiro positivo n, ímpar e composto, é um
pseudoprimo para a base b (onde 1 < b < n – 1) se bn – 1 1 (mod n). Assim, 341 é um
pseudoprimo para a base 2.
Apesar de às vezes dar errado, o teste de Leibniz é muito útil. Pelo menos para números
pequenos, ele acerta mais do que erra. Por exemplo, entre 1 e 109 existem 50.847.534 primos,
mas apenas 5597 pseudoprimos para a base 2. Logo um número menor que um bilhão que
passa no teste de Leibniz apenas com a base 2 tem uma alta probabilidade de ser primo.
Além disto, usamos apenas a base 2 até agora. Por que não testar para mais de uma base?
Fazendo isto o teste fica ainda mais eficiente. Por exemplo, 3340 56 (mod 341). Logo 3 é
uma testemunha de fato de que 341 é composto. Na verdade, há apenas 1272 pseudoprimos
simultâneos para as bases 2 e 3 no intervalo entre 1 e 109.
Como estamos limitando nossas bases ao intervalo entre 1 e n – 1, podemos considerar a
possibilidade de testar se n é pseudoprimo para todas estas bases. Se n é grande, isto vai ser
impossível, logo o interesse prático é muito limitado. Por outro lado, este problema leva a
168
CAPÍTULO 10
FERMAT, WILSON E EULER
algumas questões surpreendentes, e de real interesse prático, como veremos em seguida com
os números de Carmichael.
Números de Carmichael
Primeiramente, não existem números que sejam pseudoprimos para todas as bases. Isto é fácil
de constatar. Se n é composto, então tem um fator b, ou seja, MDC(b ,n) 1. Como também o
MDC(bn – 1, n) 1, temos pelo teorema de inversão que b não pode ser inversível módulo n.
Em particular, bn – 1 1 (mod n). Logo n não é pseudoprimo para a base b. Entretanto, pode
ocorrer que um número composto n seja pseudoprimo para todas as bases que são primas a n.
É este tipo de número que desejamos discutir.
É preferível reformular o problema da seguinte maneira: seja n um número inteiro positivo,
como caracterizar os números compostos n que satisfazem à equação bn b (mod n)?
A vantagem desta formulação é que podemos dispensar a condição MDC(b, n) = 1, o que é
muito conveniente. O primeiro matemático a dar exemplos destes números foi R. D.
Carmichael, em um artigo publicado em 1912. Por isso são chamados números de
Carmichael.
Portanto, um número composto ímpar n > 0 é um número de Carmichael se bn b (mod n)
para todo 1 < b < n – 1. Observe que, para um número ser de Carmichael é preciso que seja
composto. Um número primo p também satisfaz à equação bp b (mod p), mas não é um
número de Carmichael.
Como o próprio Carmichael determinou, o menor número de Carmichael é 561. Em princípio
podemos verificar isto usando a definição. Entretanto, mesmo para um número relativamente
pequeno como este, isto é difícil de fazer usando apenas lápis e papel.
Afinal, para mostrar que 561 é um número de Carmichael pela definição, precisamos mostrar
que b561 b (mod 561) para b = 2, 3, 4, 5, ..., 558, 559; o que dá um total de 558 bases a
serem testadas, algumas não tão pequenas. Não há duvida de que esta é uma tarefa para um
computador. Entretanto, mesmo um computador razoável pode ter dificuldade em usar este
método para verificar que, por exemplo:
349407515342287435050603204719587201
é um número de Carmichael.
Felizmente há uma maneira mais simples de verificar que um dado número composto é de
Carmichael. Consideremos 561 mais uma vez. Podemos facilmente fatorá-lo:
561 = 3 . 11 . 17
Seja agora b um número inteiro entre 2 e 559. Queremos mostrar que b561
b (mod 561).
Nossa estratégia será a seguinte: mostraremos que b561 – b é divisível por 3, 11 e 17 para
todos os valores de b entre 2 e 559. Como são primos distintos, segue que o produto destes
primos divide b561 – b. Mas este produto é 561, e dizer que 561 divide b561 – b é equivalente a
dizer que b561 b (mod 561).
169
CAPÍTULO 10
FERMAT, WILSON E EULER
Só nos resta mostrar que b561 – b é divisível por cada um dos fatores de 561, separadamente.
Como estes fatores são primos, o teorema de Fermat vem em nossa ajuda. Vamos efetuar as
contas para o primo 17 e deixar 3 e 11 como exercício. Queremos verificar que b 561 – b é
divisível por 17, isto é, b561 b (mod 17).
Há dois casos a considerar. Se 17 divide b então ambos b e b561 são membros de b
congruentes a zero modulo 17, logo a congruência é imediatamente verificada. Mas, digamos
que 17 não divide b. Neste caso, o teorema de Fermat diz que b16 1 (mod 17). Para aplicar
isto, precisamos dividir 561 por (17 – 1), mas isto resulta em 561 = 35 . 16 + 1. Assim, b561
(b16)35 . b (mod 17) (1)35 . b (mod 17) b (mod 17).
Alguns números de Carmichael (existem infinitos):
561, 1105, 1729, 2465, 2821, 6601, 8911, 10585, 15841, 29341, 41041, 46657, 52633,
62745, 63973, 75361, 101101, 115921, 126217, 162401, 172081, 188461, 252601,
278545, 294409, 314821, 334153, 340561, 399001, 410041, 449065, 488881, 512461,
530881, 552721, 656601, 658801, 670033, 748657, 825625, 838201, 852841, 997633.
Provaremos que, se t é tal que (6.t + 1), (12.t + 1) e (18.t + 1) são todos primos, então o seu
produto é um número de Carmichael. Não se sabe ainda (2005), se existe uma infinidade de
tais números t.
Demonstração: temos que n = (6t + 1).(12t + 1).(18t + 1) é composto.
E que (n – 1) = 1296.t3 + 396.t2 + 36.t 0 (mod 36)
Seja o MDC(a, n) = 1. Então, a é relativamente primo com (6.t + 1), (12.t + 1) e (18.t + 1).
Pelo PTF:
a6.t 1 (mod 6.t + 1)
a12.t 1 (mod 12.t + 1)
a18.t 1 (mod 18.t + 1)
a36.t
a36.t
a36.t
1 (mod 6.t + 1)
1 (mod 12.t + 1)
1 (mod 18.t + 1)
Logo, a36.t 1 (mod n). Como 36.t divide (n – 1), temos que an–1
Como n é composto, n é um número de Carmichael.
170
1 (mod n).
Capítulo 11
CIFRA DE CÉSAR
INTRODUÇÃO À CRIPTOGRAFIA
A palavra Criptografia
é composta por dois termos gregos kryptos (kryptos secreto,
escondido, oculto) e grapho (grapho - escrita grafia).
A criptografia é uma arte ou ciência de escrever ocultamente talvez tão antiga quanto a
própria escrita, hoje em dia é um dos métodos mais eficientes de se transferir informação, sem
que haja a possibilidade de interferência por parte de terceiros. É o estudo de técnicas
matemáticas, relacionadas com os aspectos de segurança e confidencialidade de informação, a
integridade de dados, a autenticação de entidades e a autenticidade de origem de dados, ou
seja, consiste na conversão de dados num código secreto como medida de segurança para que
possam existir comunicações seguras.
A criptografia lida de um modo muito estreito com termos como - encriptação e
desencriptação. A encriptação é a conversão de dados para uma forma que não será
compreendida facilmente por pessoas autorizadas com o objectivo de assegurar a privacidade
mantendo a informação escondida e ilegível mesmo para quem vê os dados encriptados. A
desencriptação é o processo de converter dados encriptados de volta á sua forma original, para
que a mensagem possa ser compreendida e para isso acontecer requer alguma informação
secreta, usualmente denominada chave de desencriptação. A chave de desencriptação é o
algoritmo que desfaz o trabalho do algoritmo de encriptação. Um algoritmo é um programa de
computador que pode ser visto como um algoritmo elaborado. É baseada em chaves, uma
informação pode ser codificada através de algum algoritmo de criptografia, de modo que,
tendo conhecimento do algoritmo e da chave utilizados, é possível recuperar a informação
original fazendo o percurso contrário da encriptação, a desencriptação.
171
CAPÍTULO 11
CIFRA DE CÉSAR
Embora os códigos secretos remontem aos primórdios da comunicação escrita, tem havido um
aumento recente de interesse no assunto devido à necessidade de manter a privacidade da
informação transmitida ao longo de linhas públicas de comunicação. Na linguagem da
criptografia, os códigos são denominados cifras, as mensagens não codificadas, são textos
comuns e as mensagens codificadas são textos cifrados ou criptogramas. O processo de
converter um texto comum em cifrado é chamado cifrar ou criptografar e o processo
inverso de converter um texto cifrado em comum é chamado decifrar.
As cifras mais simples, denominadas cifras de substituição (ou Código de César), são as
que substituem cada letra do alfabeto por uma outra letra.
Por exemplo, na cifra de substituição
Comum A B C D E F G H I J K L M N O P Q R S T U V W X Y Z
Cifra D E F G H I J K L M N O P Q R S T U V W X Y Z A B C
A letra de texto comum A é substituída por D, a letra de texto comum B por E e assim por
diante.
Com esta cifra, a mensagem de texto comum
ROMA NÃO FOI CONSTRUÍDA
EM UM DIA
fica
URPD QDR IRL FRQVWUXLGD HP XP
GLD
A Cifra de César - Um dos sistemas criptográficos mais antigos e simples é a chamada
“cifra de César”, em homenagem ao famoso imperador romano . Júlio César usou sua famosa
cifra de substituição para cifrar mensagens governamentais. Atualmente denomina-se
qualquer cifra baseada na substituição cíclica do alfabeto de código de César. Com o uso de
dois discos concêntricos contendo todas as letras do alfabeto, a substituição se tornava
extremamente simples.
A cifra de César baseia-se na seguinte propriedade:
“Seja m>1 um inteiro. Para cada a
x a (mod m) é bijetiva”.
172
Z m fixado, temos que f: Zm
Zm definida por f(x) =
CAPÍTULO 11
CIFRA DE CÉSAR
Demonstração:
(i) f é injetiva.
De fato:
f(x) = f(y)
x + a = y + a (mod m)
x + a - a = y + a - a (mod m)
x = y (mod m)
x = y em  m , donde f(x) = f(y) acarreta x = y
(ii) f é sobrejetiva
Seja x um elemento qualquer de  m .
Então x - a está em  m e f(x - a) = x, uma vez que em  m x – a = x – a + km, para
todo inteiro k. Logo todo x em  m é igual a f(x - a), donde f é sobrejetiva.
Como f é injetiva e sobrejetiva, então f é bijetiva.
Imaginemos, por questão de simplicidade, as 26 letras usuais e o espaço (entre duas palavras)
associados aos elementos de Z 27 conforme a Tabela 1 abaixo ( adotaremos o símbolo [ ] para
indicar um espaço entre as palavras):
A
1
N
14
[]
0
B
2
O
15
C
3
P
16
D
4
Q
17
E
5
R
18
F
6
S
19
G
7
T
20
H
8
U
21
I
9
V
22
J
10
W
23
K
11
X
24
L
12
Y
25
M
13
Z
26
Tabela 1
Fixado um elemento a Z m ( a é a chave do código – de transmissão e de recepção), a
aplicação f: x x a (mod 27) permuta os elementos de Z 27 e, conseqüentemente, os
elementos do conjunto formado pelo símbolo do espaço e as 26 letras. Dessa forma cada
mensagem se transforma em código; o fato de f ser bijetiva garante que mensagens diferentes
são codificadas de maneira diferente e, ainda, a possibilidade da decodificação.
Exemplo 11.1: Vejamos como codificar a frase “EU VOU”, usando como chave a = 14, ou
seja com y = x + 15 (mod 27)
E
U
[]
V
O
U
5
21
0
22
15
21
5 + 14 19(mod 27)
21 + 14
8 (mod 27)
0 + 14
14 (mod 27)
22 + 14 9 (mod 27)
15 + 14 2 (mod 27)
21 + 14
8 (mod 27)
S
H
N
I
B
H
Portanto o código para a frase dada é “ SHNIBH”.
Para decodificar, considerando que o simétrico aditivo de 14 módulo 27 é 13
(pois, 14 + 13 0 (mod 27)), mantendo, portanto, a chave a = 14 ), procede-se assim:
173
CAPÍTULO 11
CIFRA DE CÉSAR
S
H
N
I
B
H
19
8
14
9
2
8
19 + 13
8 + 13
14 + 13
9+13
2+13
8+13
5
21
0
22
15
21
(mod 27)
(mod 27)
(mod 27)
(mod 27)
(mod 27)
(mod 27)
E
U
[]
V
O
U
11.1. Funções Polinomiais de Codificação
No exemplo acima usamos a função polinomial f(x) = x + 15 (mod 27) para codificar a
mensagem e usamos a sua inversa f -1(x) = x + 12 (mod 27) para decodificação.
A pergunta que podemos fazer é podemos usar qualquer função polinomial módulo m para
codificar uma mensagem? A resposta é não!
Como vimos, precisamos da inversa para decodificar a mensagem. Assim se escolhermos uma
função polinomial que não seja bijetiva no domínio trabalhado teremos problemas em
decodificar em situações normais. Além disso, nos casos em que tivermos multiplicações e
divisões da variável x, dependendo de Zm podemos não ter os inversos de x módulo m nos
casos em que m não é primo impossibilitando a decodificação. Vejamos alguns exemplos:
Exp1. Use a tabela abaixo e a função f ( x) x2 23(mod 29) para codificar a palavra DJ
A
B
C
D
E
F
G
H
I
J
K
L
M
1
2
3
4
5
6
7
8
9
10
11
12
13
N
O
P
Q
R
S
T
U
V
W
X
Y
Z
14
15
16
17
18
19
20
21
22
23
24
25
26
Á
É
[]
27
28
0
D
J
42
f (4)
2
f (10) 10
23 10(mod 29)
J
23 7(mod 29)
G
Queremos agora decodificar a mensagem JG. A inversa da função y = x 2 + 23 é obtida pelo
método prático de trocar x por y e colocar y em função de x, ou seja:
y x 2 23
y2
x
23
y
2
x 23 ou
y
2
x 6(mod 29)
Ou seja, para “voltarmos” estamos interessado em saber que número y elevado ao quadrado é
igual a um número x + 6 (mod 29) ( isto é, [f(x)]2 = x + 6 ). Assim,
174
CAPÍTULO 11
CIFRA DE CÉSAR
J
y2
10 6 16(mod 29)
Que número elevado ao quadrado é igual a 16 módulo 29? Resposta: 4. Logo J é D.
G
y2
7 6 13(mod7)
Que número elevado ao quadrado é igual a 13 módulo 29? Temos um problema! Existem dois
valores que elevado ao quadrado são iguais a 13 módulo 29: 10 e 19, letras J e S
respectivamente, pois
10.10 13(mod 29) e 19.19 13(mod 29)
impedindo-nos, em situações mais complexas, de decodificar a mensagem. Isso se deve ao
fato de f(x) = x2 + 23 não ser bijetiva.
Exercícios
Use a Tabela 2 abaixo para os seus cálculos
A
1
N
14
?
27
B
2
O
15
Á
28
C
3
P
16
Ã
29
D
4
Q
17
É
30
E
5
R
18
[]
0
F
6
S
19
Z31
G
7
T
20
H
8
U
21
I
9
V
22
J
10
W
23
K
11
X
24
L
12
Y
25
M
13
Z
26
1. Utilize as funções abaixo para codificar e sua inversa* para decodificar as mensagens
dadas.
a) y 2 x(mod 31) ; y x 7(mod 31) ; O MAIS QUERIDO
b) y 3x(mod 31) ; y 2 x 30(mod 31) ; NAVEGAR É PRECISO?
c) y 4 x(mod 31) ; y 3x 2(mod 31) ; SE VOU NÃO FICO
x 1
d) y 5 x(mod 31) ; y
(mod 31) ; ELVIS NÃO MORREU
x 29
* Use o método da troca de variáveis para encontrar a função inversa
175
CAPÍTULO 11
CIFRA DE CÉSAR
LEITURA COMPLEMENTAR:
ANÁLISE DE FREQUÊNCIA
Em qualquer língua, alguns sons são utilizados com mais frequência do que outros. Isto
significa que, na linguagem escrita, algumas letras também são mais utilizadas que outras.
Determinar a frequência com que ocorrem determinadas letras em determinada língua, ou
seja, fazer uma análise da frequência de ocorrência de letras.
Apesar de não se saber quem foi o primeiro a perceber que a variação na frequência de letras
poderia ser explorada para se quebrar cifras, a descrição mais antiga de que se tem
conhecimento e que descreve esta técnica data do século 9 e é devida ao cientista Abu Yusuf
Ya 'qub ibn Is-haq ibn as-Sabbah ibn 'omran ibn Ismail al-Kindi.
Conhecido como o filósofo dos árabes, al-Kindi foi o autor de 290 livros sobre medicina,
astronomia, matemática, linguística e música. No entanto, seu maior tratado, o qual foi apenas
redescoberto em 1987 no Arquivo Sulaimaniyyah Ottoman em Istambul, na Turquia, é
intitulado "Um Manuscrito sobre Decifração de Mensagens Criptográficas.
A cifra de substituição monoalfabética parecia inquebrável devido ao número muito grande de
chaves possíveis. Entretanto, havia uma fraqueza que minava sua segurança.
A quebra da cifra de substituição marca o nascimento da criptanálise. Tal fato ocorreu durante
os anos dourados da civilização islâmica, quando muitos manuscritos estrangeiros foram
levados para Bagdá para integrarem as grandes bibliotecas árabes. Alguns destes manuscritos
estavam encriptados, o que motivou os arrombadores de códigos a quebrarem as cifras para
revelar os segredos que continham.
As letras "A" e "I" são as mais comuns em Árabe. No Inglês, as letras mais comuns são o "E",
o "T" e o "A". Já no Português, as mais frequentes são "A", "E", "O" e "S".
Se uma mensagem é cifrada de modo que cada letra seja substituída por uma outra, então a
nova letra assumirá todas os atributos da letra original, inclusive com que frequência é
utilizada.
Desta forma, se a longa mensagem estiver em Português e se a letra mais comum na
mensagem cifrada for G, então G provavelmente representa A. Se a segunda letra mais
frequente na mensagem cifrada for W, então a probabilidade de que esteja substituindo o E é
bastante grande, e assim por diante.
Devemos considerar qual foi a língua utilizada para redigir a mensagem. Esta é uma questão
essencial porque define o padrão da frequência da ocorrência de letras que deve ser usada para
fazer a comparação. É óbvio que SEMPRE existem outras pistas que podem ajudar: o
remetente da mensagem, o destinatário, o possível assunto, etc.
Caso estejamos considerando o Português, é claro que se vai fazer uma análise de frequência
usando esta língua como base. Quanto mais longo for o texto, maior a probabilidade dos
valores encontrados estarem mais próximos dos valores padrão. Além disso, não se deve
esquecer que os valores padrão representam a MÉDIA da frequência de ocorrência. Se, por
exemplo, o padrão para a letra A é de 14.63%, esta frequência pode variar, digamos, de 13% a
17%. Neste caso estamos contando com um desvio de cerca de 2%.
176
CAPÍTULO 11
CIFRA DE CÉSAR
Abaixo estão as tabelas de das frequências relativas das letras nas línguas Portuguesa e
Inglesa.
Frequência relativa das letras no Português
Frequência relativa das letras no Inglês
177
CAPÍTULO 11
CIFRA DE CÉSAR
Exercícios
1. Suponha que você interceptou a mensagem:
JV
F
UZEYVZIF
JFSIRJJV V JV
VCR
DV
RDRJJV
Segundo suas fontes ela foi codificada utilizando a cifra de César de acordo com a tabela
seguinte
A
B
C
D
E
F
G
H
I
J
K
L
M
0
1
2
3
4
5
6
7
8
9
10
11
12
N
O
P
Q
R
S
T
U
V
W
X
Y
Z
13
14
15
16
17
18
19
20
21
22
23
24
25
É claro que você não sabe a chave usada. Como bom estudante de Estatística, você fez
uma análise de frequência ( isso é que é interesse nas coisas alheias!). Suponha que você
já havia interceptado um texto com mais de 1000 letras da mesma origem. De acordo
com a análise de frequência, a letra A é mais comum na língua portuguesa. Você
percebeu que a letra mais comum no texto cifrado foi a R. Com mais uma suposição de
que a chave ainda não foi trocada, decodifique a mensagem. Desconsidere os espaços.
2. Para as questões de 1 a 6, utilize a seguinte tabela de conversão:
A
B
C
D
E
F
G
1
2
3
4
5
6
7
N
O
P
Q
R
S
T
14
15
16
17
18
19
20
*O espaço em branco [ ] é representado pelo valor 0.
H
8
U
21
I
9
V
22
J
10
W
23
K
11
X
24
L
12
Y
25
M
13
Z
26
1. Utilizando a função f(x) = x + 13 (mod 27), codifique a palavra SOFTWARE.
2. Utilizando a função f(x) = 4.x + 11 (mod 27), codifique a palavra UNIVERSO. Encontre a
função inversa de f(x) e verifique se sua resposta está correta.
3. Decodifique a palavra GESIOF sabendo que a função f(x) = x + 14 (mod 27) foi utilizada
para cifragem.
4. Decodifique a palavra BVJZOL sabendo que a função f(x) = 5.x + 17 (mod 27) foi
utilizada para cifragem.
5. Dada a mensagem em texto simples FELIZ, e o seu respectivo texto cifrado YXDAR,
encontre a função afim que foi utilizada na cifragem. Determine a função inversa
6. Dada a mensagem em texto simples TEMPO, e o seu respectivo texto cifrado ORTNG,
encontre a função afim que foi utilizada na cifragem. Determine a função inversa.
178
Capítulo 12
CIFRA DE VIGENÈRE
A cifra de Vigenère tem este nome em homenagem a Blaise de Vigenère, embora realmente
tenha sido inventada antes por Giovan Batista Belaso. O que Vigenère fez foi modificar a
cifra para torná-la mais robusta.
A cifra de Vigenère é um método de encriptação que usa um série de diferentes cifras de
César baseadas em letras de uma senha. Trata-se de uma versão simplificada de uma mais
geral cifra de substituição polialfabética, inventada por Leone Battista Alberti a cerca de
1465.
A cifra de Vigenère foi uma campeã em segurança. Foram precisos 300 anos para que, quase
que simultaneamente, ao redor de 1860, Babbage (na Inglaterra) e Kasiski (na Alemanha)
quebrassem a cifra.
A invenção da cifra de Vigenère é erradamente atribuída a Blaise de Vigenère; encontra-se
originalmente descrita por Giovan Batista Belaso no seu livro datado de 1553 com o título La
cifra del. Sig. Giovan Batista Belaso.
Esta cifra é muito conhecida porque é fácil de perceber e de pôr em prática.
Conseqüentemente, muitos programadores implementaram esquemas de encriptação nas suas
aplicações que são no essencial cifras de Vigenère.
Descrição
Em uma cifra de César, cada letra do alfabeto é deslocada da sua posição um número fixo de
lugares; por exemplo, se tiver uma deslocação de 3, a letra A se torna D, B se torna E, etc. A
cifra de Vigenère consiste na sequência de várias cifras de César com diferentes valores de
deslocamento.
A cifra de Vigenère pode ser vista algebricamente. A encriptação pode ser escrita como
Ci
(Pi + ai) (mod m)
Pi
(Ci – ai) (mod m)
e a decriptação como
onde Pi corresponde aos valores das letras a serem cifradas, ai aos valores das letras da chave
e Ci aos valores das letras cifradas.
179
CAPÍTULO 12
CIFRA DE VIGENÈRE
Exemplo 12.1: Vamos supor agora, que a palavra-chave escolhida tenha sido "GREGO" e a
mensagem a ser codificada seja “PERXES PREPARA UM ATAQUE”. Para isso utilizaremos
a seguinte tabela de valores módulo 31:
A B C D E F G H I
1
2
3
4
5
6
7
8
J K L M N O P
9 10 11 12 13 14 15 16
Q R S T U V W X Y Z ? Á Ã É []
17 18 19 20 21 22 23 24 25 26 27 28 29 30 0
Para codificar a mensagem, podemos escrever as letras da palavra-chave quantas vezes for
preciso acima da frase:
G R E G O G R E G O G R E G O G R E G O G R E G
P
E R X E
S
P
R E P
A R A
U M
A T A Q U E
isto equivale a fazer os seguintes cálculos
7
18
5
7
15
7
18
5
7
15
7
18
5
7
15
7
18
5
7
15
7
18
5
7
16
5
18
24
5
19
0
16
18
5
16
1
18
1
0
21
13
0
1
20
1
17
21
5
0
5
8
4
8
4
26
12
E
H
D
H
D
Z
L
Somando termo a termo mod 31 (Ci
23
23
23
W
W
W
0
(Pi + ai)), obtemos:
20
26
18
21
25
20
23
19
23
8
15
28
T
Z
R
U
Y
T
W
S
W
H
O
Á
Para decifrar a mensagem WWW TZRUYTWSWHOÁ EHDHDZL
basta fazer a operação inversa (Pi (Ci – ai)).
180
CAPÍTULO 12
CIFRA DE VIGENÈRE
Exercícios:
Para as questões abaixo, utilize a seguinte tabela de conversão:
A
B
C D E
F
G H
I
1
2
3
4
6
7
9 10 11 12 13 14 15 16
Q R
S
T U V W X Y Z
5
8
J
K L M N O
?
P
Á Ã É []
17 18 19 20 21 22 23 24 25 26 27 28 29 30 0
Tabela de conversão de caracteres para a Cifra de Vigenère (mod 31).
1) Decifre a mensagem abaixo, usando a palavra-chave CERA:
QIOETGZTQHVIZSLARIISKE
2) Descubra a mensagem a seguir, capturada na transmissão do inimigo.
TXBQMXWFRSIIMOETVOEKMENAVKYHAVT
A palavra-chave utilizada foi TEBAS.
Codifique a mensagem “RIO DE JANEIRO” utilizando a chave TRIUNFO.
Codifique a mensagem “FRONTEIRAS DO BRASIL” utilizando a chave TIGRE.
Decodifique a mensagem “DYDSSX YNYFS” sabendo que a chave BYTE foi utilizada.
Decodifique a mensagem “XIDHY OLOWFBRL” sabendo que a chave MISTURA foi
utilizada.
7) Dada a palavra em texto simples “ILUSTRE”, e o seu respectivo texto cifrado “MCI
FYS”, encontre a chave que foi utilizada na cifragem.
8) Dada a mensagem em texto simples “CRISE DOS INFERNOS”, e o seu respectivo texto
cifrado “EEVHYHVGSBW VYZEGS”, encontre a chave que foi utilizada na cifragem.
9) Desafio: dada a mensagem
OSFYSRWXMFTANASWVNSSDMJMVAMVSISINQBLUDB”,decodifique-a e
encontre a chave que foi utilizada na cifragem.
3)
4)
5)
6)
181
Capítulo 13
CIFRA DE HILL
Uma das desvantagens de cifras de substituição é que elas preservam as frequências de letras
individuais, tornando relativamente fácil quebrar o código por métodos estatísticos. Uma
maneira de superar este problema é dividir o texto em grupos de letras e criptografar o texto
comum por grupo, em vez de uma letra de cada vez. Um sistema poligráfico é um sistema de
criptografia no qual o texto comum é dividido em conjuntos de n letras, cada um dos quais é
substituído por um conjunto de n letras cifradas. Veremos uma classe de sistemas poligráficos
chamados cifras de Hill (Em 1929 Lester S. Hill publica seu livro Cryptography in an
Algebraic Alphabet, no qual um bloco de texto claro é cifrado através de uma operação com
matrizes).
Daqui em diante, nós vamos supor que cada letra de texto comum e de texto cifrado, tem um
valor numérico que especifica sua posição no alfabeto padrão (Tabela 1). Utilizaremos o
símbolo [ ] para indicar um espaço entre as letras ou palavras.
A
1
N
14
B
2
O
15
C
3
P
16
D
4
Q
17
E
5
R
18
Tabela 1
F
G
H
6
7
8
S
T
U
19
20
21
I
9
V
22
J
10
W
23
K
11
X
24
L
12
Y
25
M
13
Z
0
Iniciemos com o caso mais simples de cifras de Hill que utiliza matrizes 2x2.
Usando a tabela acima em  26 transformaremos pares sucessivos de texto comum em texto
cifrado pelo seguinte procedimento:
Passo 1. Escolha uma matriz A, 2 x 2, com entradas inteiras para efetuar a codificação. A
matriz deve ser inversível módulo 26 .
Passo 2. Agrupe letras sucessivas de texto comum em pares, adicionando uma letra fictícia
para completar o último par se o texto comum tem um número ímpar de letras; substitua cada
letra de texto comum por seu valor numérico.
Passo 3. Converta cada sucessivo p1 p2 de letras de texto comum em um vetor-coluna p, e
forme o produto A.p. Nós chamamos p de vetor comum e Ap o correspondente vetor
cifrado.
182
CAPÍTULO 13
CIFRA DE HILL
Passo 4. Converta cada vetor cifrado em seu equivalente alfabético.
Em todas as operações converter os valores para congruência módulo 26.
Exemplo 13.1: Cifra de Hill de uma Mensagem
Use a matriz A=
1 2
0 3
para obter a cifra de Hill da mensagem de texto comum
PARAFUSO
Solução.
Se nós agruparmos o texto comum em pares de letras, obteremos
PA RA FU SO
ou, equivalentemente, usando a Tabela 1,
16 -1 18 - 1 6 -21 19 -15
Para codificar o par PA nós efetuamos o produto matricial
1 2
16
0 3
1
=
18
3
(mod 26)
que fornece o texto cifrado RC pela Tabela 1.
Para codificar o par RA nós efetuamos o produto matricial
1
2
18
0
3
1
=
20
3
(mod 26)
que fornece o texto cifrado TC.
Os cálculos para os demais vetores cifrados são
1 2
6
0 3
21
1 2
19
0 3
15
=
=
48
63
49
45
=
=
22
11
23
19
(mod 26)
(mod 26)
Estes vetores correspondem aos pares de textos cifrado VK e WS, respectivamente.
Coletando os pares, obtemos a mensagem cifrada completa
183
CAPÍTULO 13
CIFRA DE HILL
RCTCVKWS
Como o texto comum foi agrupado em pares e criptografado por uma matriz 2 x 2, dizemos
que a cifra de Hill do Exemplo 1 é uma 2-cifra de Hill. Evidentemente também é possível
agrupar o texto comum em ternos e criptografar com uma matriz 3 x 3 com entradas inteiras;
isto é chamado uma 3-cifra de Hill. Em geral, para uma n-cifra de Hill agrupamos o texto
comum em conjunto de n letras e codificamos com uma matriz codificadora n x n de
entradas inteiras.
Decifrando
Cada cifra útil deve possuir um procedimento para decifrar. Para decifrar as cifras de Hill,
usamos a inversa (mod m) da matriz codificadora. Para ser preciso, se m é um inteiro positivo,
dizemos que uma matriz A com entradas em Zm é inversível módulo m se existir uma matriz
B com entradas em Zm tal que
A . B = B . A = I (mod m)
Suponha agora que
A=
a11
a12
a21 a22
é inversível módulo m e que esta matriz é usada para uma 2-cifra de Hill. Se
p=
p1
p2
é um vetor comum, então
c=Ap
é o correspondente vetor cifrado e
p = A-1 c
Assim, cada vetor comum pode ser recuperado do correspondente vetor cifrado pela
multiplicação à esquerda por A-1 (mod m).
Em criptografia é importante saber quais matrizes são inversíveis módulo m e como obter
suas inversas. Em seguida investigaremos estas questões.
Em aritmética comum, uma matriz quadrada A é inversível se, e somente se, det (A) 0 ou,
equivalentemente, det (A) tem um inverso. O teorema seguinte é o análogo deste resultado em
aritmética modular.
184
CAPÍTULO 13
CIFRA DE HILL
Teorema 13.1 Uma matriz quadrada A em Zm é inversível módulo m se, e somente se, o
detA( mód m) tem um inverso módulo m.
Desse modo, em particular, temos que, se
a b
A=
c d
tem entradas em Zm se o det(A) = ad – bc (mod m) for relativamente primo com m, então a
inversa de det(A) (mod m) é dada por
A-1 = (ad-bc)-1
d
b
c
a
(mod m)
(I)
Onde (ad – bc)-1 (mod m) é o inverso de ad – bc (mod m).
Exemplo 13.2. Decifrando uma 2-Cifra de Hill
Decifre a seguinte 2-cifra de Hill, que foi dada no Exemplo 1: RCTCVKWS
Solução:
Pela Tabela 1, o equivalente numérico do texto cifrado é
18-3 20-3 22-11 23-19
para obter os pares de texto comum, multiplicamos cada vetor cifrado pela inversa de A:
det(A) = 1.3 – 2.0 = 3
temos que o inverso de 3 módulo 26 é igual 9, ou seja 3.9 1(mod 26).
Assim, por ( I ),
A-1 = 9
3
2
0
1
=
27
18
0
9
=
1 8
0 9
(mod 26)
Logo
1 8
18
0 9
3
1 8
20
0 9
3
=
=
42
27
44
27
=
=
16
1
18
1
(mod 26)
(mod 26)
185
CAPÍTULO 13
CIFRA DE HILL
1 8
22
0 9
11
1 8
23
0 9
19
110
=
99
=
175
171
=
=
6
21
19
15
(mod 26)
(mod 26)
Pela Tabela 1, pode-se ver que os equivalentes alfabéticos destes vetores são
PARAFUSO
Matrizes Inversas Módulo m
Teorema: Se M é uma matriz quadrada de ordem n e det M 0, então a inversa de M módulo
mé
M-1 = (det M )-1. M (mod m)
onde M é a matriz adjunta de M.
Exemplo 13.4:
9 0 5
Utilizando a matriz A
8 7 6 (mod 29) como chave, codifique e decodifique a mensagem
3 2 0
MATEMÁTICA em  29 , tal que
A
1
N
14
Á
27
B
2
O
15
É
28
C
3
P
16
[]
0
D
4
Q
17
E
5
R
18
F
6
S
19
G
7
T
20
H
8
U
21
I
9
V
22
J
10
W
23
K
11
X
24
L
12
Y
25
M
13
Z
26
Solução: Convertendo a mensagem:
13
MAT
1
20
; EMÁ
5
20
13 ; TIC
9
27
3
1
; A_ _
0
0
Para codificarmos a mensagem, basta multiplicar cada matriz 3x1 obtida pela matriz
codificadora A:
186
CAPÍTULO 13
CIFRA DE HILL
9 0 5 13
217
14
8 7 6 1
231
28 (mod 29)
3 2 0 20
41
12
9 0 5 5
180
6
8 7 6 13
293
3 (mod 29)
3 2 0
41
12
9 0 5 20
195
21
8 7 6 9
241
9
3 2 0 3
78
20
27
9 0 5 1
(mod 29)
9
8 7 6 0 = 8 (mod 29)
3 2 0 0
3
A mensagem codificada é: NÉLFCLUITIHC
DECODIFICAÇÃO:
Para decodificar precisamos da matriz inversa de A.
Cálculo da matriz inversa A 1 (mod 29):
2- Det (A) = -133, ou seja, Det (A) = 12 ( mod 29). Temos que
1
Det ( A)
1
(mod 29) 17 .
12
De fato:
12 (29) 1 17(mod 29) 1227 1(mod 29) , pois
123 17 129 173 12 1227 123 17(mod 29)
3- Matriz dos Cofatores: C
12 18 -5
17 18 24
10 -15 -18
10 14 11 (mod 29)
35 -14 63
23 15 5
4- A Matriz Adjunta ( M ) é a transposta da Matriz dos Cofatores:
17 10 23
M
C
t
18 14 15 (mod 29)
24 11 5
5- Matriz Inversa A 1 =
A
1
1
.M (mod 29), temos:
Det ( A)
17 10 23
289 170 391
28 25 14
17. 18 14 15
306 238 255
16 6 23 (mod 29)
408 187 85
2 13 27
24 11 5
Basta, então, multiplicar a mensagem codificada pela matriz inversa para obtermos a
mensagem original.
187
CAPÍTULO 13
CIFRA DE HILL
Exercícios:
Z32
A
1
N
14
Ê
27
B
2
O
15
?
28
C
3
P
16
Á
29
D
4
Q
17
Ã
30
E
5
R
18
É
31
F
6
S
19
[]
0
G
7
T
20
H
8
U
21
I
9
V
22
J
10
W
23
K
11
X
24
L
12
Y
25
M
13
Z
26
Tabela 1
1. Utilizando a tabela 1 e as matrizes dadas como chave, codifique e decodifique as
mensagens correspondentes, pelo método de Hill
a) A
b) A
c)
B
1 3
2 7
9 15
19
2
7
8
, LEÃO AZUL
1
12 23 14 , MATEMÁTICA É LEGAL
22
d)
, O PIOR CEGO É AQUELE QUE NÃO ENXERGA O QUE VÊ
4
21
1 2 7
B
0 3 1 , CATACLISMÁTICO
0 5 2
 Para as questões de 2 a 8, utilize a seguinte tabela de conversão:
A
1
N
14
B
2
O
15
C
3
P
16
D
4
Q
17
E
5
R
18
F
6
S
19
G
7
T
20
H
8
U
21
I
9
V
22
J
10
W
23
K
11
X
24
Tabela de conversão de caracteres módulo 26.
2. Codifique a palavra AFRODITE utilizando a matriz
10 11
9 12
14 13 11
3. Codifique a palavra ELETRICIDADE utilizando a matriz 15 17 18
19 16 12
188
L
12
Y
25
M
13
Z
0
CAPÍTULO 13
CIFRA DE HILL
20 21
4. Decodifique a palavra KJQCDYVU sabendo que a matriz
foi utilizada na
19 22
cifragem.
2 6 3
5. Decodifique a palavra GEIOHSSFGQUD sabendo que a matriz 1 7 4 foi utilizada
8 9 5
na cifragem.
6. Decodifique a mensagem
2 3 1 4
11 13
16
9
8
7
5
12
14 15
6
ODNINDTYTXPWVZPQ
sabendo
que
a
matriz
foi utilizada na cifragem.
10
7. Dada a mensagem em texto simples MITO, e o seu respectivo texto cifrado UQNI,
encontre a matriz 2 x 2 que foi utilizada na cifragem.
8. Dada a mensagem em texto simples ALGORITMO e o seu respectivo texto cifrado
ODVMFFYWS. Encontre a matriz 3 x 3 que foi utilizada na cifragem, sabendo que todos
os elementos desta matriz são inteiros positivos distintos menores ou iguais a 9.
 Para as questões de 9 e 10, utilize a seguinte tabela de conversão:
A
1
M
13
Y
25
B
2
N
14
Z
26
C
3
O
15
Á
27
D
4
P
16
É
28
E
5
Q
17
Í
29
F
6
R
18
Ó
30
G
7
S
19
Ú
31
H
8
T
20
Â
32
I
9
U
21
Ô
33
J
10
V
22
Ã
34
K
11
W
23
Õ
35
L
12
X
24
Ç
36
*O espaço em branco é representado pelo valor 0 (zero).
Tabela de conversão extendida de caracteres módulo 37.
9. Codifique a palavra EQUAÇÕES utilizando a matriz
31 33
29 35
26 28 29
10. Decodifique a palavra PBGEZJÃÁÓÍMJ sabendo que a matriz
21 25 27
foi
22 23 24
utilizada na cifragem.
189
Capítulo 14
RSA
Três americanos desenvolveram um sistema de código secreto, chamado RSA, baseado nas
dificuldades existentes para descobrir os fatores primos de um número muito grande. Criavase um novo ramo da Criptografia, a ciência dos códigos, fortemente baseado na Teoria dos
Números. Com o advento dos computadores e da computação algébrica, a Criptografia
ganhou um novo impulso. Neste momento, a proliferação de senhas bancárias e de cartões de
crédito, bem como a crescente necessidade de criptografar dados confidenciais que inundam a
Internet, fazem da Criptografia um dos ramos mais pesquisados da Matemática.
O sistema RSA, batizado em homenagem a seus inventores Ronald Rivest, Adi Shamir e
Leonard Adleman, foi o primeiro criptossistema de chave pública e ainda é o mais importante.
Sua segurança está intimamente relacionada à dificuldade de encontrar a fatoração de um
número inteiro positivo composto, que é o produto de dois primos gigantes.
14. 1. PRÉ-CODIFICAÇÃO
Em primeiro lugar, devemos converter a mensagem em uma sequência de números. Essa
primeira etapa é chamada de pré-codificação. Há várias maneiras de se fazer isso. Aqui
vamos supor que o texto não contém acentuação, pontuação, números etc, apenas as letras A a
Z (maiúsculas). Também vamos adicionar espaços em branco entre palavras, que será
substituído pelo número 99. A letra A será convertida no número 10, B será 11 e assim por
diante, até o Z correspondendo ao número 35. Observe que cada letra corresponde a um
número com exatamente dois algarismos. Isso evita ambigüidades.
A
B
C
D
E
F
G
H
I
J
K
L
M
10
11
12
13
14
15
16
17
18
19
20
21
22
N
O
P
Q
R
S
T
U
V
W
X
Y
Z
23
24
25
26
27
28
29
30
31
32
33
34
35
[]
99
A chave pública é um número n = p . q, onde p e q são primos. Antes de começar devemos,
então escolher esses números. O último passo da pré-codificação é quebrar a mensagem em
blocos. Esses blocos devem ser números menores que n. A maneira de escolher os blocos não
é única, mas é importante evitar duas situações:
Nenhum bloco deve começar com o número 0 (problemas na decodificação).
Os blocos não devem corresponder a nenhuma unidade lingüística (palavra, letra, etc).
Assim a decodificação por contagem de freqüência fica impossível.
190
CAPÍTULO 14
RSA
14.2 – Codificando e decodificando
Para codificar a mensagem precisamos de n = p . q e de um inteiro positivo e (1 < e < (n))
que seja inversível módulo (n). Em outras palavras,
m.d.c.(e, (n)) = m.d.c. (e, (p – 1).(q – 1)) = 1
Note que e é sempre ímpar, dado que p – 1 é par.
Chamaremos o par (n, e) de chave de codificação do sistema RSA.
Codificaremos cada bloco de mensagem separadamente e a mensagem codificada será a
seqüência de blocos codificados.
Vamos agora mostrar como codificar cada bloco b. Chamaremos o bloco codificado de C(b).
Em primeiro lugar, lembre-se que b é menor que n. Então:
C(b)
onde 0
be (mod n)
C(b) < n.
Exemplo 14.1: Considere a frase Paraty é linda. Convertendo em números,
25 10 27 10 29 34 99 14 99 21 18 23 13 10
Agora devemos escolher n.
Vamos começar com um número pequeno, por exemplo, n = 11.13 = 143.
Podemos então quebrar a mensagem acima em blocos, que devem ter valor menor que 143 e
não devem iniciar com zero.:
25 – 102 – 7 – 102 – 93 – 49 – 91 – 49 – 92 – 118 – 23 – 13 – 10
Então temos que (143) = 10.12 = 120 e portanto vamos escolher e como o menor primo que
não divide 120. O valor é 7. Logo,
C(25)
C(25)
C(25)
C(25)
C(25)
C(25)
C(25)
257 (mod 143)
254 . 252 . 251 (mod 143)
254 . 53 . 25 (mod 143)
532 . 53 . 25 (mod 143)
92 . 53 . 25 (mod 143)
14 . 25 (mod 143)
64 (mod 143)
Procedendo dessa maneira com todos os blocos, obtemos a seguinte mensagem cifrada:
64 – 119 – 6 – 119 – 102 – 36 – 130 – 36 – 27 – 79 – 23 – 117 – 10
Vejamos agora como proceder para decodificar um bloco de mensagem codificada.
191
CAPÍTULO 14
RSA
A informação que precisamos para decodificar está contida no par (n, d), onde d (1<d< (n)) é
o inverso de e módulo (n) (e.d
1 (mod (n)).
Chamaremos (n, d) de chave de
decodificação e D(c) o resultado do processo de decodificação. D(c) é dado por:
D(c)
cd (mod n)
onde 0 D(c) < n.
Observe que é possível calcular d, desde que (n) e e sejam conhecidos. Entretanto, se não
conhecemos p e q é praticamente impossível calcular d. Voltando ao nosso exemplo, temos
que n = 143 e e = 7. Para calcular d, usamos o Teorema de Euler:
d.e
1 (mod (n))
d
e
( (n)) – 1
(mod (n))
ou seja,
d
d
d
d
d
7 ( (143)) – 1 (mod (143))
7 (120) – 1 (mod 120)
732 – 1 (mod 120)
731 (mod 120)
103 (mod 120)
Como exemplo, deciframos o primeiro bloco da mensagem cifrada:
D(64)
D(64)
D(64)
D(64)
D(64)
D(64)
D(64)
D(64)
D(64)
D(64)
64103 (mod 143)
64100 . 643 (mod 143)
(6410)10 . 643 (mod 143)
((645)2)10 . 643 (mod 143)
((642 . 643)2)10 . 643 (mod 143)
((92 . 25)2)10 . 25 (mod 143)
((12)2)10 . 25 (mod 143)
(144)10 . 25 (mod 143)
(1)10 . 25 (mod 143)
25 (mod 143)
Os outros blocos ficam como exercícios.
A pergunta óbvia que surge agora é: D(C(b)) = b?
Ou seja, decodificando um bloco de mensagem codificada, encontramos um bloco da
mensagem original? Porque senão todo nosso esforço foi sem sentido. Vamos mostrar nessa
seção que a resposta para esta pergunta é: sim!
Consideremos então n = p . q. Vamos provar que D(C(b)) b (mod n)
192
CAPÍTULO 14
RSA
E por que não a igualdade D(C(b)) = b (mod n)? Observe que D(C(b)) e b são menores que (n
– 1). Por isso escolhemos b menor que n e mantivemos os blocos separados depois da
codificação. Pela definição de D e C temos que
(be)d
D(C(b))
bed (mod n)
Mas d é o inverso de e módulo (n). Logo, existe inteiro k tal que e.d = 1 + k. (n). Logo,
bed
b1 + k.
(n)
b . (b
(n) k
) (mod n)
Se m.d.c.(b, n) = 1, então podemos usar o teorema de Euler:
bed
(1)k . b
b (mod n)
Se b e n não são primos entre si, observe que n = p . q, p e q primos distintos. Logo,
bed
b1 + k.
(n)
b1 + k.(p-1).(q-1)
(b(p-1))k.(q-1). b (mod n)
Se m.d.c.(b, p) = 1, então podemos usar o teorema de Fermat [bp-1 1 (mod p)].
Caso contrário, temos que p | b e portanto
bed
b
0 (mod p)
Logo,
bed
b (mod p)
qualquer que seja b. Fazemos o mesmo para o primo q, obtendo:
bed
b (mod q)
Portanto,
bed b (mod p.q)
bed b (mod n)
c.q.d.
Exemplo 14.2:
A
B
C
D
E
F
G
H
I
J
K
L
M
10
11
12
13
14
15
16
17
18
19
20
21
22
N
O
P
Q
R
S
T
U
V
W
X
Y
Z
23
24
25
26
27
28
29
30
31
32
33
34
35
[]
99
Digamos que você está interessado em enviar a mensagem
193
CAPÍTULO 14
RSA
FINALMENTE
Você deve inicialmente numerar toda a palavra de acordo com a tabela acima:
15 18 23 10 21 22 14 23 29 14
Agora, suponha que a chave seja (n, e) = (7663, 17)
Então você deve escolher blocos numéricos para codificar que sejam menores que n e que não
comecem com zero.
Escolhamos, por exemplo, os seguintes blocos (veja que isso vai depender da máquina que
você está trabalhando):
151 – 823 – 102 – 122 – 142 – 329 – 14
Basta agora pegar cada bloco e elevar à potência e módulo n:
15117 5137 (mod 7663)
15117 5137 (mod 7663)
82317 3345 (mod 7663)
10217 1635 (mod 7663)
12217 3009 (mod 7663)
14217 2186 (mod 7663)
32917 3890 (mod 7663)
1417 2032 (mod 7663)
A mensagem codificada é
5137 – 3345 – 1635 – 3009 – 2186 – 3890 – 2032
Agora vejamos como fazer se recebemos uma mensagem codificada por alguém utilizando a
chave pública (n, e) = (7663, 17). Suponha que a mensagem recebida seja a mesma obtida
acima:
5137 – 3345 – 1635 – 3009 – 2186 – 3890 – 2032
Para entender o que significa você precisa da chave privada d, que é inversa de e (mod ( n )).
Etapas:
1. Primeiro, fatoramos o número n = 7623 ( veja abaixo a fatoração pelo método de Fermat).
Encontramos os primos 79 e 97.
2. Calculamos (n) = (7663) = (79) . (97) = 78.96 = 7488.
194
CAPÍTULO 14
RSA
3. Calculamos d
e
( (n)) – 1
(mod (n))
17
(7488) – 1
(mod 7488)
881 (mod 7488).
4. Agora basta pegar cada bloco que recebemos e elevar à potência d módulo n.
5137881
3345881
1635881
3009881
2186881
3890881
2032881
151 (mod 7663)
823 (mod 7663)
102 (mod 7663)
122 (mod 7663)
142 (mod 7663)
329 (mod 7663)
14 (mod 7663)
5. Então colocamos os resultados alinhados
151 – 823 – 102 – 122 – 142 – 329 – 14
depois separamos em blocos de dois números
15 18 23 10 21 22 14 23 29 14
que corresponde a mensagem enviada
FINALMENTE
14. 3. Assinatura digital utilizando a criptografia RSA
1º passo: Como a segurança do RSA reside na dificuldade de fatorar números que são
produtos de 2 primos; então é aconselhável escolher primos bem grandes (10 ou mais dígitos).
Mas, para fins didáticos vamos escolher primos relativamente pequenos (menores que 1000),
cuja idéia é a mesma para primos grandes.
Ainda, para efeitos deste exemplo, chamaremos o remetente de Alice (A) e o destinatário de
Bob (B). Para elaborarmos as chaves públicas e privadas do esquema, então escolhemos
adequadamente dois primos p e q para cada participante, e calculamos o produto n = p . q.
Os números primos de Alice são: pA = 859, qA = 547. Que resulta em nA = 469873.
Os números primos de Bob são: pB = 937, qB = 461. Que resulta em nB = 431957.
*Observação 1: os primos p e q não devem ser muito próximos para evitar fatorações rápidas como a de Fermat.
2º passo: O número n encontrado desta maneira faz parte tanto da chave pública como da
chave privada de cada participante. Agora, escolhemos um número e que também será público
e que seja relativamente primo à função de Euler (n)
MDC( (n), e) = 1.
195
CAPÍTULO 14
RSA
Para Alice, (nA) = (859) . (547) = 858 . 546 = 468468.
Uma boa escolha para Alice é o número primo eA = 5, pois o MDC(468468, 5) = 1.
Para Bob, (nB) = (937) . (461) = 936 . 460 = 430560.
Uma boa escolha para Bob é o número primo eB = 7, pois o MDC(430560, 7) = 1.
*Observação 2: a chave pública e não é necessariamente um número primo, este exemplo foi apenas uma mera coincidência.
3º passo: Tendo posse de (n) e da chave pública e, podemos calcular a chave secreta d.
Sendo que d é o inverso multiplicativo de e, ou seja, e.d 1 (mod (n)). Pelo teorema de
Euler, d e ( ( n)) 1(mod (n)) . Fazendo os devidos cálculos encontramos que:
Para Alice, 5. dA 1 (mod 468468) dA = 281081.
Para Bob, 7. dB 1 (mod 430560) dB = 307543.
*Observação 3: uma forma computacional rápida para se encontrar a chave privada d é:
int d = 0; for ( d = 2; d < phi; d++ ) {if ( (e * d) % phi == 1 ) ) {System.out.println( “Inverso = ” + d ); break; // d encontrado, sai do laço}}
Pronto, já temos posse das chaves necessárias para cifrar e decifrar mensagens.
O que deve ser disponibilizado nas páginas amarelas ou qualquer diretório público:
Chave pública de Alice (nA, eA) = (469873, 5).
Chave pública de Bob (nB, eB) = (431957, 7).
O que não deve ser disponibilizado, pois é secreto e pertence somente aos seus donos:
Chave privada de Alice (nA, dA) = (469873, 281081).
Chave privada de Bob (nB, dB) = (431957, 307543).
*Observação 4: note que tanto os primos p e q, assim como a função (n), também devem ser mantidos secretos!
Pois a partir de p e q, pode-se facilmente calcular (n) = (p – 1) * (q – 1). E vice-versa, a partir de (n) pode-se calcular p e q através de um
sistema linear e uma simples equação do 2º grau. Tendo posse de (n), calcula-se a chave privada d como demonstrado.
4º passo: Finalmente, como fazer para criptografar uma mensagem? Por exemplo, tomemos a
palavra “BRASIL”, que no código ASCII corresponde aos valores:
(66, 82, 65, 83, 73, 76)
Concatenando esses valores, obtemos 668265837376.
Reagrupando em blocos (de qualquer tamanho), obtemos 668, 265, 837, 376.
O valor contido em cada bloco não deve exceder o valor da chave pública n. Também se deve
tomar cuidado para que nenhum bloco inicie com o valor zero, com risco de se criar um erro
na cifragem. Os blocos evitam ataques por freqüência, já que se a mensagem fosse codificada
caractere por caractere, o RSA se tornaria uma mera cifra de substituição.
A codificação dos blocos é feita, elevando-se cada valor do bloco ao expoente público e
módulo n do destinatário, obtendo-se o menor valor positivo correspondente. Assim, se Alice
deseja enviar a palavra “BRASIL” para Bob, ela deve utilizar a chave pública (nB, eB) =
(431957, 7) de Bob, e efetuar C(b) = be (mod n), da seguinte forma:
196
CAPÍTULO 14
RSA
C(668) = 6687 (mod 431957)
C(265) = 2657 (mod 431957)
C(837) = 8377 (mod 431957)
C(376) = 3767 (mod 431957)
331318 (mod 431957)
387742 (mod 431957)
213180 (mod 431957)
121902 (mod 431957)
Alice, então, deve enviar a mensagem criptografada
(331318, 387742, 213180, 121902)
para Bob.
5º passo: Para decifrar a mensagem, Bob deve proceder de maneira inversa, utilizando a sua
chave secreta (nB, dB) = (431957, 307543), ou seja, D(c) = cd (mod n), da seguinte forma
(geralmente não é uma congruência fácil de resolver...):
D(331318) = 331318307543 (mod 431957)
D(387742) = 387742307543 (mod 431957)
D(213180) = 213180307543 (mod 431957)
D(121902) = 121902307543 (mod 431957)
668 (mod 431957)
265 (mod 431957)
837 (mod 431957)
376 (mod 431957)
Bob reagrupa a mensagem decodificada 668265837376, separa em pares,
(66, 82, 65, 83, 73, 76)
e obtém a mensagem original “BRASIL”.
Propriedades matemáticas garantem que aplicando a função de cifragem e depois decifragem,
a mensagem original continua a mesma, isto é, b = D(C(b)).
Assinatura: Assinar uma mensagem é garantir que ela realmente vem de quem diz ser o
remetente.
Como podemos assinar uma mensagem utilizando o RSA?
No exemplo, anterior, perceba que em nenhum momento chegamos a usar as chaves do
remetente (Alice), portanto é chegada a hora.
Devido às propriedades matemáticas das chaves públicas e privadas e e d, Alice pode utilizar
sua própria chave privada dA “decodificar” a mensagem original ao invés de normalmente
utilizá-la para decifrar uma mensagem cifrada com sua chave pública destinada a ela.
Isto faria com que a mensagem ficasse cifrada com a sua chave privada, e como isto seria
útil? Antes de partimos para o exemplo numérico, uma breve teoria.
O que foi feito anteriormente (no exemplo de Bob):
b  CB(b)  DB(CB(b))  b
197
CAPÍTULO 14
RSA
onde b é a mensagem em texto puro “BRASIL”, CB( ) é a função de cifragem que utiliza a
chave pública de Bob, e DB( ) é a função de decifragem que utiliza a chave secreta de Bob.
O esquema de assinatura digital consiste em
b  DA(b)  CB(DA(b))  DB(CB(DA(b)))  DA(b)  CA(DA(b))  b
Em outras palavras, Alice utiliza sua chave privada para “decodificar” a mensagem original
(que inicialmente não é uma ação muito útil, pois qualquer pessoa com conhecimento da
chave pública de Alice pode desfazer esta ação). Em seguida, Alice cifra o resultado com a
chave pública de Bob. Neste ponto, o que temos? Uma mensagem-cápsula duplamente
criptografada com a chave privada de Alice e a chave pública de Bob.
CB(DA(b)) = mensagem assinada
O que isto garante? Que Bob pode primeiramente decodificar esta mensagem-cápsula com
sua própria chave privada, e assim obter de volta a mensagem criptografada com a chave
privada de Alice. Mas aqui que surge o grande ponto: se Bob utilizar a chave pública de
Alice, ele obterá a mensagem original!
Como ninguém mais além de Alice possui a chave privada dA, então realmente aquela
mensagem foi codificada inicialmente por Alice. A outra garantia vem de que somente Bob
sabe disto, pois Alice sabiamente também utilizou a chave pública de Bob.
Em resumo, no esquema tradicional (sem assinatura), b = D(C(b)).
Entretanto, devido às propriedades matemáticas, também é verdade que b = C(D(b)).
Ou seja, no esquema assinado temos, b = CA(DB(CB(DA(b)))), onde as funções são aplicadas
de dentro para fora na ordem que aparecem. Agora o exemplo numérico.
Relembrando as chaves públicas de Alice e Bob:
Chave pública de Alice (nA, eA) = (469873, 5).
Chave pública de Bob (nB, eB) = (431957, 7).
Chave privada de Alice (nA, dA) = (469873, 281081).
Chave privada de Bob (nB, dB) = (431957, 307543).
1º passo: Alice utiliza sua chave privada para “decodificar” a palavra “BRASIL” separada em
blocos como no exemplo anterior (as mesmas observações de tamanho de bloco e valores
menores que os módulos também valem para este caso):
DA(668) = 668281081 (mod 469873)
DA(265) = 265281081 (mod 469873)
DA(837) = 837281081 (mod 469873)
DA(376) = 376281081 (mod 469873)
198
249014 (mod 469873)
82081 (mod 469873)
221564 (mod 469873)
231544 (mod 469873)
CAPÍTULO 14
RSA
2º passo: Em seguida, Alice pega o resultado e aplica na função de codificação com a chave
pública de Bob:
CB(249014) = 2490147 (mod 431957) 273379 (mod 431957)
CB(82081) = 820817 (mod 431957) 45868 (mod 431957)
CB(221564) = 2215647 (mod 431957) 414208 (mod 431957)
CB(231544) = 2315447 (mod 431957) 363416 (mod 431957)
Assim, Alice envia a mensagem criptografada assinada:
(273379, 45868, 414208, 363416)
3º passo: Bob, ao receber a mensagem criptografada assinada, deve primeiramente aplicar sua
chave privada (é uma calculeira danada, eu sei... Tente usar o computador!):
DB(273379) = 273379307543 (mod 431957) 249014 (mod 431957)
DB(45868) = 45868307543 (mod 431957) 82081 (mod 431957)
DB(414208) = 414208307543 (mod 431957) 221564 (mod 431957)
DB(363416) = 363416307543 (mod 431957) 231544 (mod 431957)
Já aqui neste resultado, Bob deve obter algo que Alice obteve no 1º passo.
4º passo: Finalmente, Bob aplica a chave pública de Alice que garante que foi ela mesma que
enviou esta mensagem (nenhuma outra pessoa além de Alice pode ter a chave privada dela,
lembra?):
CA(249014) = 2490145 (mod 469873) 668 (mod 469873)
CA(82081) = 820815 (mod 469873) 265 (mod 469873)
CA(221564) = 2215645 (mod 469873) 837 (mod 469873)
CA(231544) = 2315445 (mod 469873) 376 (mod 469873)
Novamente, Bob reagrupa a mensagem decodificada 668265837376, separa em pares, (66,
82, 65, 83, 73, 76) e obtém a mensagem original “BRASIL”.
199
CAPÍTULO 14
RSA
Exercícios
 Para os exercícios seguintes utilize a tabela abaixo ou a tabela ASCII
A
B
C
D
E
F
G
H
I
J
K
L
M
10
11
12
13
14
15
16
17
18
19
20
21
22
N
O
P
Q
R
S
T
U
V
W
X
Y
Z
23
24
25
26
27
28
29
30
31
32
33
34
35
[]
99
1) Usando as chaves públicas dadas, codifique e decodifique as frases correspondentes:
a) (n, e) = (1037, 7) SONHO
b) (n, e) = (2201, 11) REMO
c) (n, e) = (1577, 5) ESPERANDO
2) A chave pública utilizada pelo Banco Crash para codificar suas mensagens é a seguinte:
(n, e) = (4559, 5)
Os computadores do banco receberam, de local indeterminado, os seguintes blocos de
mensagem: 2621 – 2608 – 3594 – 4261
O que diz a mensagem?
3) A mensagem
96 – 61 – 751 – 9
foi codificada pelo método RSA usando a chave (n, e) = (767, 13). Decodifique a
mensagem.
4) Sabendo-se que n = 27641 é igual ao produto de dois primos e que
determine os fatores primos de n.
5) Sabendo-se que n = 3552377 é igual ao produto de dois primos e que
determine os fatores primos de n.
( n)
( n)
27300 ,
3548580 ,
6) A mensagem
6802 – 8728 – 9451
foi codificada pelo método RSA usando a chave (n, e) = (16517, 5). Além disso, sabe-se
que (n) 16236 . Decodifique a mensagem.
7) (Assinatura Digital)
Considere que o Banco Bandit possua a seguinte chave pública:
(nB, eB) = (20099, 7)
E a Empresa Explorit possua a seguinte chave pública:
(nE , eE) = (61823, 11)
Demonstre como a empresa enviaria a mensagem MONEY assinada ao banco e como o
banco confirmaria a autenticidade da assinatura.
200
Capítulo 15
PARTILHA DE SENHAS
O Teorema Chinês do Resto é utilizado em sistemas de partilha de senhas entre várias
pessoas, de modo que para cada pessoa seja dado um elemento distinto em função da senha s
a ser compartilhada. A grande vantagem do método de partilha de senhas é que cada pessoa
contém uma chave diferente das outras chaves e uma pessoa sozinha não consegue decifrar a
senha. Outra vantagem é que não é necessária a presença de todas as pessoas com suas
respectivas chaves, bastam que k ou mais pessoas estejam presentes. A chave para cada
pessoa é escolhida dentro um conjunto S composto por n pares de inteiros positivos de forma
que, para cada inteiro positivo k n previamente escolhido, tem-se que:


Qualquer subconjunto de S com k elementos permite determinar s facilmente;
É muito difícil determinar s conhecendo menos que k elementos de S.
O primeiro passo é escolher um conjunto apropriado L de n inteiros positivos distintos, dois a
dois primos entre si. Seja N o produto dos k menores números de L, e M o produto dos k – 1
maiores números de L, os elementos de L devem ser escolhidos com cuidado de forma que M
< N, e a senha s possa ser escolhida arbitrariamente dentro do intervalo M < s < N.
Os elementos até aqui mencionados são:
L = {p1, p2, ..., pn}; onde p1 < p2 < ... < pn são primos distintos entre si.
k = número de pessoas que se deseja estarem presentes para a decifragem da senha.
N = p1.p2. ... .pk = produto dos k menores elementos de L.
M = px.px+1. ... . pn = produto dos k – 1 maiores elementos de L.
s = senha aleatória que não tem relação alguma com L, mas deve ser escolhida de forma que
M < s < N.
Posteriormente, o conjunto gerador de chaves S será constituído pelos pares da forma (p, s p)
onde p L e sp é a forma reduzida de s (mod p). Um limite k 2 implica em s > p para
qualquer p L. Logo sp < s para qualquer p L.
Supõe-se que sejam conhecidos, em um dado momento, t k pares de elementos de S, ou
seja, existem t pessoas presentes para a decifragem da senha s. Denota-se esses pares por (p1,
s1), (p2, s2), ..., (pt, st).
S = {(p1, sp1), (p2, sp2), ..., (pt, spt)}
Para se chegar à senha s é necessário resolver o seguinte sistema de congruências:
x
x
s1 (mod p1)
s2 (mod p2)
...
x st (mod pt)
Pelo Teorema Chinês do Resto, obtém-se x0 como solução, tal que:
x0
s (mod p1.p2. ... .pt)
201
CAPÍTULO 15
PARTILHA DE SENHAS
É sabido que como t k, (p1.p2. ... .pt)
solução menor que (p1.p2. ... .pt).
N > k, e o sistema de congruências tem uma única
Observações:
 É possível escolher os módulos de s de modo que seja impraticável
encontrar s através de uma busca, conhecendo-se apenas uma das chaves;
 É sempre possível escolher um conjunto L que satisfaça todas as condições.
A seguir, um exemplo prático para efeito de esclarecimento.
Exemplo 15.1:
No banco “Golden Luck” há 5 funcionários responsáveis pela manutenção da senha de um
cofre, e pelo menos 2 pessoas (k = 2) têm que estar presentes para a abertura do mesmo.
Logo, o conjunto L deve possuir 5 elementos e seu limiar deve igual a 2. Uma possível
escolha para L envolvendo somente primos pequenos é
L = {11, 13, 17, 19, 23}
A partir do qual se calcula os valores dos limites N e M:
N = 143 = 11.13 = produto dos (k = 2) menores elementos de L.
M = 23 = produto dos (k – 1 = 2-1=1) maiores elementos de L.
O valor da senha s pode ser escolhido aleatoriamente como qualquer inteiro no intervalo que
vai de 23 a 143 (M < s < N). Por exemplo, suponha que a senha seja s = 50. Então o conjunto
S que contêm os elementos da senha é:
S = {(11, 6), (13, 11), (17, 16), (19, 12), (23, 4)}
O segundo termo de cada elemento de S, sm, é o resto da divisão de s = 50 por cada termo
correspondente de L, ou seja, s(mod p). Se os funcionários que possuem as senhas (17, 16) e
(23, 4), por exemplo, estão no banco, para obter a senha seria preciso resolver o sistema:
x
x
16 (mod 17)
4 (mod 23)
Solução:
m = 17. 23 = 391
m
m
M1 =
= 23 ; M2 =
= 17
m1
m2
Os inversos x1 e x2 de M1 e M2 são dados por:
23 x1
202
1 (mod 17) e 17x1
1 (mod 23)
CAPÍTULO 15
PARTILHA DE SENHAS
Aplicando o método de Euler nas equações acima obtemos as soluções respectivas:
x1 = 3 e x2 = 19 .
Portanto, temos
x a1M1x1 + a2M2x2 (mod m)
x 16 . 23 . 3 + 4 . 17 . 19 (mod 391)
x 2396 (mod 391)
x = 50
Assim, determina-se que 50 é o menor valor inteiro positivo congruente a x, que é a senha
correta.
Exercícios
1. Por motivo de segurança o banco “ Golden Luck” trocou a senha do cofre. Dois
funcionários possuem as chaves (13, 5) e (19, 17). Qual a nova senha?
2. Após uma nova troca de senhas no banco “Golden Luck”, 2 funcionários tem agora as
chaves (31, 6) e (41, 20). Qual é a senha agora?
3. Como duas pessoas só sabem guardar um segredo se uma delas já estiver morta, o banco
“Golden Luck” resolveu fazer uma mudança completa no esquema de segurança e
novamente a senha foi trocada, além disso ficou estabelecido que estejam presentes, no
mínimo, 3 pessoas para que o cofre possa ser aberto. Três funcionários estão com as
chaves (53, 21), (61, 9) e (71, 35). Qual é a senha?
203